Hospital Medicine

Ace your homework & exams now with Quizwiz!

What common cardiovascular medications are okay to continue while a patient is doing a stress test?

ACEi. ARBs. Digoxin. Statins. Diuretics.

20 year old woman with pulsatile headaches for 6 weeks that are worse at night and awaken her from sleep. BMI is 35. Sixth nerve palsy on exam. Papilledema on funduscopy. CT is normal. Elevated opening pressure on a lumbar puncture with otherwise normal cerebrospinal fluid analysis. Next best step? If symptoms of are not relieved what is an additional option?

Acetazolamide is the first line treatment for idiopathic intracranial hypertension (pseudotumor cerebri). Furosemide. It works by inhibiting choroid plexus carbonic anhydrase decreasing CSF production.

What medicines help with the treatment of Lewy bodies dementia?

Acetylcholinesterase inhibiters like rivastigmine for motor and psychiatric symptoms. If hallucinations are refractory the use of antipsychotics can help.

50 year old man comes to the emergency department with squeezing chest pain 2 hours ago. Severe dyspnea, worse when lying down. 100/60 pressure and 110 pulse. 3/6 holosystolic murmur at apex and bibasilar crackles in the lungs. STEMI on leads II, III and aVF. Clinical diagnosis and mechanism of disease.

Acute inferior myocardial infarction with papillary muscle displacement, leading to acute mitral regurgitation and pulmonary edema. Physiology findings would include left ventricular end diastolic pressure.

Patient was treated for a skin infection and now has eosinophilia and white blood cell casts on urinalysis. Diagnosis and likely cause?

Acute interstitial nephritis likely due to nafcillin.

70 year old post op patient day 5, new-onset abdominal discomfort. over past 12 hours voided 200 mL of urine. Review of chart shows infrequent recording of I's and O's. Portable bladder scan is inconclusive. Hyperkalemia, hyponatremia with elevated urea nitrogen and creatinine. Clinical problem? Best next step and why? Diagnosis and pathogenesis?

Acute kidney injury with oliguria (oliguria is less than 250 in 12 hours) Bladder catheterization, don't want hydronephrosis, renal injury or tubular atrophy. Post-op urinary retention is a common complication of surgery and anesthesia. Precipitating event is bladder distention during anesthesia which leads to inefficient detrusor muscle activity and acute urinary retention.

30 year old cocaine user found on the floor of his apartment. He is confused. Potassium is 5 and CPK 25,000. Chemistry panel. What is the patient at risk for and why?

Acute renal failure due to rhabdomyolysis because he's been immobilized and has been taking cocaine. Note: elevated CPK and potassium is directly from lysed muscle content.

What are the equations for appropriate acute respiratory acidosis and alkalosis?

Acute respiratory acidosis - increase serum HCO3 by mEq/L for each 10 mm Hg rise in PaCO2. Acute respiratory alkalosis - decrease HCO3 by 2 mEq/L for every 10 mm Hg decrease in PaCO2.

Patient was almost stabbed to death and survived 2 weeks ago. Symptoms of intrusive memories, nightmares and flashbacks of the event. Patient avoids cooking with knives. Diagnosis?

Acute stress disorder.

Muddy brown casts

Acute tubular necrosis, (think "muddy for necrosis")

What defines hyper hemolytic crisis in sickle cell disease?

Acute, severe anemia with reticulocytosis.

How do you treat myasthenia crisis?

Address respiratory failure via endotracheal intubation. Administer corticosteroids and IVIG or plasmapheresis.

Most common lung cancer in nonsmokers.

Adenocarcinoma.

What medication can be given to patients with PSVT and why does this work?

Adenosine. Slows done atrioventricular nodal conduction to interrupt the reentry pathway and terminate PSVT.

Generally what does a BUN/Cr ratio of less than 20 indicate?

Adequate renal perfusion and function.

For asplenic patients, what is the recommended meningitis vaccines?

Administer PCV13, Hib and meningococcal vaccine. Read minister the meningococcal vaccine every 5 years and with the pneumococcal vaccine, a second dose and again at age 65.

Best initial step in a stable patient with chest pain and suspected acute coronary syndrome?

Administer aspirin as soon as possible.

Patient is being worked up for hypertension with hypokalemia. and has an abnormal plasma aldosterone to renin concentration ratio and abnormal adrenal suppression test. Next best step and explain the process of why these things are being done.

Adrenal imaging and or adrenal vein sampling. 1. If discrete unilateral adenoma is present in a patient less than 40 years of age, treat with surgery 2. If normal CT or patient is older than 40 with an abnormal CT, sample the adrenal vein. If unilateral pathology remove with surgery or treat medically if booth glands are involved.

Hyperkalemia, hypoglycemia and eosinophilia. What diseases should you be thinking about?

Adrenal insufficiency. Various granulomatous diseases cause adrenal insufficiency such as: 1. Tuberculosis. 2. histoplasmosis. 3. coccidioidomycosis. 4. crytpococcosis. 5. sarcoidosis.

50 year old hospitalized for renal failure. History of recurrent episodes of bilateral flank pain for several years. Nocturia 3 times per night for 10 years. No weight loss. Hypertension. Enlarged liver and mass felt in right flank. Diagnosis?

Adult dominant polycystic kidney disease.

Crigler-Najjar syndrome type 2, presentation?

Adult with bilirubin levels below 20.

What is acute acalculous cholecystitis? What are some causes for this condition?

An acute inflammation of the gallbladder in the absence of gallstones. 1. Extensive burns. 2. severe trauma. 3. prolonged TPN. 4. prolonged fasting. 5. mechanical ventilation.

Patient with febrile neutropenia is being treated with empiric antibiotic coverage and is now day 4 without improvement with no source of infection identified. What medication would be suggested?

An anti fungal such as voriconazole.

What is edrophonium? Does it treat myasthenia gravis?

An anti-cholinesterase. No, it only helps diagnostically.

60 year old male presents to ER because of syncope. Clonic jerks witnessed while patient was unconscious. History of acute myocardial infarction six months ago and well-controlled hypertension. Patient is on captopril, metoprolol, hydrochlorothiazide, clopidogrel, and aspirin. Ectopic beats. Holosystolic apical murmur radiating to axilla. Most likely cause of syncope?

Arrhythmia likely because of sudden onset of syncope without warning, presence of structural heart disease (post-infarction scar and probable mitral regurgitation) and frequent ectopic beats. Presence of thiazide diuretic may hint at electrolyte disturbance.

What is an "immediate" or phase 1a Ventricular arrhythmia?

Arrhythmias occurring within 10 minutes of coronary occlusion. Heterogenous pockets of ischemia lead to marked conduction slowing and delayed activation, hence reentrant arrhythmia.

What is a "delayed" or phase 1b arrhythmia?

Arrhythmias that occurs 10 to 60 minutes after an acute infarction, which is thought to result from abnormal automaticity.

Patient presents with a concern of a stroke. PMH of atrial fibrillation. Cold right hand and undetectable arterial radial pulse. What is the diagnosis? Treatment?

Arterial occlusion. Immediate vascular surgery consultation and IV heparin bolus followed by infusion.

What complications occur due to heparin induced thrombocytopenia? Pathogenesis?

Arterial thrombosis. Heparin-platelet-PF4 + antibody immune complex will cause... 1. platelet activation leading to procoagulant micro particle release and platelet aggregation, which causes thrombosis. 2. Splenic macrophages remove complex and thrombus formation causes platelet consumption, which causes thrombocytopenia.

For patients who are not treated for Lyme disease for months to years what symptoms will these patients experience?

Arthritis Encephalomyelitis Peripheral neuropathy

What rhythm disturbances is adenosine effective in terminating?

Artrioventricular nodal reentrant tachycardia

Pleural plaques, diagnosis?

Asbestosis.

Flu-like illness, malaise, vomiting photophobia, stupor and fever in July. CSF shows only elevated WBC. Diagnosis? Treatment.

Aseptic meningitis. Supportive care.

What populations are at an increased risk of acute angle-closure glaucoma?

Asian and inuit populations and individuals with farsightedness.

A non smoker patient less than 40 years old. with a 1.5 cm nodule in a region of the lung that is well aerated. Next best step?

Ask for an old X-ray.

Hemoptysis, fever, weight loss and chronic cough. Mobile mass on x-ray.

Aspergilloma.

How would aspergillosis and Blastomycosis in terms of fungal culture and staining?

Aspergillosis would appear as a mold on exam unlike blastomycosis which is a yeast.

Which condition is associated with a risk of dissemination of a liver cyst with aspiration? Echinococcus or E. histolytica?

Aspiration is not recommended with E. histolytica.

60-year old. Chest pain that started 2 hours ago and diaphoresis. Feels week. Diabetes and hypercholesterolemia. Pressure of 160 over 95. ECG shows STEMI. Treatment?

Aspirin and immediate cardiac catheterization.

40 year old lady presents with nausea, fever and upper abdominal discomfort. History includes depression, migraine and rheumatoid arthritis. Tinnitus and vertigo. Admits to overdosing one of her medications. Temp of 38 pulse of 115 and respiration of 24. Likely diagnosis? Expected arterial blood gas findings?

Aspirin intoxication as evidence by her tinnitus, fever, tachypnea, nausea and gastrointestinal irritation in relation to overdose. ABGs should show signs for respiratory alkalosis and metabolic acidosis. Therefore pH should be balanced. Findings should demonstrate low CO2 and low HCO3

Patient who is already on aspirin and is suspicious of having a stroke. What is the best therapy for this patient?

Aspirin plus dipyridamole or clopidogrel.

What therapy does a patient need to be on if they are already on aspirin therapy and have a history of a large intracranial large artery atherosclerosis?

Aspirin with clopidogrel.

What 4 drugs should all patients with an MI receive upon discharge?

Aspirin, beta blocker, ACEi, and statins.

Which combination of analgesics are associated with analgesic nephropathy? What pathologies occur from analgesic nephropathy?

Aspiring and naproxen. 1. Papillary necrosis. 2. Chronic tubulointerstitial nephritis.

Patient is evaluated for RUQ pain. He has NASH. With what information would a bariatric referral be considered?

BMI greater than 35 in this setting.

Tick bite, fever, sweats, jaundice, dark urine, no rash, and history of asplenia in a patient from New York. Diagnosis? Pathogenesis? Confirmatory diagnostic test?

Babesiosis. Parasite enters RBC and causes hemolysis. Giemsa-stained thick and thin blood smear.

Large pedunculated exophytic papule with a collarette of scale in an AIDs patient

Bacillary angiomatosis (Bartonella)

Large, pedunculate exophytic papule with a claret scale which resembling a large pyogenic granuloma or cherry angioma. Diagnosis?

Bacillary angiomatosis.

Bradycardia, AV block, hypotension and diffuse wheezing in a suicide attempt in patient with a long history of coronary artery disease and hypotension. Diagnosis? Management?

Beta blocker overdose. 1. Secure airway, give fluid boluses and IV atropine to treat hypotension and bradycardia. 2. Administer IV glucagon (increaes cyclic AMP, which creates both beta blocker and calcium channel blocker toxicity). 3. Other treatment options - IV calcium, vasopressors, high-dose insulin and glucose, and IV lipid emulsion.

What is the best anti-hypertensive to use when treating an acute aortic dissection? Why?

Beta blocker. It decreases both heart rate and blood pressure, minimizing shear forces to the aorta.

Patient has a history of substance use disorder and hypertension. What antihypertensive do you need to be cautious about prescribing and why?

Beta blockers can exacerbate cocaine intoxication leading to unopposed alpha activation, causing vasoconstriction of the heart.

What medications need to be withheld 48 hours prior to cardiac stress testing? How about prior to a vasodilator stress test at 48 hours and 12 hours?

Beta blockers, calcium channel blockers and nitrates. Dipyridamole. Caffeine-containing food or drinks.

Contraindication to cocaine use?

Beta blockers.

Bacterial endocarditis is expected. First best step?

Blood culture.

What is the protein diet goals for patients with diabetic nephropathy and azotemia?

0.8 g/kg/day

What size of kidney stones need surgical intervention?

1 cm or greater.

Generally how long does it take for drug exposure to cause an acute kidney injury?

1 to 2 days.

What is the prevalence of renal artery stenosis with the following: Mild hypertension? Severe Hypertension? Peripheral artery disease?

1%. 45%. 25-35%.

An intrauterine pregnancy should be seen via trans-vaginal ultrasound at what levels of beta-HCG?

1,500-2000

What patients are at high risk for influenza complications?

1. >65 years of age. 2. pregnant woman & those up to 2 weeks postpartum. 3. Chronic medical illness (pulmonary, cardiovascular, renal and hepatic). 4. immunosuppression. 5. morbid obesity. 6. Native americans. 7. nursing home or chronic care facility residents

How does one calculate the A-a gradient? What diagnostic purpose does a normal A-a gradient serve? How does one calculate PAO2?

1. A-a gradient = PAO2 - PaO2 2. A normal gradient of less than 15 is evident in diseases due to reduced inspired oxygen tension and hypoventilation. 3. PAO2 = (FiO2 times Patm - PH2O - PaCO2/R... "or" 0.21 times 13 minus PaCO2/0.8

With acute pancreatitis, what complications can occur other than pathologies of the pancreas? (name 4)

1. ARDS. 2. Ileus. 3. Pleural effusion. 4. renal failure. ("P.air" think of the lungs and air in the bowel?)

Besides the mnemonic of A.E.I.O.U, what parameters indicate for urgent hemodialysis?

1. Acidosis. Ph of less than 7.1. 2. Electrolyte abnormalities. symptomatic hyperkalemia such ECG change or ventricular arrhythmias or potassium levels over 6.5. 3. Ingestion Toxic alcohols (methanol, ethylene glycol), salicylate, lithium, sodium valproate, and carbamazepine. 4. Volume overload refractory to diuretics. 5. Symptomatic uremia Encephalopathy, pericarditis, and bleeding.

What are the general features of the 2010 ACLS guidelines for adult tachycardia with pulse greater than 150?

1. Airway, oxygen and cardiac and blood pressure monitor and oximetry. 2. Assess whether patient is unstable. (Cardioversion if yes) 3. Evaluate QRS duration if greater than 0.12 seconds. Purpose of this is to determine if this is a ventricular issue or supraventricular one. (A. If prolonged, consider antiarrhythmic infusion) (B. If normal, give beta-blocker or calcium channel blocker) (In A or B do the following, IV access & 12-lead ECG and consult cardiology and consider adenosine if regular rhythm and monomorphic)

What are the associated causes with papillary necrosis?

1. Analgesic overuse. 2. diabetes. 3. infections. 4. urinary tract obstructions. 5. hemoglobinopathies. 6. cirrhosis. 7. congestive heart failure. 8. shock. 9. hemophilia.

What 3 diseases cause low DLCO and normal spirometry?

1. Anemia. 2. pulmonary embolism. 3. pulmonary HTN.

What medications cause QT prolongation?

1. Antibiotics - macrolides & fluoroquinolones. 2. psych - antipsychotics, TCAs and SSRIs. 3. opioids - methadone & oxycodone. 4. antiemetics - odansetron & granisetron. 5. antiarrhythmics - quinidine, procainamide, flecainide, amiodarone, sotalol.

What medications can lead to serotonin syndrome?

1. Antidepressants. 2. MAOIs. 3. analgesics such as tramadol. 4. antiemetics (odansetron). 5. triptans (sumatriptan). 6. herbal products (St. John's wort).

Causes of aortic regurgitations?

1. Aortic root dilation (Marfan Syndrome, syphilis). 2. Post-inflammatory (rheumatic heart, endocarditis). 3. Congenital bicuspid aortic valve.

What wide complex supraventricular tachycardia are there?

1. Associated bundle branch block. 2. aberrancy

How is Mobitz type I different from II in terms of the following: 1. level of block. 2. QRS complex 3. Exercise or atropine. 4. Vagal maneuvers. 5. Risk of complete heart block.

1. Av node is blocked in I while II is at the bundle of His below the node. 2. Mobitz II can be wide not just narrow like I. 3. Exercise and atropine improves 1 not II 4. Carotid sinus massage worsens I but not II 5. High risk and an indication for pacemaker placement with II not I.

What diagnostic findings support acute tubular necrosis?

1. BUN:Cr ratio less than 20:1. 2. urine osmolality of 300-350. 3. Urine sodium of greater than 20. 4. FENa of >2%. 5. Muddy brown granular casts on urinalysis.

What cause of metabolic alkalosis are associated with high urine chloride, hypovolemia/euvolemia and is saline unresponsive?

1. Bartter syndrome. 2. Gitelman Syndrome.

Initially Alzheimer's presents as memory deficits. What other symptoms progress afterwards?

1. Behavioral (apathy, agitation...). 2. Neuropsychiatric (hallucinations, changes in alertness). 3. non-cognitive neurologic deficits (myoclonus, seizures). 4. urinary incontinence. 5. apraxia (difficulty using a comb...)

Pulmonary-renal syndrome: what systemic vasculitis can cause this?

1. Behcet syndrome. 2. Churg-Strauss syndrome. 3. Cryoglobulinemia. 4. Henoch-Schonlein purpura. 5. Microscopic polyarteritis. 6. Granulomatosis with polyangiitis.

Management for acute chest pain due to cocaine intoxication?

1. Benzodiazepines for blood pressure and anxiety control. 2. Aspirin. 3. Nitroglycerin & calcium channel blockers for pain. 4. immediate cardiac catheterization with repercussion when indicated.

Management of a diagnosed septic abortion?

1. Blood and endometrial cultures. 2. Antibiotics. 3. IV fluids. 4. Broad-spectrum antibiotics. 5. Immediate evacuation of infected intrauterine products of conception (like an draining and abscess).

Causes for esophageal perforation? (name 4)

1. Boerhaave syndrome. 2. instrumentation. 3. esophagitis. 4. esophageal ulcer.

Edema: causes of increased capillary permeability?

1. Burns, trauma & sepsis. 2. allergic reactions. 3. ARDS. 4. Malignant ascites.

What are the 4 main causes of pleural effusions?

1. CHF. 2. Cirrhosis. 3. Nephrotic syndrome. 4. Peritoneal dialysis.

Symptoms of cyanide poisoning include?

1. CNS (HA and AMS). 2. Cardio (arrhythmia). 3. Resp (tachypnea and resp depression). 4. GI (vomiting). 5. Skin (flushing).

Isopropyl alcohol ingestion is associated with what clinical features?

1. CNS depression. 2. disconjugate gaze. 3. absent ciliary reflex

Theophylline toxicity causes what?

1. CNS stimulation (HA, insomnia, seizures). 2. GI (Nausea, vomiting). 3. Cardiac (arrhythmia).

What indications would noninvasive positive-pressure ventilation be recommended?

1. COPD. 2. Cardiogenic pulmonary edema. 3. acute respiratory failure. 4. facilitate early extubation.

What is the management of nephrolithiasis?

1. CT imaging. 2. Narcotics and NSAIDs, preferably NSAIDs as there is no nausea and vomiting. 3. Stone size assessment. If less than 5mm then hydrate the patient with IV fluids. 4. Urology referral - for patients with anuria, urosepsis or acute renal failure (during the incidence of nephrolithiasis).

Antibiotics for patients who have symptoms of meningitis in the setting of neurosurgery/shunt or penetrating trauma to the skull?

1. Ceftazidime or meropenem. 2. vancomycin.

For a patient with severe hypovolemic hypernatremia due to nephrogenic diabetes insipidus. List out the management plan for hydration.

1. IV Normal saline to address hemodynamic instability. 2. Administer half normal saline with dextrose. Establish parameters for infusion rate to limit chance for cerebral edema. Keep rate at 0.5 mEq/dL/hr without exceeding 12 mEq/dL/24hr

Patient presents with chorioamnionitis. What is the management?

1. IV antibioitics - ampicillin, gentamicin and clindamycin. 2. oxytocin to accelerate labor. 3. Cesarean delivery for standard obstetrical reasons. 4. Antipyretics (reduce fever and fetal tachycardia because of mom's cytokines)

What pharmacologic therapies have a role in treating hyperkalemia?

1. IV calcium gluconate. 2. Insulin. 3. sodium bicarbonate. 4. beta-2 agonist.

Management of acute pancreatitis, non-complicated?

1. IV fluids. 2. NPO. 3. Analgesia - meperidine or fentanyl favored over morphine. Notes: 1. NGT can help by decreasing gastrin release. 2. expected 4-7 day recovery.

Management of emphysematous cholecystitis includes?

1. IV fluids. 2. electrolyte replacement. 3. antibiotics (ampicillin-sulbactam, piperacillin-tazobactam, or a combination of an amino glycoside or quinolone with clindamycin or metronidazole)

Patient with infective endocarditis from punching a person. Culture shows penicillin sensitive strep. What do you do for therapy?

1. IV penicillin G. 2. IV ceftriaxone. Note: the wrong answer could be oral penicillin. Only give IV antibiotics for these patients!

Pulmonary-renal syndrome: what renal disorders can cause this?

1. Idiopathic immune complex glomerulonephritis. 2. IgA nephropathy. 3. RPGN with heart failure.

Name some etiologies of constrictive pericarditis?

1. Idiopathic or viral pericarditis. 2. Cardiac surgery or radiation therapy. 3. Tuberculous pericarditis (in endemic areas)

What lung diseases are associated with digital clubbing?

1. Idiopathic pulmonary fibrosis. 2. asbestosis. 3. pulmonary arterio-venous malformations

What are causes of mild glomerulonephritis?

1. IgA nephropathy. 2. Lupus nephritis. 3. thin basement membrane disease.

Echinococcus is an infection of the liver. What patients are diagnosed with this disease?

1. Immigrants from endemic countries. 2. those who live in the southwest exposed to sheep and dogs.

What 5 tests results should you associate with hereditary spherocytosis?

1. Increase MCHC. 2. spherocytes on peripheral smear. 3. negative Coombs test. 4. Increased osmotic fragility on acidified glycerol lysis test. 5. abnormal eosin-5-maleimide binding test.

What 4 physiologic reasons does peripheral edema occur?

1. Increased capillary hydrostatic pressure. 2. decreased capillary oncotic pressure (hypoalbuminemia). 3. increased capillary permeability. 4. lymphatic obstruction and increased interstitial oncotic pressure.

What physiologic effects lead to an increase in the murmur of hypertrophic obstructive cardiomyopathy? Decrease in murmur intensity?

1. Increased murmur with any maneuver that leads to decreased preload (Valsalva, abrupt standing, and nitroglycerin). 2. Decreased murmur with any maneuver that leads to increased after load or increased preload (hand grip, squatting, and passive leg raise).

What are the urease producing bacteria?

1. Klebsiella. 2. Morganella. 3. Pseudomonas. 4. Providencia. 5. Staphylococcus. 6. Ureaplasma.

Causes of anion gap acidosis?

1. Lactic acidosis. 2. Ketoacidosis. 3. Methanol & Formaldehyde ingestion (formic acid) 4. ethylene glycol ingestion (glycol acid and oxalic acid). 5. salicylate poisoning (salicylic, lactic, sulfuric and phosphoric acids) 6. Uremia (ESRD)

What infections are patients with hemochromatosis more susceptible to?

1. Listeria. 2. Vibrio vulnificus. 3. Yersinia enterocolitica.

Patient with a history of hypertension and bipolar disorder has symptomatic hypercalcemia. What medicine would be contraindicated in this setting?

1. Loop diuretics, unless there is CHF exacerbation. 2. Lithium.

What types of cancer have multiple brain metastases?

1. Lung cancer. 2. melanoma.

Metastatic brain cancer. What are the most common cancers (in order with the most common first)?

1. Lung. 2. Breast. 3. Unknown primary. 4. Melanoma. 5. COlon.

Causes of hypercalcemia with low PTH include...

1. Malignancy. 2. Vitamin D toxicity. 3. Granulomatous disease. 4. Drug-induced. 5. Milk-alkali syndrome. 6. Thyrotoxicosis. 7. Vitamin A toxicity. 8. Immobilization. 9. Misc (adrenal insufficiency, acromegaly, and pheochromocytoma).

What is needed for impending or actual respiratory arrest?

1. Mechanical ventilation with intubation & 100% O2. 2. Nebulized SABA & ipratropium. 3. IV corticosteroids. 4. IV magnesium sulfate.

Causes of intravascular hemolysis?

1. Microangiopathic hemolytic anemia. 2. transfusion reactions. 3. infections (clostridial sepsis). 4. paroxysmal nocturnal hemoglobinuria. 5. intravenous Rho(D) immune globulin infusion.

What tests can be done to evaluate proprioception?

1. Moving patient's digits when their eyes are closed. 2. Romberg test.

What viruses have been associated with acute pancreatitis?

1. Mumps. 2. HBV. 3. HIV 4. Coxsackievirus. 5. CMV. 6. HSV.

What 4 clinical features are part of making a probable diagnosis of Creutzfeldt-Jakob disease?

1. Myoclonus. 2. akinetic mutism. 3. cerebellar or visual disturbance. 4. pyramidal or extrapyramidal dysfunction (hypokinesia).

What is the initial treatment choices for patients with an acute viral or idiopathic pericarditis?

1. NSAIDs. 2. Colchicine.

Name some causes of papillary necrosis.

1. NSAIDs. 2. Sickle cell. 3. Alcohol. 4. Immune (transplants). 5. Diabetes. 6. Urinary tract obstruction. Note: Notice 1-5 makes "NSAID"

65 year old woman. 2 months of fatigue and weight gain. History of rheumatoid arthritis, peptic ulcer disease and hypertension. Takes hydrochlorothiazide and acetaminophen. Vitals are normal. Exam shows edema and liver is palpable 5 cm below costal margin. Urinalysis shows 4+ proteinuria. Ultrasound shows bilateral enlargement of kidney. What is the clinical problem? Likely diagnosis? What will a kidney biopsy show?

1. Nephrotic syndrome (proteinuria). 2. Amyloidosis because of the history of rheumatoid arthritis, enlarged kidneys and hepatomegaly - think infiltration and size increase! 3. Biopsy would stain with congo red demonstrating apple-green birefringence under polarized light. Thin fibrils seen under an electron microscope. Note: Likely AA amyloidosis ("abnormal proteins") vs AL (light chain) because of the rheumatoid arthritis association.

What are the side effects of cyclosporine?

1. Nephrotoxicity. 2. Hypertension. 3. Neurotoxicity, like tremors, visual disturbances, seizure, akinetic mutism and headache... 4. Glucose intolerance. 5. Infection. 6. Malignancy. 7. Gingival hypertrophy and hirsutism. 8. GI manifestations.

When is a long-term urinary catheter placement required? (name 2)

1. Neurogenic bladder. 2. urinary retention due to anatomic obstruction.

For uncomplicated cystitis, what drugs are prescribed?

1. Nitrofurantoin. 2. TMP-SMX 3. Foxfomycin 4. fluoroquinolones if the 1st three can't be used.

What are the best indicators of a severe asthma attack?

1. Normal or increased PCO2 values. 2. speech difficulty. 3. diaphoresis. 4. AMS. 5. cyanosis. 6. silent lungs.

When is ampicillin required in the empiric treatment of meningitis?

1. Older than 50. 2. immunocompromised state

What immunologic phenomena are associated with infective endocarditis?

1. Osler's nodes. 2. Roth spots. 3. Glomerulonephritis. 4. arthritis or positive rheumatoid factor.

Name some causes of V/Q mismatch.

1. PE. 2. atelectasis. 3. pleural effusion. 4. pulmonary edema.

Causes of hypercalcemia with low PTH. What laboratories should your order now?

1. PTHrP. 2. 25 (OH) D. 3. SPEP/UPEP. 4. 1,25 (OH) 2D.

For a primary metabolic alkalosis, what is the equation for compensated metabolic alkalosis? How about respiratory compensation?

1. PaCO2 = 0.9 times HCO3 + 16 +/- 2 2. PaCO2 = 1.5 times HCO3 + 8 +/- 2

What are the criteria for initiating long-term oxygen therapy in COPD patients?

1. PaO2 < 55 mmHg or SaO2 <88% (Very similar to goals of keeping someone alive!) 2. Patients with cor pulmonale, evidence of pulmonary hypertension or hematocrit > 55% despite PaO2 or SaO2 levels. 3. If patients become hypoxic during exercise or sleep (nocturnal hypoxia), despite ideal PaO2 or SaO2 levels.

How does abruptio placenta present?

1. Painful vaginal bleeding. 2. uterine tenderness. 3. abnormal uterine contractions. 4. fetal distress.

What are various symptoms that mixed cyroglobulinemia can present with other than asymptomatically.

1. Palpable purpura. 2. Raynaud's phenomenon. 3. MPGN. 4. motor sensory axonopathy. 5. arthralgias.

What 2 ways can empyema occur from?

1. Parapneumonic effusions (no evidence of organisms in effusion, but presence of a nearby infection). 2. Direct contamination via rupture of a lung abscess, bronchopleural fistula, penetrating trauma, thoracotomy ruptured viscus (esophagus) and infection from hepatic or subphrenic abscess.

What is the management algorithm for acetaminophen intoxication?

1. Patient who takes more than 7.5 grams in a single dose within less than 4hours needs activated charcoal. 2. check acetaminophen levels (if patient is a chronically ingests these pills start here in decision making tree). 3. Evidence or liver injury if acetaminophen levels are above 10 micrograms per mL, administer N-acetylcysteine.

ECG changes of hyperkalemia?

1. Peaked T waves with shortened QT interval. 2. PR prolongation. 3. QRS widening. 4. Disappearance of P wave. 5. conduction blocks, ectopy, or sine wave pattern.

What is the most common cause of abnormal hemostasis in a patient with chronic renal failure? How do these patients present? Therapy?

1. Platelet dysfunction. 2. These patient present with normal PT, PTT, and platelet count and prolonged bleeding times. 3. DDAVP.

What are the criteria that would indicate intubating a patient for acute exacerbation for COPD?

1. Poor mental status. 2. Hemodynamic instability. 3. pH < 7.1. 4. If a 2h trial of NPPV fails to relieve COPD exacerbation.

What are the 5 extrahepatic manifestations of HCV?

1. Porphyria cutanea tarda. 2. cyroglobulinemia. 3. lichen planus. 4. leukocytoclastic vasculitis. 5. membranoproliferative glomerulonephritis.

What are causes of moderate-to-severe glomerulonephritis?

1. Postinfectious. 2. Lupus. 3. membranoproliferative GN. 4. rapidly progressive GN. 5. vasculitis (eg, cryoglobulinemia)

Name a few drugs that cause drug-induced esophagitis. (Name 7 on the card, 4 of which you should be able to reason out as direct damage to the GI tract).

1. Potassium Chloride. 2. Aspirin. 3. NSAIDs. 4. Iron. 5. tetracycline. 6. Quinidine. 7. Alendronate.

When should clopidogrel be prescribed?

1. Prescribed for at least 12 months for patients with unstable angina or Non-STEMI. 2. Prescribed for at least one month to a year following a PCI (shown to prevent subacute stent thrombosis). Note: unclear whether clopidogrel should be used with thrombolytic therapy or no reperfusion therapy.

Cancer-realted anorexia or cachexia syndrome is best treated with?

1. Progesterone (preferred due to less side effects) analogs such as megestrol acetate and medroxyprogesterone acetate. 2. corticosteroids. These drugs have increased appetites, weight gain and provide a sense of well-being.

Edema: causes of decreased capillary oncotic pressure (hypoalbuminemia)?

1. Protein loss (nephrotic syndrome & protein-losing enteropathy). 2. Decreased albumin synthesis (cirrhosis & malnutrition).

3 causes which patient presents with renal colic but no stones are identified on conventional imaging?

1. Radiolucent stones (urica acid and xanthine stones. 2. calcium stones less than 1-3 mm. 3. Non-stone ureteral obstruction (blood clot, tumor).

Management of patients with exertional heat stroke?

1. Rapid cooling via ice water immersion or high-flow cool water dousing, ice/wet towel rotation, and evaporative cooling. 2. Fluid resuscitation. 3. electrolyte correction. 4. management of end-organ complications.

What is the presentation of acute angle-closure glaucoma?

1. Rapid onset of severe eye pain. 2. Seeing halos around light. 3. The pupil involved will be dilated and poorly responsive to light as well as injected conjunctiva. 4. Tearing 5. headache, nausea and vomiting (increased ICP)

What cardiac manifestations are associated with hemochromatosis?

1. Restrictive or dilated cardiomyopathy. 2. Conduction abnormalities.

Ultrasound finding for someone with septic abortion?

1. Retained products of conception. 2. increased vascularity. 3. echogenic material in the cavity. 4. thick endometrial stripe.

What are the complications associated with heat stroke?

1. Rhabdomyolysis 2. renal failure 3. ARDS 4. coagulopathic bleeding.

Paracentesis findings for malignancy-related ascites? (2 things)

1. SAAG < 1.1. 2. PMN > 250.

Paracentesis findings due to chronic pancreatitis? (3 things)

1. SAAG <1.1. 2. total protein > 3. 3. amylase of >1000

Patient presents with hyponatremia, low serum osmolarity, euvolemia, and high urine sodium with high urine osmolality. Name a cause.

1. SIADH

What paraneoplastic syndromes are associated with small cell lung cancers.

1. SIADH. 2. Parathyroid hormone-related protein. 3. ACTH. 4. hypercoagulability. 5. Lambert-Eaton myasthenic syndrome. 6. Hypertrophic osteoarthropathy (clubbing). 7. Dermatomyositis & polymyositis.

Management for patient with caustic ingestion.

1. Secure airway, breathing, circulation. 2. decontamination: remove contaminated clothing and visible chemicals; irrigate exposed skin. 3. Chest x-ray if respiratory symptoms or assess for perforation. 4. endoscopy within 24 hours. 5. esophagectomy if needed.

Most common causes of hypopituitarism in the postpartum period?

1. Sheehan's syndrome. 2. Lymphocytic hypophysitis.

ECG findings associated with Wolff-Parkinson-White syndrome?

1. Short Pr interval. 2. Delta wave - slow initial upstroke of QRS complex. 3. ST and T wave abnormalities.

Complications of Prader-Willi?

1. Sleep apnea. 2. type 2 diabetes. 3. gastric distension and rupture. 4. death by choking.

What cancers are associated with Lambert-Eaton syndrome?

1. Small cell lung cancer. 2. Hodgkin Lymphoma.

What forms of pneumonia are associated with brain abscesses?

1. Staph aureus. 2. anaerobes.

What 3 settings will there be an elevated BUN and a mild to absent increase in creatinine?

1. Steroids. 2. Blood in the GIT. 3. Preazotemia.

What and why is sodium bicarbonate used for what 2 drug overdose situations.

1. TCAs - increasing sodium in the blood helps compensate TCA effect on the hearts fast channels. 2. Aspirin - bicarbonate alkalizes the urine to excrete salicylate excretion.

Diabetic neuropathy, what main options are possible for the treatment of paresthesia? What are some atypical medicines used to treat this?

1. TCAs such as amitriptyline and imipramine. 2. NSAIDs. 3. Gabapentin. Atypical. 1. mexiletine. 2. Anticonvulsants - phenytoin, topiramate, carbamazepine. 3. Capsaicin, topical

For uncomplicated acute pyelonephritis what are the drug options?

1. TMP-SMX. 2. Fluoroquinolones. 3. Ceftriaxone (severe)

What are the non-obvious disadvantages of dialysis over renal transplantation.

1. anemia, bone disease and hypertension persist. 2. in diabetics, autonomic neuropathy persists or worsens after dialysis while it stabilizes with transplantation. 3. There is no return of normal endocrine, sexual and reproductive functions and energy levels with dialysis unlike transplantation.

Causes of poor platelet adhesions include what?

1. anti-platelet therapy drugs such as aspirin, clopidogrel and eptifibatide. 2. von Willebrand disease. 3. chronic kidney disease.

Autoimmune hepatitis what labs diagnose this condition?

1. anti-smooth muscle antibodies. 2. ANA, serum.

Causes of acute tubular necrosis?

1. antibiotics. 2. NSAIDs. 3. Diuretics.

Immediate management of esophageal perforation includes what?

1. antibiotics. 2. parenteral nutrition. 3. surgery.

What is morphine do in ST-elevation MI?

1. anxiolytic. 2. preload-reduction.

What are 2 ways a post-ictal state can be associated with respiratory acidosis?

1. apnea or hypopnea during prolonged seizures. 2. aspiration event.

Complications of fibromuscular dysplasia?

1. arterial stenosis. 2. aneurysm. 3. dissection.

Traumatic spinal cord injury, what role does urinary catheterization have and why?

1. assess urinary retention. 2. prevention of acute bladder distension and damage.

List of things amyloidosis can present with clinically.

1. asymptomatic proteinuria or nephrotic syndrome. 2. restrictive cardiomyopathy. 3. hepatomegaly. 4. peripheral neuropathy and or autonomic neuropathy. 5. visible organ enlargement (eg macroglossia). 6. bleeding diathesis. 7. waxy thickening, easy bruising of skin.

Mitral valve prolapse syndrome, what are the symptoms.

1. atypical chest pain. 2. dyspnea. 3. dizziness. 4. anxiety. 5. panic disorder.

What pathophysiologically occurs because of severe hypertension?

1. auto regulation failure. 2. vascular endothelial disruption. 3. fibrinous deposition. 4. narrowed vascular lumen.

Treatment for cold autoimmune hemolytic anemia?

1. avoidance of cold temperatures. 2. rituximab plus or minus fludarabine.

What makes empyema different from complicated or uncomplicated parapneumonic effusions?

1. bacterial colonization. 2. purulent fluid. 3. positive gram stain. 4. usually positive pleural fluid culture.

What respiratory tract infections does azithromycin treat?

1. bacterial sinusitis. 2. streptococcal pharyngitis. 3. community-acquired pneumonia.

What food products is E. coli contamination associated with?

1. beef. 2. produce. 3. milk products.

What 4 things need to be done for treatment of thyroid storm?

1. beta blocker (propranolol). 2. thionamide (PTU). 3. iodine. (block hormone release) 4. glucocorticoids. (decrease T4 to T3 conversion).

Contraindications when managing cocaine intoxication?

1. beta blocker. 2. fibrinolytic not preferred dude to increased risk of intracranial hemorrhage.

Name the valve disorder. 1. Ejection click. 2. Opening snap

1. bicuspid aortic vavle 2. mitral stenosis.

What conditions are associated with coarctation of the aorta?

1. bicuspid valve. 2. ventricular septal defect. 3. Turner syndrome.

Where are alpha receptors found on in the urinary system?

1. bladder neck. 2. base of detrusor. 3. distal ureter. 4. urethra.

Fever, backspin and focal spinal tenderness. What is the workup for this problem?

1. blood cultures, x-ray and ESR/CRP. 2. MRI. 3. CT guided biopsy.

Major ADE of valproic acid?

1. blood dycrasias. 2. liver toxicity.

Indications for treating Paget's disease?

1. bone and joint pain. 2. hypercalcemia of immobilization. 3 neuro-compressive syndromes. 4. high-output cardiac failure. 5. preparation for orthopedic surgery for active disease site. 6. involvement of weight-bearing bones. 7. symptomatic pseudofractures.

What types of cancers metastasize primarily as a solitary brain lesion?

1. breast. 2. colon. 3. renal cell carcinoma.

What can be done to prevent vertical transmission of HCV?

1. breastfeeding. 2. not placing fetal scalp electrodes. 3. lowering the viral load. Note: C-section is not protective.

What cancers are associated with digital clubbing?

1. bronchogenic carcinoma. 2. metastatic cancer. 3. malignant mesothelioma. 4. lymphoma.

What key physical exam findings are not associated with acute thyrotoxic myopathy when a patient presents with distal or proximal muscle weakness?

1. bulbar weakness. 2. respiratory muscle weakness.

In the setting of overdose, what types of medications would cause bradycardia, AV block and hypotension? What additional symptom in combination with those previously would suggest beta blocker toxicity?

1. calcium channel blockers. 2. beta blocker overdose. 3. digoxin. 4. cholinergic agents. Presence of wheezing would suggest beta blocker toxicity.

What drugs can cause hypocalcemia?

1. calcium chelators. 2. bisphosphonates. 3. phenytoin.

What are some key notes to keep in mind about the following infections? 1. Campylobacter. 2. salmonella. 3. shigella.

1. campylobacter - prominent abdominal pian, "pseudoappendicitis", bloody diarrhea. 2. Salmonella - frequent fevers. 3. shigella - fever, bloody diarrhea & abdominal pain.

Drugs which can cause SIADH include what?

1. carbamazepine. 2. cyclophosphamide. 3. SSRIs.

What sequelae are associated with aortic dissections?

1. cardiac tamponade. 2. acute aortic regurgitation. 3. stroke. 4. renal failure.

What conditions can cause pulsus paradoxes?

1. cardiac tamponade. 2. severe asthma. 3. COPD.

Other than migratory arthralgias, facial palsy and conduction abnormalities what other symptoms of early disseminated Lyme disease cause? Treatment for Lyme disease?

1. cardiomyopathy. 2. conjunctivitis. 3. generalized lymphadenopathy. Doxycycline.

What are the goto third generation cephalosporins for empiric treatment of meningitis?

1. ceftriaxone. 2 cefotaxime.

For mild to moderate, complicated cases of acute pyelonephritis what are the drug options?

1. ceftriaxone. 2. cefepime. 3. fluroquinolones (ciprofloxacin and levofloxacin).

Name 3 pathologies that cause steatorrhea via an impaired enteric mucosal surface epithelium. (name 3)

1. celiac's. 2. AIDs enteropathy (mechanism is unclear). 3. Giardiasis.

What endocrine disorders are associated with sarcoid?

1. central diabetes insipidus. 2. hypercalcemia.

Name the species of gas-forming bacteria that cause emphysematous cholecystitis.

1. clostridium. 2. escherichia. 3. staphylococcus. 4. streptococcus. 5. pseudomonas. 6. klebsiella.

What is the reason for the following in hypothermic patients? 1. Elevated lipase. 2. Elevated hematocrit. 3. leukopenia and thrombocytopenia. 4. hyperglycemia.

1. cold-induced pancreatitis. 2. hemoconcentration. 3. splenic sequestration. 4. loss of insulin effect <30C.

Azithromycin what infections does it treat?

1. community acquired pneumonia. 2. sinus infections. 3. streptococcal pharyngitis. 4. Chlamydia.

Doxycycline, what infections does it treat?

1. community acquired pneumonia. 2. zoonotic infections like Lyme disease. 3. Chlamydia. 4. acne.

What respiratory complications are pregnant women at greater risk for?

1. community-acquired pneumonia due to decreased cell-mediated immunity. 2. aspiration pneumonia from increased intra-abdominal pressure and a relaxed lower esophageal sphincter.

What is the workup for paroxysmal nocturnal hemoglobinuria?

1. complete blood count (hypoplastic or aplastic anemia, thrombocytopenia, leukopenia). 2. elevated LDH and low haptoglobin. 3. indirect hyperbilirubinemia. 4. urinalysis (hemoglobinuria). 5. flow cytometry (absence of CD55 and CD59, these being glycosylphosphatidylinositol anchors).

4 causes of low renin and low aldosterone? (name 4)

1. congenital adrenal hyperplasia. 2. Deoxycorticosterone-producing adrenal tumor. 3. Cushing syndrome. 4. Exogenous mineralocorticoids.

What heart diseases are associated with a positive hepatojugular reflex?

1. constrictive pericarditis. 2. right ventricular infarction. 3. restrictive cardiomyopathy Note: test reflex the failure of the right ventricle to accommodate an increase in venous return with abdominal compression.

Treatment for Wilson's Disease?

1. copper chelators such as d-penicillamine or trientine. 2. oral zinc for prevention of copper absorption. 3. liver transplant of fulminant liver failure occurs.

How does excess iron ingestion damaged the body?

1. corrosion. 2. potent vasodilator. 3. toxic to cellular processes. Note: hence shock. and anion-gap metabolic acidosis from poor perfusion and lactic acid accumulation.

Treatment for warm autoimmune hemolytic anemia?

1. corticosteroids. 2. splenectomy for refractory disease.

How are TB and systemic fungal diseases like blastomycosis and histoplasmosis alike?

1. cough. 2. erythema nodosum. 3. granulomas

What viruses can cause myocarditis?

1. coxsackievirus B. 2. parvovirus b19. 3. human herpesvirus 6. 4. adenovirus. 5. enterovirus.

How does glomerulonephritis present?

1. creatinine elevation. 2. proteinuria. 3. nephritic urinary sediment (microscopic hematuria with dysmorphic red blood cells).

What parameters clinically define patients at heightened risk for contrast-induced nephropathy? What can be done to reduce the risk of this nephropathy?

1. creatinine of > 1.5. 2. diabetes. Use non-ionic contrast agent.

Male who was born in a mother taking diethystillbestrol could develop what symptoms?

1. cryptorchidism. 2. microphallus. 3. hypospadias. 4. testicular hypoplasia.

What predisposes patients to portal vein thrombosis?

1. decompensated cirrhosis. 2. hypercoagulable states.

Dietary recommendations for patients with renal calculi?

1. decrease protein and oxalate. 2. decrease sodium intake (more sodium in urine precipitates calcium in the urine.. 3. increase fluid intake. 4. increase calcium intake.

How does nephritic glomerulonephritis lead to edema?

1. decreased GFR. 2. retention of sodium and water by the kidneys.

Symptoms of hypermagnesemia?

1. decreased deep tendon reflexes. 2. loss of deep tendon reflexes. 3. muscle paralysis. 4. apnea. 5. cardiac arrest.

Excessive magnesium sulfate can lead to what side effects?

1. decreased respiratory effort. 2. muscle paralysis. 3. somnolence. 4. visual disturbances. 5. decreased DTR

Why are the elderly frequently hospitalized for dehydration?

1. decreased thirst response to dehydration. 2. impaired renal sodium conservation. 3. impaired renal concentration ability.

What is the ACLS algorithm for ventricular fibrillation and pulseless ventricular tachycardia?

1. defibrillation first. 2. epinephrine administration is repeated every 3 minutes while CPR is ongoing. 3. A repeat attempt at defibrillation is done again followed by the use of an antiarrhythmic such as amiodarone, lidocaine and magnesium.

Histology: Describe what happens with blood-borne antigens that enter the splenic artery?

1. dendritic cells phagocytose the pathogen and present them via MHCII to T helper cells. Helper cells migrate to marginal zone of spleen coming into contact with B-cells to produce antibodies.

What are the risk factors for cervical actinomycosis?

1. dental caries 2. gingivitis. 3. diabetes or immunosuppression. 4. malnutrition. 5. local tissue damage from malignancy or irradiation.

What diseases does griseofulvin treat?

1. dermatophytosis. 2. onychomycosis.

What disease do you give an ACE inhibitor for?

1. diabetes. 2. chronic kidney disease. 3. congestive heart failure.

What patients are at risk of complicated pyelonephritis?

1. diabetes. 2. kidney stones. 3. immunosuppression. 4. anatomic abnormalities of the urinary tract.

Rare drug causes of pancreatitis?

1. didanosine. 2. azathioprine. 3. valproic acid.

Treatment of serotonin syndrome?

1. discontinue SSRIs. 2 . Sedation with benzodiazepines (lorazepam). 3. Cyproheptadine if supportive measures fail.

What are the 3 primary uses of N-acetylcysteine?

1. dissolution of mucus. 2. protection against contrast induced renal failure. 3. therapy for acetaminophen overdose.

What major side effects are associated azathioprine?

1. dose-related diarrhea. 2. leukopenia. 3. hepatotoxicity. Note: these are areas where cells are actively reproducing.

What mental disturbance are associated with barbiturates?

1. drowsiness. 2. incoordination. 3. slurred speech.

What etiologies cause warm agglutinin autoimmune hemolytic anemia?

1. drugs (penicillin). 2. viral infections. 3. autoimmune (SLE). 4. immunodeficiency states. 5. lymphoproliferative (CLL).

What are the 6 general causes of aplastic anemia?

1. drugs. 2. toxic chemicals (benzene, glue...) 3. idiopathic. 4. viral infections. 5. immune disorders. 6. thymoma.

5 weeks since last menstrual period. Positive pregnancy test with no evidence of intrauterine or extrauterine pregnancy (confirmed via ultrasound). What is the differential? Next best step and why.

1. early viable intrauterine pregnancy. 2. ectopic pregnancy. 3. nonviable intrauterine pregnancy (completed abortion). Best step is to do a 48 hour beta HCG test, if it doubles there's a viable pregnancy.

What patient groups are prone to orthostatic hypotension?

1. elderly. 2. hypovolemic. 3. autonomic neuropathy. 4. antihypertensives.

What are the physical exam findings suggestive of constrictive pericarditis?

1. elevated JVP with hepatojugular reflux. 2. Kussmaul's sign (lack of decrease or an increase in JVP on inspiration). 3. pericardial knock. 4. pericardial calcifications on chest radiograph.

What orders need to be put into play with spinal cord compression?

1. emergency MRI. 2. IV glucocorticoids. 3. radiation-oncology and neurosurgery consultations.

Why is endoscopy and barium the wrong answer with an esophageal perforation? (give an answer for each)

1. endoscopy protocol increases pressure in the esophagus, which can worsen pneumomediastinum. 2 barium is known to cause granulomatous reactions in the mediastinum and pleura.

What is Lofgren's syndrome?

1. erythema nodosum. 2. hilar adenopathy. 3. migratory polyarthralgia. 4. fever Associated with sarcoidosis.

What laboratory findings are associated with exogenous androgen use?

1. erythrocytosis with elevated hemoglobin. 2. cholestasis. 3. hepatic failure. 4. dyslipidemia. 5. creatinine elevations.

Ethylene glycol poisoning, pathogenesis?

1. ethylene glycol breaks down into oxalic acid and glycolic acid from alcohol dehydrogenase. 2. glycolic acid damages the renal tubules. 3. oxalic acid binds calcium leading to hypocalcemia and calcium oxalate crystal deposition.

What are the standard indications for a Cesarean delivery?

1. fetal distress. 2. breech presentation. 3. multiple prior Cesarean deliveries Note chorioamnionitis by itself does not indicate the need for C-section.

What are clinical features of a cardiac myxoma?

1. fever and weight loss. 2. Raynaud phenomenon. 3. valvular abnormalities. 4. heart failure due to anatomic obstruction. (dyspnea, orthopnea, cough, pulmonary edema and hemoptysis) 5. Myocardial invasion causing arrhythmias, heart block, or pericardial effusions. 6. Embolization. 7. Lung invasion (mimicking bronchogenic carcinoma).

Symptoms of adult polycystic kidney disease?

1. flank pain. 2. new onset hypertension. 3. abdominal mass. 4. renal insufficiency. 5. hematuria.

What are causes of nephrotic syndrome? (6)

1. focal segmental glomerulosclerosis. 2. minimal change disease. 3. diabetes. 4. primary amyloidosis. 5. IgA nephropathy. 6. membranous nephropathy.

Angelman syndrome define this.

1. frequent smiling and laughter. 2. hand-flapping. 3. ataxia. 4. seizure.

Hepatic complications of Wilson's disease?

1. fulminant hepatitis. 2. chronic hepatitis. 3. portal hypertension. 4. macronodular cirrhosis.

Symptoms of alcoholic cerebellar degeneration?

1. gait dysfunction. 2. truncal ataxia. 3. nystagmus. 4. dysdiadochokinesia (impaired rapid alternating movements) 5. intention tremor or dysmetria (limb-kinetic tremor when attempting to touch a target). 6. pendular reflexes.

Ischemic changes related to acute cholecystitis can lead to what?

1. gangrene. 2. perforation. 3. generalized peritonitis. 4. abscess.

What 2 imaging studies will diagnose esophageal perforation?

1. gastrograffin-contrast esophagram 2. CT esophagogram

What risks of having HCV during pregnancy?

1. gestational diabetes. 2. cholestasis of pregnancy. 3. preterm delivery.

What 3 things must be present to make a DKA diagnosis?

1. glucose over 250. 2. ph less than 7.3 or bicarb of less than 15-20. 3. ketones.

What are signs and symptoms of spinal cord compression?

1. gradually worsening severe local back pain. 2. Pain worse in the recumbent position/at night 3. Early signs: symmetric lower-extremity weakness, hypoactive/absent deep-tendon reflexes. 4. Late signs: Bilateral Babinski reflex, decreased rectal sphincter tone, paraparesis/paraplegia with increased deep-tendon reflexes, sensory loss.

What is the workup for ventilator-associated pneumonia?

1. gram stain, culture endotracheal tub, and start empiric antibiotics (cover for MRSA and pseudomonas). 2. if positive without clinical improvement consider changing antibiotics and or assess for complications such as abscess or empyema.

What are cirrhotic stigmata are due to hyperestrogenism?

1. gynecomastia. 2. testicular atrophy. 3. palmar erythema. 4. decreased body hair. 5. spider angiomas.

Benign causes of pulmonary nodules can include what?

1. hamartoma. 2. infectious granuloma.

Albright hereditary osteodystrophy features?

1. hand and facial features. 2. pseudohypoparathyroidism

Edema: causes of increased capillary hydrostatic pressure include?

1. heart failure (left ventricle & for pulmonale). 2. primary renal sodium (renal disease & drugs). 3. venous obstruction (cirrhosis & venous insufficiency).

What patient groups are more likely to experience fulminant hepatic failure?

1. heavy users of acetaminophen. 2. alcohol. 3. methamphetamines 4. coinfection with HBV and HDV

Patient experiences a crystal-induced kidney injury from a prescribed medicine. What urinalysis findings are expected?

1. hematuria. 2. pyuria. 3. crystals visualized with polarizing light.

How does paroxysmal nocturnal hemoglobinuria present?

1. hemolysis and fatigue. 2. cytopenias due to impaired hematopoiesis. 3. venous thrombosis (intraabdominal, cerebral veins).

What are the strict exclusion criteria to TPA use in an ischemic stroke?

1. hemorrhage or multilobar infarct involving > 1/3 of cerebral hemisphere on CT 2. stroke/head trauma in past 3 months. 3. hx of intracranial hemorrhage, neoplasm or vascular malformation. 4. recent intracranial or spinal surgery. 5. Active bleeding or arterial puncture in past 7 days at noncompressible site. 6. BP > 185/110 7. platelets < 100k or glucose < 50 8. Anticoagulant use with INR of >1.7, PT of > 15 or elevated aPTT

Findings associated with constrictive pericarditis?

1. hepatic congestion with hepatomegaly or possibly cirrhosis (cardiac cirrhosis). 2. pericardial knock (mid-diastolic sound). 3. Elevated JVP with prominent x and y descents and hepatojugular reflux and or Kussmaul's sign.

What are 3 causes of asterixis?

1. hepatic encephalopathy. 2. uremic encephalopathy. 3. CO2 retention.

Phenytoin toxicity, symptoms are?

1. horizontal nystagmus. 2. cerebellar ataxia. 3. confusion.

What major products of combustion in a fire are absorbed in the body?

1. hydrogen cyanide. 2. carbon monoxide.

Treatment for cyanide toxicity?

1. hydroxocolbalamin. 2. sodium thiosulfate. 3. induction of methemoglobinemia with nitrites to increase ferric iron (Fe3+) this is done to provide another substance for cyanide to bind to.

Symptoms of hypocalcemia and hypomagnesemia?

1. hyperactive deep tendon reflexes. 2. muscle cramps. 3. convulsions.

What electrolyte toxicities causes the following: diarrhea, nausea, vomiting, generalized muscle weakness and decreased or absent deep tendon reflexes?

1. hypercalcemia. 2. hypermagnesemia

Classic triad of milk-alkali syndrome?

1. hypercalcemia. 2. renal insufficiency. 3. metabolic alkalosis.

Describe the various aspects of hyper metabolic changes in the first week after a burn. (name 3)

1. hyperglycemia due to insulin resistance. 2. muscle wasting and protein loss. 3. hyperthermia and increased energy expenditure.

Patient presents with hyponatremia, high serum osmolarity. Name 2 causes.

1. hyperglycemia. 2. exogenous solutes (radiocontrast, mannitol).

Patient presents with hyponatremia and normal serum osmolarity

1. hyperproteinemia. 2. hyperlipidemia.

Risk factors for aortic dissection? (3)

1. hypertension. 2. Marfan syndrome. 3. Cocaine use.

What endocrine disorders need to be worked up for new onset atrial fibrillation?

1. hyperthyroidism. 2. diabetes.

Less common causes of acute pancreatitis other than alcohol and gallstones.

1. hypertriglyceridemia. 2 hypercalcemia. 3. recent ERCP. 4. trauma. 5. infection. 6. ADE of medications like (VPA).

What electrolyte disorders cause QT prolongation?

1. hypocalcemia. 2. hypokalemia. 3. hypomagnesemia

Wat laboratory clues are associated with Legionella pneumonia?

1. hyponatremia. 2. hepatic dysfunction. 3. hematuria & proteinuria. 4. Sputum stain showing neutrophils without organisms.

What abnormal laboratory studies are associated with hypothyroidism?

1. hyponatremia. 2. hyperlipidemia. 3. elevated creatinine kinase. 4. elevated aminotransferases.

Define massive pulmonary embolism.

1. hypotension. 2. acute right heart strain.

What physical exam findings are associated with acute aortic dissection?

1. hypotension. 2. increased jugular venous pressure. 3. pulsus paradoxus. (different in blood pressure between arms)

Vital signs of myxedema coma are...

1. hypothermia. 2. bradycardia. 3. hypoventilation. 4. hypotension.

Secondary causes of polycythemia?

1. hypoxemia (lung disease, smoking, OSA, high altitude). 2. congenital (high affinity hemoglobin). 3. hepatic or renal tumors. 4. post-renal transplantation. 5. androgen supplementation (testosterone).

Causes of constrictive pericarditis?

1. idiopathic or viral. 2. cardiac surgery or radiation. 3. Tb

What are the 4 phases HBV can have?

1. immune tolerance. 2. immune clearance. 3. inactive carrier state. 4. HBeAg-negative chronic hepatitis.

Pathophysiology of acute respiratory distress syndrome?

1. impaired gas exchange. 2. decreased lung compliance. 3. pulmonary hypertension.

What are some complications of nephrotic syndrome?

1. increased atherosclerosis due to increase lipid profile. 2. decreased immunity. 3. hypercoagulability.

Pathophysiology of scleroderma renal crisis?

1. increased vascular permeability. 2. activation of coagulation cascade. 3. increased renin secretion.

What features define complicated pyelonephritis?

1. indwelling urinary catheter. 2. urinary obstruction or retention. 3. recent urologic procedure. 4. hospital acquired infection. 5. underlying renal impairment with azotemia. 6. immunosuppression. 7. comorbid diabetes.

Risk factors for splenic abscess include?

1. infection with hematogenous spread. 2. immunosuppression. 3. IV drug use. 4. trauma. 5. hemoglobinopathies.

Etiologies for cold agglutinin autoimmune hemolytic anemia.

1. infections (mycoplasma pneumoniae and mono). 2. Lymphoproliferative diseases

Patient presents from the nursing to the hospital with altered mental status and a recent history of urinary incontinence. What are the main reversible causes of this condition?

1. infections. 2. delirium. 3. medications. 4. restricted mobility. 5. fecal impaction. 6. excess urine output.

What is the pathophysiology for the following findings in cavernous sinus thrombosis? 1. headache. 2. periorbital edema. 3. vomiting.

1. inflammation of the sinus and intracranial hypertension. 2. impaired venous flow from orbital veins. 3. increased ICP.

Secondary causes of amyloidosis?

1. inflammatory arthritis (rheumatoid arthritis). 2. chronic infections (bronchiectasis, tuberculosis, osteomyelitis). 3. Inflammatory bowel disease (Crohn's disease). 4. Malignancy (lymphoma) 5. vasculitis.

What methods of entry does hydrogen cyanide have?

1. inhalation. 2. dermal. 3. intestinal.

Blood glucose below 60 and insulin levels inappropriately elevated. What are the 2 possible diagnoses?

1. insulinoma. 2. surreptitious use of insulin or sulfonylurea.

HBV treatments consist of what medicines?

1. interferon (for younger patients with compensated livers). 2. lamivudine. 3. entacavir. 4. tenofovir (most potent and preferred drug of the 4).

What 2 ways can multiple myeloma cause hypercalcemia?

1. interleukin 6. 2. osteolysis.

What are the extra-renal complications of autosomal dominant polycystic kidney disease?

1. intracranial berry aneurysm. 2. hepatic cysts. 3. valvular heart disease. 4. colonic diverticula. 5. abdominal wall and inguinal hernia.

Causes of extravascular hemolysis?

1. intrinsic RBC enzyme deficiency (G6PD). 2. hemoglobinopathies (sickle cell, thalassemia). 3. membrane defects (hereditary spherocytosis. 4. Hypersplenism, intravenous immunoglobulin infusion. 5. Warm- or cold-agglutinin autoimmune hemolytic anemia. 6. infections (Bartonella, malaria)

Treatment for paroxysmal nocturnal hemoglobinuria?

1. iron & folate. 2. eculizumab.

Name 6 secondary causes of restless leg syndrome.

1. iron deficiency anemia. 2. uremia (ESRD, CKD). 3. Diabetes. 4. MS, Parkinson's 5. Pregnancy. 6. drugs (antidepressants, metoclopramide).

Complications of cold autoimmune hemolytic anemia?

1. ischemia & peripheral gangrene. 2. lymphoproliferative disorder.

How does benign recurrent hematuria present?

1. isolated microscopic hematuria without worsening kidney function. 2. gross hematuria after a URI.

What are some drugs are known to cause drug-induced hepatitis?

1. isoniazid. 2. chlorpromazine. 3. halothane. 4. antiretroviral

Name some long acting nitrates.

1. isosorbide dinitrate. 2. isosorbide mononitrate.

When working up elevated blood pressures. What findings would help rule out malignant hypertension? (name 1 for each of the following, the physical exam, labs, and imaging)

1. lack of eye findings on exam. 2. normal creatinine. 3. normal head CT.

What does chronic tachycardia do to the heart?

1. left ventricular dilation. 2. myocardial dysfunction.

What ECG findings are associated with hypertrophic cardiomyopathy?

1. left ventricular hypertrophy: Tall R wave in aVL & deep s wave in V3. 2. depolarization changes in anterolateral leads (I, aVL, V4, V5 and V6).

When is a flexible bronchoscopy used?

1. life-threatening hemoptysis. 2. Diagnostic tool for hemoptysis if imaging is inconclusive.

What conditions will you find Burr cells (echinocytes)? (name 2)

1. liver disease. 2. ESRD.

Lung cancer metastasis locations?

1. liver. 2. bone. 3. brain. 4. adrenal glands.

What 3 mechanisms does supplemental oxygen cause CO2 retention?

1. loss of compensatory vasoconstriction in areas of ineffective gas exchange (worsening V/Q). 2. Increase oxyhemoglobin reduces uptake of CO2 from tissues (Haldane effect). 3. Decreased respiratory drive and slowing of RR.

Explain how renal failure leads to the expected electrolyte abnormalities and changes in PTH.

1. low GFR and low excretion of phosphate leads to hyperphosphatemia. 2. phosphate bind calcium and interferes with 25 vitamin D from converting into 1,25 vitamin D causing hypocalcemia. low vitamin D prevents additional calcium from gut to be absorbed. 3. hyperphosphatemia stimulates parathyroid to release more PTH hence hyperparathyroidism.

With HELLP syndrome, what 4 variables lead to pulmonary edema?

1. low albumin. 2. increased vascular permeability. 3. decreased renal function. 4. increased pulmonary capillary pressure because generalized arterial vasospasm (systemic HTN) lead to increased after load for the hear to work against.

Patient has vitamin D deficiency what abnormalities is expected with electrolytes and PTH?

1. low calcium. 2. high PTH (response to low calcium). 3. low phosphate (because of PTH). Note - the pathology is vitamin D def which PTH is responding to.

What are the general characteristics of saline-responsive metabolic alkalosis?

1. low urinary chloride excretion and volume contraction. 2. saline infusion corrects it.

What intrathoracic suppurative diseases are associated with clubbing?

1. lung abscess. 2. empyema. 3. bronchiectasis. 4. cystic fibrosis. 5. chronic cavitary infections (fungal or mycobacterial)

What are the 3 most common tumors that cause about 75% of all malignant pleural effusions?

1. lung carcinoma. 2. breast carcinoma. 3. lymphoma.

What are the 3 most common causes of secondary digital clubbing?

1. lung malignancies. 2. cystic fibrosis. 3. right-to-left cardiac shunts.

Systemic involvement due to histoplasmosis can lead to what findings?

1. lymphadenopathy. 2. pancytopenia. 3. hepatosplenomegaly.

Prior to ordering PTH studies, what variables does one need to assess causes of hypocalcemia? (3)

1. magnesium. 2. medication induced. 3. recent blood transfusion (increased citrate, volume).

What diseases are most associated with panic disorder?

1. major depression. 2. agoraphobia. 3. bipolar disorder. 4. substance abuse.

Edema: causes of lymphatic obstruction or increased interstitial oncotic pressure?

1. malignant ascites. 2. hypothyroidism. 3. lymph node dissection.

What 2 diseases are unlikely in a patient who presents with a BP of 230/100 without papilledema?

1. malignant hypertension. 2. intracranial hypertension (Cushing's).

Name 2 causes of pulseless electrical activity?

1. massive PE. 2. Tamponade.

Name some posterior mediastinal tumors.

1. meningocele. 2. enteric cysts. 3. lymphomas. 4. diaphragmatic hernias. 5. esophageal tumors 6. aortic aneurysms. Note: neurogenic tumors are located in the posterior mediastinal compartment (when compared to other mediastinal compartments).

45 year old woman experiencing night sweats, insomnia and irregular menses. Differential?

1. menopause. 2. hyperthyroidism Order FSH and TSH.

What are the central nervous system effects of TCA overdose?

1. mental status changes. 2. seizures. 3. respiratory depression.

Causes of hypocalcemia with elevated PTH include (4).

1. metabolic - Vit D deficiency and Chronic kidney disease. 2. infection - pancreatitis and sepsis. 5. oncology - tumor lysis syndrome. 6. PTH resistance - pseudohypoparathyroidism.

First line antihypertensives in pregnancy?

1. methyldopa. 2. labetalol. 3. hydralazine. 4. calcium channel blockers.

What 2 diseases is associated with microvascular obstruction?

1. microangiopathic hemolytic anemia. 2. hemolytic crisis in sickle cell anemia.

Other than vitamin deficiencies, what other causes of macrocytic anemia are there? How does one exclude B12 and folate deficiency in a macrocytic anemia workup?

1. myelodysplastic syndrome. 2. AML. 3. Drug-induced (hydroxyurea, zidovudine, chemotherapy agents). 4. Liver disease. 5. Alcohol abuse. 6. hypothyroidism. The lack of hypersegmented neutrophils

Signs and symptoms of opioid withdrawal?

1. nausea, vomiting, cramps, diarrhea and hyperactive bowels. 2. dysphoria. 3. restlessness. 4. rhinorrhea, lacrimation. 5. myalgias, arthralgias. 6. mydriasis, piloerection.

Presentation of lupus nephritis?

1. nephritic syndrome. 2. nephrotic syndrome. 3. rapidly progressive glomerulonephritis. 4. pulmonary renal syndrome.

When is cefepime required in the empiric treatment of meningitis?

1. neurosurgery or shunt involved. 2. penetrating trauma to the skull. 3. immunocompromised state.

What 3 things need to be done when cauda equina syndrome presents?

1. neurosurgical eval. 2. MRI. 3. intravenous glucocorticoids.

What are the criteria for ARDS?

1. new or worsening respiratory symptoms during the past week or within 1 week of known clinical insult. 2. Bilateral lung opacities on imaging consistent with pulmonary edema. 3. no signs of heart failure or fluid overload. 4. Objective assessment to exclude hydrostatic pulmonary edema in patients without ARDS risk factors. Severe hypoxemia defined as PaO2/FiO2 ratio < 300 mm Hg and PEEP > 5 cm H2O.

Big key findings indicative of right heart strain?

1. new-onset jugular venous distension. 2. RBBB.

What distinguishes pyomyositis from necrotizing fasciitis

1. no crepitus. 2. disease does not evolve as rapidly.

Solitary pulmonary nodule, what are the high risk variables?

1. nodule size of at 2 cm at least. 2. age over 60. 3. current smoker. 4. smoking cessation of less than 5 years. 5. corona radiate or speculated nodule margins on imaging.

What are some medication causes of hyperkalemia?

1. nonselective beta-adrenergic blockers 2. ACEi, ARBs, and potassium sparing diuretics. 3. digitalis. 4. cyclosporine. 5. heparin. 6. NSAID. 7. Succinylcholine.

What neurohormones are elevated with impaired myocardial contractility?

1. norepinephrine. 2. renin. 3. ADH.

What are symptoms of benzodiazepine intoxication?

1. normal pupil size (generally). 2. mild respiratory depression.

What are the most common causes of acute urinary retention in the elderly?

1. obstruction. 2. neurogenic bladder. 3. detrusor muscle underactivity.

What pulmonary diseases are associated with atrial fibrillation?

1. obstructive sleep apnea. 2. pulmonary embolism. 3. COPD. 4. Acute hypoxia (pneumonia).

Early phases of Wernicke encephalopathy involve what symptoms?

1. ocular disturbances. 2. alteration of consciousness. 3. ataxia.

HCV treatment criteria?

1. older than 18. 2. detectable HCV RNA in serum. 3. Liver biopsy with bridging fibrosis or worse and chronic hepatitis. 4. compensated liver (INR & Bili <1.5, albumin > 3.4, no ascites). 5. Stable laboratory studies (CBC and Cr <1.5).

Contraindications to hepatitis C treatment?

1. ongoing alcohol or drug abuse. 2.major uncontrolled depression.

Patient presents with hyperemesis gravidarum. what workup needs to be done?

1. orthostatic vital signs. 2. CBC and CMP. 3. thyroid function testing. 4. urinalysis.

What factors can contribute to syncope and a patient with hypertrophic obstructive cardiomyopathy?

1. outflow obstruction. 2. arrhythmia. 3. ischemia. 4. ventricular baroreceptor response that inappropriately causes vasodilation.

What are the causes of hyperbilirubinemia characterized mainly by unconjugated bilirubin?

1. overproduction (hemolysis). 2. reduced uptake (drugs, portosystemic shunts). 3. conjugation defects (Gilbert's syndrome).

Risk factors for idiopathic intracranial hypertension?

1. overweight women of childbearing age. 2. tetracyclines 3. vitamin A excess

What things are given during a STEMI?

1. oxgen if less than 90%. 2. Nitrates. 3. Antiplatelet therapy. 4. Beta blockers. 5. PCI. 6. Statin Therapy.

When examining a wound. What findings would suggest a necrotizing surgical site infection?

1. pain, edema and erythema beyond the incision. 2. systemic signs (fever, hypotension and tachycardia). 3. paresthesia or anesthesia at the edges of the wound. 4. purulent, cloudy-gray dischage (dishwater drainage) 5. subcutaneous gas or credits.

For a workup for renal disease when is imaging needed?

1. patient with unresolved pyelonephritis in 2 to 3 days. 2. history of nephrolithiasis 3. unusual urinary findings like gross hematuria, suspicion for urinary tract obstruction.

Treatment for HCV?

1. pegylated interferon. 2. ribavirin.

Name drugs that cause allergic interstitial nephritis.

1. penicillin. 2. cephalosporins. 3. rifampin. 4. trimethoprim. 5. NSAIDs. 6. diuretics. 7. captopril.

Most common antibiotics to cause c. diff colitis include what? (name 4)

1. penicillins. 2. cephalosporins. 3. clindamycin. 4. fluoroquinolones.

Other treatment options for pneumocystis pneumonia?

1. pentamidine. 2. atovaquone. 3. clindamycin-primaquine.

What findings on CT suggest diverticulitis? (4 things)

1. pericolic fat. 2. soft tissue masses (phlegmons). 3. pericolic fluid collections (abscess). 4. diverticula.

Hallmark ECG findings of a ventricular aneurysm?

1. persistent ST-segment elevation after a recent MI 2. deep Q waves.

What are the 2 best indicators of an empyema and the need for a thoracostomy?

1. ph < 7.2. 2. Glucose < 60 mg/dL

When is bicarbonate administered in DKA patients (3).

1. ph less than 7.1. 2. bicarbonate less than 5. 3. severe hyperkalemia.

What drugs can cause peripheral neuropathy?

1. phenytoin. 2. isoniazid. 3. amiodarone. 4. hydralazine.

When a patient presents with DKA what variables present would make you decide on delivering phosphates? What needs to be monitored while this is happening?

1. phosphate less than 1. 2. cardiac dysfunction. 3. respiratory depression. Monitor calcium levels.

Pulmonary-renal syndrome: What connective tissue disorders can cause this?

1. polymyositis or dermatomyositis. 2. progressive systemic sclerosis. 3. rheumatoid arthritis. 4. SLE

Causes of hypomagnesemia?

1. poor nutrition. 2. diuretic use. 3. malabsorption. 4. alcohol abuse.

What non-renal findings are chronic analgesic users likely to experience?

1. premature aging. 2. atherosclerotic vascular disease. 3. urinary tract cancer.

Treatment goals of idiopathic intracranial hypertension?

1. prevention of vision loss. 2. reduction of intracranial pressure. 3 symptomatic relief of headache.

What anaerobic bacteria are associated with sinusitis?

1. prevotella. 2. peptostreptococcus. 3. bacteroides.

What are causes of vanishing bile duct syndrome?

1. primary biliary cirrhosis. 2. failing liver transplantation. 3. Hodgkin's disease. 4. Graft-versus-host disease. 5. sarcoid. 6. CMV infection. 7. HIV. 8. Medication toxicity.

What causes of metabolic alkalosis are associated with high urine chloride, hypervolemia symptoms and are saline unresponsive?

1. primary hyperaldosteronism. 2. Cushing disease. 3. excessive black licorice ingestion.

Calcium phosphate stones present typically with what disease conditions?

1. primary hyperparathyroidism. 2. renal tubular acidosis.

Name five causes of PTH-dependent hypercalcemia

1. primary hyperparathyroidism. 2. tertiary hyperparathyroidism. 3. familial hypercalcemic hypocalciuria. 4. lithium-induced. 5. teriparatide.

What are the complications of nephrotic syndrome?

1. protein malnutrition. 2. iron-resistant microcytic hypochromic anemia. 3. vitamin D deficiency due to increased excretion of cholecalciferol-binding protein. 4. decreased thyroxin levels due to loss of thyroxine-binding globulin. 5. increased susceptibility to infection.

What history findings in a diabetic patient would suggest an additional workup for non diabetic renal disease?

1. proteinuria less than 5 years after disease onset. 2. active urine sediment (red cell, cellular casts). 3. > 30% reduction of GFR in 2-3 months of starting an ACEi or an ARB.

How does nephrotic glomerulonephritis present?

1. proteinuria of greater than 3.5 g/day 2. bland urinary sediment (few cells or casts) 3. possible microscopic hematuria.

Patient presents with hyponatremia, low serum osmolarity, euvolemia, and high urine sodium with low urine osmolality. Name 2 causes.

1. psychogenic polydipsia. 2. beer potomania.

What 2 disease cause increased DLCO and normal spirometry?

1. pulmonary hemorrhage. 2. polycythemia.

Causes of hypoventilation include what?

1. pulmonary/thoracic diseases: COPD, OSA, obesity hypoventilation and scoliosis. 2. Neuromuscular disease: myasthenia gravis, Lambert-Eaton syndrome, Guillain-Barre. 3. Drug-induced hypoventilation: anesthetics, narcotics, sedatives. 4. Primary central nervous system dysfunction: brainstem lesion, infection, stroke.

Babesiosis what is the treatment?

1. quinidine-clindamycin. 2. atovaquone-azithromycin.

What tests ought to be ordered to assess renal transplant dysfunction post-operatively?

1. radioisotope scanning. 2. renal ultrasound. 3. MRI. 4. renal biopsy.

Viral myocarditis. What are the symptoms?

1. recent URI. 2. fatigue. 3. dyspnea. 4. elevated JVP. 5. cardiomegaly on imaging. 6. s3. 7. pulmonary vascular congestion on imaging.

What complications of hypercoagulability occur with nephrotic syndrome?

1. renal vein thrombosis. 2. arterial thrombosis. 3. pulmonary embolism.

What are the 5 mechanisms that cause pulmonary hypertension?

1. respiratory system disorder and hypoxemia. 2. pulmonary venous hypertension due to heart disease or abnormal valves. 3. chronic thromboembolic disease. 4. idiopathic. 5. d/o of pulmonary vasculature (pulmonary capillary hemangiomatosis)

What is malignant hypertension associated with?

1. retinal hemorrhages. 2. retinal exudates. 3. papilledema.

Generally what 4 categories of cardiovascular effects do you need to think about with thyrotoxicosis?

1. rhythm. 2. hemodynamics. 3. heart failure. 4. angina.

What defines a solitary pulmonary nodule?

1. rounded opacity. 2. < 3 cm. 3. completely surrounded by lung parenchyma. 4. no associated lymph node enlargement.

Name some causes of interstitial lung disease?

1. sarcoidosis. 2. amyloidosis. 3. alveolar proteinosis. 4. Infections (fungal, Tb and viral). 5. Silicosis. 6. hypersensitivity pneumonitis. 7. Systemic lupus erythematous. 8. Scleroderma. 9. Idiopathic pulmonary fibrosis. 10. interstitial pneumonia. 11. Cryptogenic organizing pneumonia.

Exertional heat stroke complications?

1. seizures. 2. acute respiratory distress syndrome. (hypoxemia & crackles) 3. disseminated intravascular coagulation. (prolonged coagulation markers) 4. hepatic or renal failure.

What diagnostic tests are specific in diagnosing mixed cryoglobulinemia?

1. serum cryoglobulins. 2. low complement levels. 3. kidney biopsy.

High doses of phencyclidine can lead to what complications?

1. severe hypertension. 2. seizures. 3. life-threatening hyperthermia.

With regards to pancreatic disease, when is exploratory laparotomy appropriate.

1. severe pancreatic necrosis. 2. biliary pancreatitis.

Pathologic causes of sinus bradycardia?

1. sick sinus syndrome. 2. myocardial ischemia or infarction. 3. obstructive sleep apnea. 4. hypothyroidism. 5. increased intracranial pressure. 6. medications.

What are the contraindications for liver transplant?

1. significant cardiopulmonary disease. 2. cancer within the last 5 years. 3. active alcohol or drug abuse.

Silicosis or asbestosis for the following findings? 1. parenchymal nodules. 2. pleural plaques.

1. silicosis. 2. asbestosis.

What are the neuromuscular findings in hepatic encephalopathy?

1. slurred speech. 2. ataxia. 3. bradykinesia. 4. asterixis. 5. hyperactive DTRs. 6. Babinski. 7. Clonus. 8. Nystagmus.

What patients are more prone to developing legionella infection?

1. smokers 2. chronic lung disease. 3. immunosuppressed patients.

What 2 treatment modalities are proven to decrease mortality from COPD?

1. smoking cessation. 2. home oxygen.

What risk factors can precipitate placental abruption?

1. smoking. 2. maternal hypertension. 3. cocaine.

What is the stepwise approach of treatment for ascites?

1. sodium and water restriction. 2. spironolactone. 3. loop diuretics (not more than 1 L/day of diuresis. 4. Frequent abdominal paracentesis (2-4 L/day as long as renal function is okay).

Causes of spinal cord compression include what?

1. spinal injury (MVA). 2. malignancy (lung, breast, prostate cancers and myeloma). 3. infection (epidural abscess).

If a patient has a high pre-test probability for coronary artery disease, what is the best next steps in therapy?

1. start pharmacologic therapy. 2. defer to expert evaluation.

Complications of untreated syphilis in a pregnant mom?

1. stillbirth. 2. neonatal death. 3. mental retardation.

What are the organisms of viridans strep are associated with sinusitis?

1. strep. intermedius. 2. s. mitus. 3. s. oralis. 4. s. mutans.

What symptoms are associated with an aortic dissection and name the correlating structure that was compromised.

1. stroke - carotid artery 2. acute A.R. - aortic valve 3. Horner's syndrome - superior cervical sympathetic ganglion. 4. acute myocardial ischemia and infarction - coronary artery. 5. pericardial effusion and cardiac tamponade - pericardial cavity 6. hemothorax - pleural cavity. 7. lower-extremity weakness or ischemia - spinal or common iliac artery. 8. abdominal pain - mesenteric artery.

Presentation of amniotic fluid embolism?

1. sudden hypoxemic respiratory failure and hypotensive shock during labor or delivery.

HTN during pregnancy puts women at risk for what complications?

1. superimposed preeclampsia. 2. placental abruption. 3. intrauterine growth retardation. 4. preterm birth. 5. cesarean delivery.

For mild/intermittent symptoms of restless leg syndrome what is the treatment?

1. supplement iron when serum ferritin 75 micrograms. 2. use supportive measures (leg message, heating pads, exercise) 3. avoid aggravating factors (sleep deprivation, medications.

What is management strategy for TCA overdose?

1. supplemental oxygen, intubation. 2. IV fluids. 3. activated charcoal for patients within 2 hours of ingestion (unless ileus present). 4. IV sodium bicarbonate for QRS widening or ventricular arrhythmia.

Management of patients with carcinoid syndrome.

1. surgery. 2. octreotide for symptoms.

What states of health lead to activation of neuroendocrine stress and lead to findings similar to SIADH?

1. surgery. 2. trauma. 3. painful stimuli.

Causes of hypocalcemia with normal to depressed PTH levels include (4).

1. surgical - parathyroidectomy, thyroidectomy, radical neck surgery. 2. autoimmune - polyglandular autoimmune syndrome. 3. infiltrative disease - metastatic cancer, wilson disease, hemochromatosis. 4. genetic - PTH gene or calcium sensing receptor gene mutations.

What are the clinical features of Guillain-Barre syndrome?

1. symmetric ascending muscle weakness with absent or depressed deep-tendon reflexes. 2. bulbar symptoms. 3. facial nerve palsy. 4. respiratory compromise 5. autonomic dysfunction. 6. back and extremity pain.

What vascular phenomena are associated with infective endocarditis?

1. systemic arterial emboli (focal neurologic deficits, renal or splenic infarcts). 2. Septic pulmonary infarcts. 3. mycotic aneurysm. 4. conjunctival hemorrhages. 5. Janeway lesions.

Hemodynamic effects of thyrotoxicosis (4)

1. systolic hypertension and increase pulse pressure. 2. increased cardiac contractility and CO. 3. decrease systemic vascular resistance. 4. increase myocardial oxygen demand.

What are the 3 main treatment options for HBV?

1. tenofovir. 2. entecavir. 3. interferon (young patient without liver decompensation). note: never pick lamivudine.

What medications are associated with idiopathic intracranial hypertension?

1. tetracyclines. 2. excess vitamin A (isotretinoin is common).

For patients who had DKA, what variables need to be in place before injecting patients with NPH insulin?

1. they can tolerate oral feedings. 2. recovering from DKA (check anion gap).

Laboratory findings associated with portal hypertension?

1. thrombocytopenia 2. coagulopathy.

Name some middle mediastinal masses.

1. tracheal tumor. 2. pericardial cysts. 3. lymphoma. 4. lymph node enlargement. 5. aortic aneurysms of the arch.

If there is no response to IV atropine in the treatment of severe bradyarrhythmia what do you do next?

1. transcutaneous pacing ~or~ 2. IV dopamine 2-10 microgram/kilogram/min infusion. ~or~ 3. IV epinephrine 2-10 microgram/kilogram/min infusion.

What are causes of central diabetes insipidus?

1. trauma. 2. hemorrhage. 3. infection. 4. tumors.

What is a common side effect of beta-2 agonists?

1. tremor. 2. headache. 3. palpitations. Hypokalemia as potassium is driven into the cells

What neurological findings are associated with lithium toxicity?

1. tremor. 2. hyperreflexia. 3. ataxia. 4. seizures.

What infections does TMP-SMX treat?

1. uncomplicated UTIs. 2. Pneumocystis jirovecii pneumonia. 3. community-acquired MRSA skin infections. 4. nocardiosis.

What conditions would a lumbar puncture be warranted in?

1. unexplained fever. 2. focal neurologic signs (nuchal rigidity) 3. Negative initial investigation for delirium.

Complications with caustic ingestion?

1. upper airway compromise. 2. perforation & mediastinitis. 3. strictures/stenosis (2-3 weeks). 4. ulcers. 5. cancer.

Renal transplant dysfunction in early post-operative period can be explained by what causes?

1. ureteral obstruction. 2. acute rejection. 3. cyclosporine toxicity. 4. vascular obstruction. 5. actue tubular necrosis.

What options are available to treat symptoms of BPH?

1. urinary catheter for cases of hydronephrosis complications. 2. alpha blocker - tamsulosin. 3. 5-alpha reductase inhibitors - finasteride. 4. transurethral resection of prostate - for patients with refractory symptoms.

With the workup of metabolic alkalosis what variables are involved?

1. urine chloride levels. 2. hypervolemia or not. 3. saline response.

Patient with an episode of narrow-complex SVT with minimal symptoms and is hemodynamically stable. What treatment options can you do?

1. vagal maneuvers. 2. IV adenosine. 3. calcium channel blockers.

Complications of warm autoimmune hemolytic anemia?

1. venus thromboembolism. 2. lymphoproliferative disorder.

What would a right-sided lateral medullary infarction cause? What is this syndrome known as?

1. vertigo, falling to side of lesion, difficulty sitting upright without support. Diplopia & nystagmus and ipsilateral limb ataxia. 2. loss of pain and temp sensation on face ipsilateral to lesion and opposite to trunk and limb. 3. dysphagia, aspiration, dysarthria, dysphonia and hoarseness. 4. Autonomic dysfunction (hiccups, automatic respirations, and Horner's) Wallenberg syndrome.

Risk factors for hepatic angiosarcoma?

1. vinyl chloride gas. 2. inorganic arsenic compounds. 3. thorium dioxide.

Most common cause of constrictive pericarditis in the USA? How about developing countries?

1. viral pericarditis. 2. radiation therapy. 3. cardiac surgery Tb

What are some ways hypocalcemia presents in the hospital setting? With some of the causes what notes ought to be taken into consideration about

1. volume expansion. 2. hypoalbuminemia. 3. blood transfusions.

What metabolic alkalosis is associated with low urine chloride and is saline responsive?

1. vomiting. 2. nasogastric aspiration. 3. prior diuretic.

What are the initial means of treating sleep apnea syndrome?

1. weight reduction. 2. avoidance of sedatives and alcohol. 3. avoidance of supine posture during sleep.

Erythropoietin therapy side effects include?

1. worsening of hypertension (possible to cause hypertensive encephalopathy) 2. Headaches. 3. Flu-like syndrome. 4. Red cell aplasia.

What is the range for non-anion gap acidosis

10-14

What is the best treatment regimen for aborting a cluster headache.

100% nasal oxygen.

Presentation of myotonic muscular dystrophy? Mode of inheritance?

12-30 year old with facial weakness, hand grip myotonia an dysphagia. Comorbidities include arrhythmias, cataracts, balding, and testicular atrophy. Autosomal dominant.

Dialysis patient with low-grade fever, acute watery diarrhea, abdominal pain, and guaiac-positive stool in the setting of prolonged omeprazole use. Diagnosis?

C. Diff colitis.

Watery diarrhea last night. Patient was treated 7 days ago for pneumonia. Currently she has not passed gas and abdomen appears distended. Morning labs demonstrate WBC of > 20,000 and Lactate > 2.2. Diagnosis? Likely course of management?

C. diff colitis with concerns of ileus and toxic megacolon. 1. X-ray to confirm and stool antigen. 2. Surgery - subtotal colectomy and diverting loop ileostomy with colonic lavage.

Patient is hospitalized for urosepsis and is treated accordingly for E. coli. History of COPD, gout, and hypertension. Treatment is working. However, on day 6 she becomes confused and has several episodes of watery diarrhea. Low grade fever, tachycardia and wheezing. WBC is 14,000 and creatinine is 1.2 times higher than baseline Diagnosis and management?

C. diff colitis, mild to moderate. Metronidazole is used for C. diff that is mild to moderate. Criteria for mild to moderate: less than <15,000 WBC and and a creatine level < 1.5 times baseline.

Define chorea.

Movement disorder characterized by brief, irregular, unintentional muscle contractions. Movements tend to flow from one to another but are not repetitive or rhythmic.

Chemosis, proptosis and diplopia with nasal discharge in a diabetic. Diagnosis?

Mucormycosis.

Patient with nephrotic syndrome develops renal thrombosis. What is the pathogenesis for this?

Multifactorial reasons for hypercoagulability: 1. increased urinary loss of antithrombin 3. 2. Altered levels of protein C and S. 3. increased platelet aggregation. 4. hyperfibrinogenemia due to increased hepatic synthesis. 5. impaired fibrinolysis.

What differentiates complicated parapneumonic effusion from uncomplicated?

Persistent bacterial invasion into pleural fluid. Ph <7.2,

What are symptoms of bradyarrhythmia that require atropine?

Persistent bradyarrhythmia. 1. hypotension or signs of shock. 2. acute mental status changes. 3. chest discomfort concerning for cardiac ischemia. 4. acute heart failure.

Vertical nystagmus. Diagnosis?

Phencyclidine. It is pathognomonic. Rotary and horizontal nystagmus can occur with this substance.

25 year old complaints of periorbital swelling and dark urine. History of skin infection 3 weeks ago that was treated. Urinalysis shows 8 RBCs per high power field with RBC casts and mild proteinuria. Low serum C3, BUN is 40 and creatinine is 2. Diagnosis and why?

Post-streptococcal glomerulonephritis. Usually seen after a strep throat or skin infection. Findings include what symptoms the patient is experiencing. Lab values correlate to pathogenesis where immune complexes are being used up hence the low C3 levels.

Acute stress disorder symptoms that linger past a month become what diagnosis?

Post-traumatic stress disorder.

S. aureus pneumonia, presentation?

Post-viral URI necrotizing pulmonary bronchopneumonia with nodular infiltrates that can cavitate to cause small abscesses.

Headache, confusion, amnesia, difficulty concentrating or with multitasking, vertigo, mood alteration, sleep disturbance and anxiety after a mild traumatic brain injury (box falling on head)

Postconcussive syndrome

65 year old undergoes CABG. Everything is well until day 2, he developed jaundice. bilirubin is 9, AST and ALT are mildly elevated. ALP is 3x higher than upper normal. Diagnosis? Treatment?

Postoperative cholestasis. Condition is benign, reassure and observe.

Presentation of early dumping syndrome? Treatment?

Postprandial abdominal cramps, weakness, light-headedness and diaphoresis in someone with a recent gastrectomy. Dietary modifications (small frequent meals and avoidance of simple carbohydrates). Octreotide if necessary.

What etiologies cause GBS?

C. jejuni, herpes virus, mycoplasma, and hemophilus influenzae.

25 year old woman with sudden-onset palpitations and generalized weakness. Pressure of 100 over 60 and pulse of 160 and regular. No significant past medical history. Immersion in cold water relieves her symptoms. Why is this the case? What are most cases of paroxysmal supraventricular tachycardia?

Cold water immersion is similar to vagal maneuvers and increases vagal tone and decreases conduction though the AV node. Re-entrant pathways through the AV node. Note: Cold water immersion increases vascular tone

65 year old man with inadequately controlled hypertension. Patient experiences right thigh pain. 20 years of hypertension. Stable angina that required a coronary angioplasty. Heavy smoker. pressures of 180 over 100 bilaterally on the arms. Expected additional physical finding? What is the clinical problem?

Continuous bruit in the right periumbilical area. Resistant hypertension: elevated blood pressure while on multiple medications (at least 3 antihypertensives, one being a diuretic). Note: there's evidence of diffuse atherosclerosis in the setting of resistant hypertension.

15 year old presents with confusion, rapid breathing, and abdominal pain. History of runny nose, dry cough and fever 3 days ago. Patient is dehydrated. BMP shows elevated sodium and potassium. (Glucose is not given in the prompt). Diagnosis?

DKA

What causes renal tubular acidosis in diabetic patients?

Damage to the juxtaglomerular apparatus which leads to hyporeninemic hypoaldosteronism, thus causing type 4 RTA leading to retention of hydrogen and potassium.

Alternate treatment for pneumocystis prophylaxis?

Dapsone.

Intrauterine fetal demise what defines this diagnosis? What steps are in the management of this disease and why?

Death of fetus in utero after 20 weeks gestation prior to onset of labor. 1. coagulation profile out of concern for DIC because a dead fetus causes chronic consumptive coagulopathy due to release of thromboplastin. 2.

Removal of the thick fibrinous peel in a complicated empyema, what is the surgery procedure called that removes this?

Decortication.

What is thought to be the reason why statins cause myopathy?

Decrease coenzyme Q10 synthesis which is involved in muscle cell energy production.

65 year old complains of palpitations and progressive shortness of breath for the past few hours. Choking sensation when he lies down. Hypertension for 20 years and non-compliance. Pressure of 170/100 and pulse of 130 irregularly irregular. Bibasilar crackles and pitting edema on exam. 55% ejection fraction on echo. What is likely responsible for his symptoms?

Diastolic dysfunction. Note: if this was cariogenic shock ejection fraction would be less than 50%.

What HIV medication causes pancreatitis?

Didanosine.

What defines a DASH diet?

Diet high in fruits and vegetables and low in saturated fat and total fat.

When is an adrenal vein sampling done?

Differentiate between adrenal hyperplasia and adenoma by looking for evidence of feedback inhibition.

What pattern of lung involvement is seen on CT with pneumocystis pneumonia?

Diffuse interstitial pattern.

When questioning digoxin toxicity what should be ordered? Why?

Digoxin level, ECG andPT/INR, to rule out life-threatening arrhythmias and coagulopathy.

What anti-hypertensive drug is associated with edema?

Dihydropyridine calcium channel antagonists such as amlodipine.

Recent upper respiratory tract infection followed by a sudden onset of cardiac failure in an otherwise healthy patient. Likely diagnosis?

Dilated cardiomyopathy secondary to acute viral myocarditis.

What kind of cardiomyopathy does alcohol cause?

Dilated cardiomyopathy with left ventricular cavity dilation and impaired left ventricular systolic dysfunction.

How does nitroglycerin alleviate anginal pain?

Dilation of capacitance vessels therefore decreasing preload and heart distension.

How does nitroglycerine work in pain relief for patients with anginal pain?

Dilation of veins (capacitance vessels) which leads to pooling of blood, decreasing ventricular preload. By doing this the heart has less volume to pump and less oxygen requirements.

Patient overdosed on an unknown medication. Presents confused, drowsy and ataxic with blurred vision. Dry mucus membrane and skin, pupils are 8mm, reduced bowel sounds, and 600 ml collected over 1 hour. Diagnosis and why? Treatment?

Diphenhydramine overdose. Sedation from antihistamine effects and sympathetic activity from anti-muscarninic effects. Treat with physostigmine, a cholinesterase inhibitor.

65 year old non-insulin dependent diabetic presents with pneumonia. Patient is on a atorvastatin, metformin, sitagliptin and aspirin. Vital signs 39 C, pressure of 95 over 60, pulse of 115 and respiration of 25. Exam shows dry mucous membranes. X-ray shows infiltrate. Antibiotics are started. Best next step and why?

Discontinue metformin first to lessen the impact of metabolic acidosis. This is withheld until renal function improves.

35 year old woman who presents for a follow up blood pressure check. She has been averaging at 150 over 90. No symptoms. Medications include oral contraceptive for the past 5 years and occasional acetaminophen for tension headaches. No social history risk factors. Normal exam and initial screening labs. Next most appropriate step and why?

Discontinue oral contraceptive and switch to alternate birth control. This patient is taking an OCP, a risk factor that could be addressed with by discontinuation. Since she is asymptomatic it would be wise to assess if the pill is the cause first. If it is not the pill, try lifestyle changes like exercise. If that doesn't work, try a thiazide diuretic.

Normal cardiac index, low systemic vascular resistance and high left ventricular end-diastolic volume is associated with what diseases?

Disease which cause high-output congestive heart failure such as.. 1. severe anemia. 2. thyrotoxicosis. 3. aortic regurgitation.

What pathway is disturbed that causes the cape like distribution associated with syringomyelia?

Disturbance of the crossing spinothalamic tracts in the anterior commissure.

25 year old presents to the ER because of dizziness and syncope. Recently broke up with her boyfriend. 10 pound weigh loss and amenorrhea. Exam shows orthostatic hypotension and dry skin and mucous membranes. Serum sodium and potassium is low. Urine sodium and potassium. Diagnosis and why?

Diuretic abuse - in physiological normal patient her urine sodium and potassium should be low considering her state of dehydration and low plasma levels of sodium and potassium.

Infarction of medial vermis, presentation?

Dizziness, ataxia, weakness and tendency to sway toward side of lesion. Note: there is no oscillopsia nor diminished hearing with this lesion.

If a patient has more than one episode of nephrolithiasis does he need drug therapy instead of changes in diet? What medicines would be appropriate if so?

Do both! Drug therapy includes: thiazide diuretic, urine alkalinization with potassium citrate or bicarbonate salt and allopurinol for hyperuricosuria-related stones.

What happens to homocysteine levels and nitric oxide in those with end stage renal disease and dialysis?

Homocysteine levels are higher due to impaired metabolism and decreased removal. Nitric oxide is inhibited in these patients and cause vasoconstriction and hypertension.

Latina-American female with intolerance to fatty food and RUQ pain. Patient has multiple gallstones and an intra hepatic ill-defined cyst on ultrasound. CT shows a well-defined-cyst with egg shell calcifications. Diagnosis. Treatment? What therapy is contraindicated.

Hydatid cysts, Echinococcus granulosus. Albendazole and surgical treatment. Never aspirate out of concern for anaphylaxis reaction.

What is the acute management of sickle cell anemia?

Hydration, analgesia, and NSAIDs or opioids.

How does primary hyperparathyroidism cause acute renal failure? Is the same true for secondary hyperparathyroidism?

Hypercalcemia leads to dehydration because of polyuria. . With regards to the kidney, secondary hyperparathyroidism is caused by renal failure due to hypocalcemia and hyperphosphatemia.

In what setting would just administering IV NS not be enough to handle a severe hypercalcemia patient.

Hypercalcemia with the following: 1. volume overload. 2. renal insufficiency. Bottom line with calcium, gut answer is to fix with IV NS iff there is no issues with giving an bolus of fluid.

Patient with CKD and prolonged immobility for 3 days, what is this person at greatest risk for?

Hypercalcemia.

Patient presents for the past 3 weeks with nausea and vomiting that have gotten progressively worse resulting in weight loss. Diagnosis? Risk factors of concern?

Hyperemesis gravidum. Concerning risk factor associated with this disease is gestational trophoblastic disease.

60 year old admitted for a COPD exacerbation and respiratory failure. After several hours of invasive ventilation he is intubated. Which of the following is a contraindication to use of succinylcholine for rapid-sequence intubation?

Hyperkalemia because this depolarizing neuromuscular blocker can cause a significant potassium release leading to life-threatening arrhythmias.

Patient is treated for Burkitt lymphoma with chemotherapy at the hospital. 2 days later, new peaked T-waves develop on ECG. Likely diagnosis?

Hyperkalemia due to tumor lysis syndrome.

How are the pathology findings of hypertension different from diabetes in end stage renal disease?

Hypertension - arteriosclertoic lesions of afferent and efferent arterioles and glomerular capillary tufts. Diabetes - increased extracellular matrix, basement membrane thickening, mesangial expansion and fibrosis.

When assessing problems of fatigue and muscle cramps and other nonspecific signs and symptoms. What is a good pivot to pay attention for? If positive, what is the next best step? Then the next if normal?

Hypertension and hypokalemia. Plasma aldosterone to renin concentration. Adrenal suppression test.

What is Cushing reflex and why is it worrisome?

Hypertension, bradycardia and respiratory depression. Suggests brainstem compression.

Hypertensive emergency, define this.

Hypertensive urgency with either of the following conditions. 1. Malignant hypertension - severe hypertension with retinal hemorrhages, exudates or papilledema. 2. Hypertensive encephalopathy - severe hypertension with cerebral edema and non-localizing neurologic symptoms and signs.

50 year old man complains of weakness and chest tightness on and off over the last 24 hours. Family history of myasthenia gravis. ECG shows atrial fibrillation. Work up should include what?

Hyperthyroidism.

20 year old hospitalized after a severe car accident suffering multiple fractures and a blunt abdominal trauma that required laparotomy. During the post-operative period the patient is noted to have hyperactive deep tendon reflexes. Diagnosis and why?

Hypocalcemia can occur during or immediately after surgery in patients who require multiple blood transfusion because of citrate binding in the blood products.

What electrolyte disorder can present as fatigue, anxiety, and depression?

Hypocalcemia.

Pt complaining of difficulty walking. She describes severe weakness and occasional pain in her thighs. ECG shows flat and broad T waves with occasional PVCs. Pt is taking a water pill. Likely diagnosis?

Hypokalemia.

What relevance does magnesium have with calcium? What patient population does one need to remember to watch for this?

Hypomagnesemia causes the body to build a resistance to PTHs effects and its secretion.

What behavioral changes can be seen with HSV encephalitis?

Hypomania. Kluver-Bucy syndrome (hyperphagia and hypersexuality). Amnesia.

Patient presents with symptoms of nausea, vomiting, abdominal pain, back pain, anorexia and increasing confusion. Calcium is 14. Best next step? Diagnosis?

IV normal saline. Consider calcitonin too. Multiple myeloma - back pain and hypercalcemia. Bottom line - symptoms of excess calcium complicated with ongoing cancer process of the back.

Treatment for acute multiple sclerosis?

IV steroids.

Pulsatile headache associated with pulsatile tinnitus or a "whooshing" sound in the ears in a female. What diagnosis must you consider ruling out?

Idiopathic intracranial hypertension.

Any time you need empiric antibiotic coverage and a macrolide like gentamicin is used. What do you need to be cognizant about the effectiveness of the antibiotics being used as empiric therapy?

If a macrolide is used, then there needs to be coverage for anaerobes if it is going to be used as part of an empiric antibiotic combination.

Patient is worked up for hematochezia, NGT has no blood and is positive for bile while colonoscopy is negative. Patient is still bleeding. What do you do next? How about if the patient is no longer bleeding?

If bleeding still: Labeled erythrocyte scintigraphy. If not: capsule endoscopy.

Patient has a CT done for an acute abdominal complaint. Imaging shows a cyst in the kidney. What variables make a simple renal cyst vs a mass?

If it is a simple renal cyst it will be the following: 1. thin, smooth, regular wall. 2. unilocular. 3. no septae. 4. homogenous content. 5. absence of contrast enhancement on CT/MRI. 6. usually asymptomatic. Key points with malignant cystic masses - thick and irregular wall with multiple septae (thick or calcified), heterogenous in content, contrast enhancement, and symptoms of pain, hematuria or hypertension.

What is the last therapeutic option for trigeminal neuralgia?

If medications fail to control pain, options include surgical gangliolysis or suboccipital craniectomy for decompression of the trigeminal nerve.

What is the recommendation for PSA?

If men ask for it and if there is a life expectance of > 10 years.

When should a retrograde endoscopic pancreatography be considered in a patient with acute pancreatitis history?

If the patient has had multiple episodes of pancreatitis with an unidentified cause.

Is it contraindicated to give live vaccines to an HIV patient?

Iff there CD4 count is below 200. Otherwise they can take the following: 1. Varicella. 2. MMR.

Anaphylaxis that occurs within seconds to minutes after a blood transfusion. Angioedema, hypotension and difficulty breathing. Patient is progressing into shock. Diagnosis? Management goals?

IgA deficiency. Circulatory and respiratory support.

Henoch-schonlein purpura. Pathogenesis?

IgA involvement of glomeruli, skin, joints and intestines.

Proteinuria and transient gross hematuria following acute pharyngitis. Diagnosis?

IgA nephropathy

How does one distinguish IgA nephropathy vs Post-streptococcal glomerulonephritis?

IgA nephropathy occurs within 5 days and will have normal complement levels while PSGN happens 10 to 20 days after an infection. Also IgA is more associated with young adults (20-30) while PSGN is more common in 6-10 year olds. Testing can be done for anti-streptolysin O or DNAse B for PSGN.

When diagnosing allergic bronchopulmonary aspergillosis what serum test can be ordered?

IgE

What is ureteral colic?

Ileus due to a vagal reaction associated with ureterolithiasis.

Treatment of patients who are unstable with Torsades? How about stable?

Immediate defibrillation. Intravenous magnesium.

Treatment of exertional heat stroke?

Immediate rapid cooling, preferably with ice-water immersion. Fluid resuscitation. management of metabolic and end-organ complications.

With regards to the patient history, what history would strongly suggest syncope over seizure?

Immediate spontaneous return to baseline neurologic function. Otherwise there is no post-ictal state associated with seizures.

21 year old with acute palpitations and dizziness provoked by fatigue. Vitals show pressure of 65/40 and pulse of 250. He is diaphoretic and cold at his extremities. ECG shows regular, narrow-complex tachycardia. Next best step?

Immediate synchronized DC cardioversion.

What is the mechanism of disease for idiopathic intracranial hypertension? How does this present clinically?

Impaired CSF absorption or excess production. Acute thunderclap headache worsened with lying flat associated with vision changes and nausea.

What is the best indicator of dementia?

Impaired daily functioning.

What does an elevated A-a gradient indicate?

Impaired gas exchange

What was implemented in hospitals to help with communication failures among physicians with handoffs?

Implementing a signout checklist.

How does disseminated histoplasmosis present? Best means of diagnosing this?

In an immunocompromised patient with pancytopenia, hepatosplenomegaly, adenopathy, diffuse reticulonodular or cavitary lesions on x-ray and mucocutaneous lesions. Do an urine antigen and not fungal culture. culture takes weeks.

Generally how does one distinguish non-glomerular hematuria from glomerular hematuria?

In non-glomerular there is, 1. no dysmorphic red blood cells 2. no significant proteinuria. 3. presence of gross hematuria.

How does primary metabolic alkalosis with renal compensation?

In the setting of primary metabolic alkalosis not related to an injury pathology, there is too much bicarb that it overwhelms the kidneys ability to reabsorb it, hence renal losses of bicarb automatically increase.

With regards to loss of sensation due to axonal damage at various levels up and down spinal cord. If someone was to loose pain and temperature sensation on L2 where would the lesion need to be?

In the spinal cord 2 levels above (T12 in this case) and on the contralateral side due to the nature of how the spinothalamic tracts run.

50 year old woman comes to the physician for evaluation of foul-smelling, blood tinged vaginal discharge. Patient has poor appetite and 10 pound weight loss. Exam shows hooded cervix and irregular polypoid mass on anterior vaginal wall. Biopsy shows clear cell adenocarcinoma. What risk factor caused this patient's condition?

In utero exposure to diethylstillbestrol.

For patients with an intermediate pretest probability of coronary artery disease what is the management flow diagram for this patient group?

Is the patient able to exercise? (pharmacologic stress imaging test for those unable). Normal ECG? (follow up with exercise ECG test for normal ECGs). Abnormal ECG? do an exercise imaging test. If any of the following stress test is positive evaluate with coronary angiography.

What is the thought process behind a hepatitis case?

Is there ongoing inflammation? Is there an active hepatitis infection? Are they immune?

What diseases are associated with a thymoma?

Myasthenia and pemphigus.

Fever, cough, abdominal pain, diarrhea, weight loss and splenomegaly and elevated alkaline phosphatase in a HIV patient with CD4 count less than 50. Diagnosis

Mycobacterium avium complex.

Is fibromuscular dysplasia only limited to the renal vasculature? If not list symptoms and arteries involved.

Nope. Cerebrovascular - TIA, stroke, nonspecific symptoms such as headache, pulsatile tinnitus, dizziness. Note: carotid bruit may indicate FMD! (clearly not a specific finding).

IV drug user presents with swelling and redness of his right arm is admitted and treated with IV clindamycin. Swelling improves but he develops nausea, vomiting, abdominal cramps and diarrhea. Patient is restless and asks for pain medications to treat his aching muscles and joints. Vitals are 37 C, pressure of 150/90 and pulse of 100 and respiration of 15. Diagnosis?

Opiate withdrawal.

Somnolence, decreased respiratory rate, miosis, bradycardia and hypotension is associated with what drug overdose? Treatment?

Opioid intoxication. Naloxone.

40 year old homeless man is obtunded and unable to speak. Vitals are 34 C, pressure of 90 over 60, and respirations of 5. Pupils are 5 mm. Several needle marks on exam. Diagnosis and treatment? What is the discrepancy in this clinical picture that could use an explanation?

Opioid intoxication. Naloxone. Patient has normal pupillary size, normal or even enlarged pupils are not uncommon in opioid intoxication as co-ingestants can counteract mitosis.

69 year old patient admitted for hematemesis and has history of an MI. Patient is tachycardic. Endoscopy shows gastric ulcer with clean base. Hemoglobin is 6.9. He is receiving IV fluids. Next best step and why?

Packed RBC transfusion. Patient is experiencing symptoms of anemia (tachycardia) and has a PMH of MI. The cut off for such a patient is a hemoglobin of less than 9. For a stable patient without these risk factors, the hemoglobin cut off for transfusion is at 7.

Patient presents with right upper quadrant pain, fatigue and cola-colored urine. Positive Murphy's sign. From Scotland. Temp of 39 C. 105 pulse. Hepatosplenomegaly and jaundice. MHCH 45%, indirect hyperbilirubinemia, lactate of 1000 and Coombs test negative. Clinical picture? Diagnosis?

Pain related to gall bladder, signs of extravascular hemolysis and elevated MCHC suggest hereditary spherocytosis.

How does analgesic nephropathy present?

Painless and prominent hematuria due to papillary ischemia from analgesic-induced vasoconstriction of medullary blood vessels (vasa recta).

Clinical presentation of uterine atony?

Painless hemorrhage that occurs after hemorrhage.

How does central retinal artery occlusion present? Treatment?

Painless monocular vision loss associated with carotid artery disease, endocarditis, bone fracture, hypercoagulable conditions, vasculitis, atrial myxoma and etc. Ocular massage and high flow oxygen.

What exam findings would suggest syncope over seizure?

Pallor and weak pulses

Nagging epigastric pain that is worse at night, food intolerance, weight loss and enlarged gallbladder with signs of icterus. Diagnosis?

Pancreatic cancer.

60 year old with soft S1 and apical pansystolic murmur radiating to axilla and bibasilar crackles on day 3 after an MI. Diagnosis.

Papillary muscle dysfunction.

Psammoma bodies are features of what thyroid cancer type?

Papillary thyroid cancer

What are medullary thyroid cancer cells classified as?

Parafollicular C cells.

What is an etiology for eosinophilic myositis?

Parasitic infections and inflammatory disorders.

With a geriatric patient admitted to the hospital for decreased oral intake is now presenting with labs consistent of acute kidney injury. She has mild inspiratory crackles in the left lung. She is on multiple medications for chronic disease such as hypertension and osteoarthritis. WBC are elevated. What is the thought process behind her renal failure?

Elderly patients with impaired thirst response are predisposed to preener azotemia due to intravascular volume depletion and poor renal perfusion. Diuretics worsen volume depletion and NSAIDS and ACE inhibitors impair normal physiologic response of renal arterioles, worsening GFR and renal function.

Elevations in which forms of vitamin D in the work up of hypercalcemia with findings of suppressed PTH correlate with what?

Elevated

What is expected on LP of someone with GBS?

Elevated CSF protein and normal WBC count.

When working up a cause of metabolic alkalosis what is the purpose of evaluating urine chloride?

It is efficient and screens out vomiting, nasogastric aspiration and prior diuretic use out of the differential.

A patient with CHF that is on multiple medication for the condition has hyponatremia. What significance does hyponatremia have with regards to this patients outcome and why?

It is indicator the patient has severe CHF because this sign indicates that his body has high levels of renin, norepinephrine and antidiuretic hormone.

Why is TEE better than TTE when evaluating an aortic dissection? Why is TEE done vs CT or MRI in the setting of an acute aortic dissection.

It is more sensitive and can visualize the distal ascending, transverse, and descending aorta. It can be done at the bedside.

For tuberculosis, are there specific findings to this disease on a full bound count?

It is not diagnostic. What is commonly seen is normocytic anemia and lymphopenia.

What does sodium polystyrene sulfonate do?

It is used in the removal of potassium in the gastrointestinal tract.

Why is nitrofurantoin a poor choice in treating pyelonephritis?

It lacks renal tissue penetration.

Liver pathology states presence of regenerative nodules in an alcoholic. What does this mean?

It means this patient has true cirrhosis that is irreversible even with alcoholic cessation.

Why is alkalinizing the urine in rhabdo important? (2 reasons)

It minimizes the breakdown of myoglobin into nephrotoxic metabolites and prevents crystallization of uric acid.

How does eosinophilic pneumonia present?

It presents like asthma but with peripheral eosinophilia. Exam would show diffuse wheezes and fine inspiratory crackles suggestive of bronchial and interstitial involvement.

In the setting of a glomerulonephritis workup, what does low complement mean?

It would suggest either a postinfectious GN, lupus nephritis, MPGN or mixed cryoglobulinemia.

When there is clubbing in a COPD patient what should you think about?

It's not likely COPD or there's another underlying pathology that needs to be diagnosed.

Best treatment for histoplasmosis?

Itraconazole.

"Patchy areas of white matter" on neuroimaging of a HIV patient. Pathogen and diagnosis?

JC virus. Progressive multifocal leukoencephalopathy. Findings are characteristic of demyelination.

Polycythemia vera. Mutation involved? Why is the ESR normal or low in this condition? What is the expected iron level and why?

Jak. The blood sample is too viscous. Low iron because of rapid RBC production.

What is a Laneway lesion? Osler's nodes? Roth spots?

Janeway - macular, erythematous, nontender lesions on the palms and soles. Osler's - painful, violaceous nodules seen on fingertips and toes. Roth - edematous & hemorrhagic lesions of the retina.

What drug used in treating an infectious process, is associated with adrenal insufficiency?

Ketoconazole.

Testicular fibrosis, azoospermia, gynecomastia, decreased intelligence, increased axial skeletal growth, and high FSH and LH?

Klinefelter syndrome (XXY seminiferous tubule dysgenesis)

Hypogonadism, small testes, and decreased virilization.

Klinefelter syndrome.

How can one avoid febrile nonhemolytic transfusion reactions and why?

Leukoreduction, by doing so there will be less cytokines released from these cells to be stored. Techniques to remove these cells include saline washing, freeing and deglycerolizing or buffy coat removal.

Family member brings in an 80 year old man complaining that he's been hallucinating for the past month. Exam shows decreased alertness without much memory concerns. Diagnosis?

Lewy bodies dementia.

What form of dementia causes alterations in alertness, disorganized speech, visual hallucination s and extrapyramidal symptoms?

Lewy bodies dementia.

65 year old presents with fluctuating cognitive impairment and bizarre, visual hallucinations. Diagnosis?

Lewy body dementia.

Visual hallucinations and parkinsonism with fluctuating cognition in a 60 year old. Diagnosis?

Lewy body dementia.

Normal pupil size in the light? In the dark?

Light - 2 to 4 mm. Dark - 4 to 8 mm. For question stems assume the values are for the light.

What is "use dependence"? What types of medicine cause this phenomena? Why is this important?

Enhanced pharmacologic effects of a drug during faster heart rates. Class I (especially class 1C) and class IV (calcium-channel blockers) anti arrhythmic drugs. This happens with class IC because with faster heart rates, drug has less time to dissociate from sodium channels, leading to a higher number of blocked channels, worsening the number of blocked channels hence prolong the QRS complex. This is the mechanism behind treating supra ventricular arrhythmias.

What is the pathogenesis that leads to watery diarrhea and colonic necrosis in C. difficile?

Enterotoxin A causes watery diarrhea. Cytotoxin B causes colonic epithelial cell necrosis and firin deposition.

Lewy bodies are described as what on pathology?

Eosinophilic intracytoplasmic inclusions representing accumulations of alpha-synuclein protein.

Fever, focal spinal tenderness or back pain and neurologic dysfunction in a patient who was recently treated for cellulitis 3 weeks ago.

Epidural abscess.

Of thyroid malignancies, what is the most common cell origin. Of those what is the most common histological type?

Epithelial. Papillary (70%).

High-grade fever and painful swelling of left side face. Exam shows warm, tender, erythematous, raised rash with a well-demarcated border. Diagnosis and likely etiology?

Erysipelas due to Group A streptococcus.

Bacillary angiomatosis. Treatment?

Erythromycin.

30 year old with midline chest pain and diaphoresis. Nausea and vomiting from a party. HIV, medical non-compliance, alcohol abuse and alcoholic hepatitis. Widened mediastinum and left-sided pleural effusion. ECG shows tachycardia. Pleural fluids show yellow exudate with high amylase. Likely diagnosis?

Esophageal perforation.

Circulating immune complexes that deposit in small to medium vessels and maybe associated with low serum complement levels that is most associated with HCV. Diagnosis?

Essential mixed cryoglobulinemia.

Think! how do you think out the pathology findings of alcoholic hepatitis?

EtOH is a poison. Causes hepatocyte swelling and necrosis due to inflammation. The inflammatory response to damage causes neutrophils to come. Mallory bodies are just a histologic finding of damaged hepatocytes demonstrated by their messed up intermediate filaments.

Flank pain, hematuria, oliguria, cranial nerve palsy, and tetany in the setting of overdose. Diagnosis and laboratory findings associated with this?

Ethylene glycol ingestion. 1. High osmolar gap. 2. increased anion gap metabolic acidosis. 3. calcium oxalate crystals in urine.

What psychosis occurs with marijuana? Physical symptoms?

Euphoria and paranoia. Conjunctival injection, impaired reaction time.

What is an important thing to do when assessing a ventricular tachycardia on ECG?

Evaluate whether or not it is supraventricular or not by judging the QRS complex.

What is the main purpose of bronchoalveolar lavage?

Evaluating for suspected malignancy and opportunistic infections (like Pneumocystis pneumonia). Note: this is done after sputum induction.

Any patient with hepatitis C should have what in there management?

Evaluation if they are immune to HAV or HBV and given vaccines if not even if they are pregnant.

Non-exertional heat stroke, treatment?

Evaporative cooling

Post-op day 3 of a laparotomy for adhesions. History of diabetes and hypothyroidism. Fever. Abundant cloudy-gray discharge and dusky, friable subcutaneous tissue. Sensation is decreased at the wound edges. Glucose is 300. Most appropriate next step in management of patient and why?

Exam suggests a necrotizing surgical site infection, parenteral antibiotics and urgent surgical exploration would be the next best step.

How do gastrinomas cause steatorrhea?

Excess acid enter into duodenum and causes lipases to lose their functional capacity of breaking fats down.

What is the presentation of somatic symptom disorder?

Excessive anxiety & preoccupation with 1 or more unexplained symptom. Patients typically will constantly badger you about a symptom excessively for 6 months that is ruled out for pathology.

Define somatic symptom disorder.

Excessive preoccupation with more than 1 medically unexplained somatic symptom for more than 6 months, causing social and occupational impairment.

Sickle cell patient presents with stroke symptoms. Treatment?

Exchange transfusion.

Patient presents with confusion and a body temperature of greater than 40 C (104F) and with central nervous system dysfunction (encephalopathy) after a long day of intense exercise out in the Arizona desert during the summer. Diagnosis?

Exertional heat stroke

Acute confusion, hyperthermia, tachycardia and persistent tachycardia after exertion under direct sunlight.

Exertional heat stroke.

Meningitis symptoms in a pregnant woman or immunocompromised person what unique etiology targets these groups. What symptoms are experienced from this bug?

Listeria. Diarrhea, nausea, and vomiting.

What dermatologic features are associated with cholesterol crystal emboli?

Livedo reticularis. Ulcers. Gangrene. Blue toe syndrome.

What conditions will you see spur cells (acanthocytes)

Liver disease.

What are anti-hypertensive medications that are absolutely contraindicated in pregnancy?

Furosemide and any blocker of the RAAS system - ACEi, ARB, aldosterone blocker or direct renin inhibitors.

Furosemide vs Thiazides?

Furosemide excrete calciums while thiazide saves it.

65 year old with substernal chest pain, severe shortness of breath and diaphoresis within the past 30 minutes. Pain radiates to left arm. ECG shows STEMI in anterior leads. Vitals are 98.5 F, pressure of 110 over 70, pulse of 60 and respirations of 30. Oxygen saturation is 90%. S3 auscultated and bibasilar crackles halfway up lungs on exam. Receives aspiring, clopidogrel and atorvastatin. Next best step and why? Complication risks for the appropriate treatment?

Furosemide. Patient is experiencing acute anterior wall myocardial infarction causing acute pulmonary edema (flash pulmonary edema). Acute kidney injury from hypotension if patient is normal or hypotensive.

Bite cells and Heinz bodies presenting together. Diagnosis?

G6PD deficiency.

60 year old man presents with several weeks of vague abdominal pain and increased fatiguability with findings of microcytic anemia, positive fecal occult blood and hepatomegaly with a hard edge on liver palpation. Mildly elevated AST and ALT. CXR show small left-sided pleural effusion. Diagnosis? Next best step?

GI malignancy likely colon cancer metastatic to the liver. CT scan.

Proton pump inhibitor side effects?

GI side effects (overall). 1. nausea. 2. diarrhea or constipation. 3. abdominal pain. 4. flatulence. Others. 1. rash. 2. headache.

60 year old with right upper quadrant pain, mildly elevated liver enzymes and firm hepatomegaly.

Liver metastasis (likely colon cancer).

What is the disease distribution associated with amyloid angiopathy?

Lobar involvement.

What is pretibial myxedema?

Localized nonpaying thickening and induction of the skin over the lower legs, pretrial area or dorm of the hands in Grave's disease.

Patient with a stroke and evidence of atrial fibrillation. Best therapy to initiate?

Long-term anticoagulation (warfarin, dabigatran, and or rivaroxaban)

Patient with chronic kidney disease presents with hyperkalemia with no exam findings or ECG changes typical for it. Treatment?

Loop Diuretic.

Patient with severe COPD history presents with increasing shortness of breath. Exam shows elevated JVP, diffuse wheezing, muffled heart sounds, mild hepatomegaly, and pitting edema halfway to the knees. X-ray reveals hyper inflated lungs and flattened diaphragm. He is started on steroids, bronchodilators and furosemide. Chemistry panel from admission to 5 days later shows doubling of urea nitrogen and creatinine and lower potassium and bicarbonate levels. Diagnosis and why?

Loop diuretic induced acute kidney injury. This occurs because decreased circulating intravascular volume reduces cardiac output which leads to prerenal azotemia. In the setting of a patient with pulmonary hypertension and stigmata for for pulmonale this is dangerous. Low potassium is expected from the diuretic.

What is the relationship between magnesium and hypokalemia?

Magnesium is a cofactor for potassium uptake and maintenance of intracellular potassium levels Hypomagnesemia may need to be corrected to address hypokalemia even though IV KCl is being administered.

Treatment for Torsades de Pointes?

Magnesium sulfate.

What would happen if a hepatic adenoma ruptured?

Major intra-abdominal bleeding with peritonitis and hypotension requiring surgical intervention.

What complication is not always found in malignant hypertension and thus is not required for this definition?

Malignant nephrosclerosis. 1. acute renal failure. 2. hematuria. 3. proteinuria.

What descriptors describe alcoholic hepatitis?

Mallory bodies, infiltration by neutrophils, liver cell necrosis, and perivenular distribution of inflammation.

Patient presents with supra ventricular tachycardia with aberrancy and is stable. What do you do?

Maneuvers to determine rhythm such as carotid massaging rate control (gold is to diagnose and treat respectively).

75 year old operated for strangulated inguinal hernia. No past medical history. Third postoperative day falls while getting out of bed. Patient is confused and has slurred speech. Pressure of 90/50 and pulse of 125 and respiration of 25. Distended neck veins and decreased bi-basilar lung sounds. ECG shows new-onset right bundle branch block and nonspecific ST- and T-wave changes. Patient dies despite resuscitation efforts. Diagnosis?

Massive PE. Note: only Massive PE not regular PE causes syncope.

a young boy presents top the pediatrician with pain and limited motion of the right knee. he had a similar episode a few months earlier. Has a history of easy bruising. Diagnosis? What is the pathogenesis of his current complaint.

Hemophilia. Iron/hemosiderin deposition which leads to synovitis and fibrosis of the joint. He's a boy and has bruising issues. Current issue is hemarthrosis.

20 year old male bumps into a table which causes swelling and pain in his thigh. Exam shows marked ecchymosis and swelling. Family history of bleeding problems in a maternal uncle. What areas of bleeding are typical?

Hemophilia. 1. Hemarthrosis. 2. Intramuscular hematomas. 3. Gastrointestinal tract bleeding. 4. Genitourinary tract bleeding.

60 year old at the ED after losing consciousness at work. Difficulty walking for the past few days due to wound infection of foot. Diabetes and hyperlipidemia. Pressure of 80/40 and pulse of 120 and regular. Cold and clammy skin. Right catheter shows right atrial pressure of 18 (high), pulmonary artery pressure of 40/20 (high), and normal pulmonary capillary wedge pressure (6-12). Diagnosis?

Massive pulmonary embolism.

How is abrupt placenta different from chorioamnionitis?

Maternal fever and leukocytosis is not associated with placental abruption.

Diagnostic criteria for chorioamnionitis?

Maternal fever with at least one of the following: 1. uterine tenderness. 2. maternal or fetal tachycardia. 3. malodorous amniotic fluid. 4. purulent vaginal discharge.

What functions are compromised with lateral mid-pontine lesions?

Motor & sensory nuclei of the ipsilateral trigeminal nerve leading to the following: 1. weakness of mastication. 2. diminished jaw jerk reflex, 3. impaired tactile and position sensation of the face.

Strokes of the internal capsule cause what?

Motor deficits.

How does carcinoid syndrome lead to niacin deficiency?

Serotonin is made from tryptophan, something that is needed for niacin production.

Side effect of MAO-B inhibitors. What form of dementia is this used in?

Serotonin syndrome Parkinson's disease.

How is subarachnoid hemorrhage different from intracerebral?

Subarachnoid presents with sudden onset of severe headache, loss of consciousness, n/v and meningismus.

New onset headache, with episode of vomiting in a 40 year old. CT shows hyperdense (bright) signals in the frontal inter hemispheric cisterns, sylvan cistern and peri-pontine cisterns.

Subdural arachnoid hemorrhage (ruptured saccular or berry aneurysm).

55 year old admitted for a COPD exacerbation and respiratory failure after being rescued from a burning building. Patient is intubated and within minutes he develops ventricular tachycardia. Diagnosis?

Succinylcholine administration in setting of hyperkalemia likely related to motor vehicle accident.

Ruptured ovarian cyst presentation?

Sudden onset of lower abdominal pain, usually at mid-cycle following strenuous physical activity or sexual intercourse.

Define conus medullaris syndrome.

Sudden-onset of severe back pain, perianal anesthesia, symmetric motor weakness, hyperreflexia and early-onset of bowel and bladder dysfunction.

25 year old with an attempt at suicide via medication overdose suffers 2 seizures. Exam shows 102 F, pupils are dilated but reactive to light and accommodation, skin is flushed and dry, and bowel sounds are reduced. EKG shows QRS complex of 190 milliseconds. Diagnosis? Best prognostic factor?

TCA overdose. QRS duration.

Hyperthermia, seizure, mydriasis and intestinal ileus in a patient with a history of depression. Diagnosis? Treatment?

TCA overdose. Sodium bicarbonate.

Patient presents with paresthesia of new onset due to his diabetes and has a history of BPH, syncope and arrhythmias. What drug should not be prescribed for this patient

TCAs are contraindicated

How does TCA affect the heart in an overdose situation?

TCAs inhibit fast sodium channels in the His-Purkinje system and myocardium, causing the following: 1. Hypotensions. 2. QRS prolongation. 3. Ventricular arrhythmias (v. tach, v. fib). This is why sodium administration mitigates the effects of TCAs effect on the heart.

Other than TMP-SMX what other drug could be used for the treatment of pneumocystis pneumoniae? What adverse drug effect is associated with this?

TMP & Dapsone. Hemolytic anemia in patients with G6PD deficiency.

Treatment for nocardia?

TMP-SMX

Treatment of Nocardia skin infection?

TMP-SMX

What antibiotic prophylaxis should be given to organ transplant patients and why? How about vaccinations?

TMP-SMX out of concern for pneumocystis pneumonia. Hepatitis, meningococcus, and influenza.

HIV patient, dyspnea, fever, and dry cough with CD4 count of 150. Blood gas shows PaO2 of 50 and low CO2. What is the treatment plan?

TMP-SMX with prednisone for pneumocystis pneumonia. Prednisone helps minimize the inflammatory response so respiratory function doesn't tank in this patient. (less than PaO2 of 70 is indicated)

If a patient has a penicillin allergy and needs empiric treatment for meningitis what could be given instead?

TMP-SMX.

HIV patient recently is treated for pneumocystis jirovecii. After admission labs show hyperkalemia and elevated creatinine. Diagnosis and how?

TMP-SMX. This drug blocks epithelial sodium channels in the collecting tubule similar to potassium-sparing diuretic amiloride. There is also an increase in creatinine due to the drugs competition with this metabolite.

Elevated BUN and creatinine. Altered mentation. Indirect hyperbilirubinemia, elevated reticulocytes and normocytic anemia. Thrombocytopenia. Diagnosis? Best initial diagnostic test?

Thrombotic thrombocytopenic purpura. Blood smear (schistocytes)

If medical therapy is not working with myasthenia gravis. What other options exist?

Thymectomy and plasmapheresis.

Patient is suspected of having a myasthenia gravis. What imaging findings are associated with this disease?

Thymoma

What is methylene blue used for?

Treatment of methemoglobinemia seen in the setting of drug ingestion like dapsone or anesthetic agents that is causing cyanosis and respiratory depression.

Theophylline, what is it used for? What adverse drug effects are associated with this?

Treats respiratory symptoms. 1. seizures. 2. hyperthermia. 3. cardiac arrhythmias. 4. hypotension.

What infection causes a patient to experience gastrointestinal symptoms during the first week of infection such as pain, nausea, vomiting and diarrhea, then causes patient to experience muscle pains 3 weeks after the intestinal symptoms. Other symptoms during the muscle aches include fever, sublingual splinter hemorrhages, and periorbital edema. Diagnosis? Where did the infection come from?

Trichinellosis. During a vacation from Mexico, Argentina, China and Thailand.

nursing home resident with altered mental status and decreased oral intake. Fever, 100 heart rate and cloudy Foley bag. Leukocytosis and creatinine of 1.5. Urinalysis shows pH of 8.5, leukocyte esterase positive and bacteria. Diagnosis? What is a possible complication of this pathogen?

Urinary tract infection due to Proteus mirabilis. Struvite stones.

Enterococcus faecalis is cultured from blood in the setting of infective endocarditis. What is the likely source of infection?

Urinary tract infection.

How do patients with chronic hypercapnia respiratory failure due to COPD maintain an otherwise normal pH value?

Renal tubular compensation.

With the evaluation of urinary tract anatomy for children what is the preferred imaging modality and why?

Renal ultrasound due to lack of radiation.

Pathogenesis of contrast-induced nephropathy?

Renal vasoconstriction and tubular injury.

Sudden abdominal pain, fever and hematuria in a patient with a nephrotic syndrome. Diagnosis?

Renal vein thrombosis.

What is the most common cause of resistant hypertension?

Renovascular hypertension (renal artery stenosis).

What finding on a physical exam would quickly rule out cauda equina syndrome in the workup of back pain and bilateral loss of leg movement?

Sensation at the umbilicus.

Oscillopsia. Define this. What physical exam maneuver can elicit this? what antibiotic can cause this phenomena?

Sensation of objects moving around in the visual field when looking in any direction. Abnormal head thrust test. Aminoglycosides.

Fever, lower abdominal pain, uterine/cervical motion tenderness, and foul-smelling discharge in the setting of a recent elective abortion. Next best step?

Septic abortion. Ultrasound - findings to suggest would be an irregular and echogenic thick endometrial stripe with increased vascularity, and no adnexal mass.

What HIV medicine causes hypersensitivity syndrome?

abacavir.

Symptoms of amebiasis?

abdominal pain and bloody diarrhea.

Autoimmune hepatitis. ALT & AST values?

above 1,000

What is the liver pathology findings of acetaminophen toxicity?

centrilobular or diffuse necrosis.

Why is there a precaution with bicarbonate use in kids?

cerebral edema.

How does neurosyphilis appear?

decreased concentration. memory loss. dysarthria. tremors of fingers and lips. Psychotic. Confused Sloven.

What is the presentation of hemorrhage to the pons?

deep coma and total paralysis within minutes with exam findings of pinpoint reactive pupils.

What are the indications of plasmapheresis for myasthenia gravis?

When all medical therapy fails. Myasthenia crisis. Stabilizing a pt prior to thymectomy.

What defines complicated pyelonephritis?

When an initial clinical problem of pyelonephritis becomes a management in the following complications. 1. Renal corticomedullary abscess. 2. perinephric abscess. 3. emphysematous pyelonephritis. 4. papillary necrosis.

What is a positive hepatojugular reflex?

When applying firm and sustained pressure for 10-15 seconds over the abdomen, a sustained elevation of JVP greater than 3 cm is continued with abdominal compression.

Window period regarding HBV?

When both the antigen and antibody are not present in a patient's hepatitis panel, otherwise the lag time after an infection is over and the time prior to clinical detection of immunity from observable antibodies.

For patients in a non-ketotic hyperglycemic state, when do we administer glucose? And why?

When glucose is less than 250. Glucose helps prevent the development of cerebral edema. *Clinical significant cerebral edema occurs in 1% of pts presenting with diabetic comas due to insulin and hydration on the body's osmolality and serum glucose.

What defines pulsus paradoxus?

When inspiring air there is a 10 mm Hg drop in blood pressure.

Define selection bias.

When the treatment regimen selected for a patient depends on the severity of the patient's condition. This is known as confounding by indication.

When is a suprapubic catheter placed?

When transurethral catheterization is not possible due to urethral strictures or a man with a severely enlarged prostate.

When is urine cytology done?

When urinary tract cancer is of concern.

With regards to healing a wound, what should not be done?

Whenever a frost bite occurs, no debridement of the wound should happen because it takes weeks before an accurate assessment between viable and dead tissue can be made.

How does acute peri-infarct pericarditis present? Treatment?

Within 1-3 days after an MI the findings of pericardial friction rub with or without chest pain. Supportive.

Patients in need of an antipsychotic generally are prescribed what?

Without any consideration to certain circumstances, usually 2nd generation antipsychotics are preferred.

30 year old woman with months of left-sided chest pain that lasts for several hours. Everything else on the exam is unremarkable. ECG done and shows normal sinus rhythm. Next best step and why?

Women is less than 50 years old and has atypical chest pain in an otherwise normal workup. Pretest probability is less than 10%. Observe.

3 year old boy presents with torticollis. What do you do now and why?

X-ray of the neck because acquired torticollis can be due to URIs, minor trauma, cervical lymphadenitis, and retropharyngeal abscesses, so ordering this test is to evaluate if for the presence or absence of a cervical spine fracture or dislocation. Bottom line: neck pathology = do an x-ray first.

What findings are seen in patients with constrictive pericarditis on chest x-ray? How about echocardiogram?

X-ray: pericardial calcifications. Echo: increased pericardial thickness, abnormal septa motion and bi-atrial enlargement.

Gum hypertrophy can be due to tacrolimus or cyclosporine.

Yes for cyclosporine and no for tacrolimus.

Are the toxicity profiles of tacrolimus and cyclosporine similar? What side effects would suggest cyclosporine toxicity?

Yes they are very similar with respect to nephrotoxicity, HTN, tremor. Cyclosporine causes hirsutism and gum hypertrophy

Absent achilles tendon in a 70 year old. Normal?

Yes this can be a normal finding.

Can patients with rapidly progressing liver failure have decreasing transaminases with worsening PT/INR and bilirubin? What does these values mean?

Yes! Decrease in functional liver tissue. (decreasing transaminases) And worsening synthetic function (PT/INR and bilirubin>

Is it possible for a pneumonia to present with blunting of the costophrenic angles?

Yes, if there is an effusion involved.

Can deep-brain stimulation be done to help with patients who have OCD?

Yes, iff it is necessary for severe cases of OCD not responsive to therapy, high-dose SSRIs and antipsychotics or clomipramine. Electrodes are placed at the nucleus accumbens.

Seizures and severe hyponatremia, medical emergency?

Yes, immediate 3% saline infusion no faster than 0.5 mEq/L/hr.

Is alcohol associated with hypogonadism. If so how?

Yes, it suppresses testosterone production.

Patient has had a Tdap during childhood. Do you need to give Tdap again during adulthood?

Yes, start over with Tdap and immunize with Td booster each 10 years.

Describe the presentation of bacillary angiomatosis? How does one diagnose this?

a disease associated with immunosuppressed patients. It causes fever, weight loss, malaise and abdominal pain in addition to cutaneous and visceral angioma-like blood vessel growths (hence the part of the name angiomatosis). Tissue biopsy and microscopic identification and characteristic angiomatous histology.

What anti fungal is useful for both candida and cryptococcus?

Flucytosine.

Are Pseudomonas and Klebsiella common causes of pneumonia?

Nope.

What ought to be done to the renal system to minimize damage from rhabdomyolysis?

Fluids Alkalinize the urine

Sinusitis is often due to what kind of flora?

Viridans streptococci and anaerobic bacteria.

What causes the peripheral neuropathy in B12 deficiency?

Defective myelin synthesis because homocysteine cannot be converted into methionine.

Flumazenil vs fomepizole?

Flumazenil treats benzodiazepine intoxication. Fomepizole treats methanol or ethylene glycol poisoning.

When is hematoma evacuation treatment of choice (in regards to hematomas in the skull)?

Large or symptomatic epidural or subdural hematomas.

Electrical alternates can be seen with what kind of effusions?

Large pericardial effusions (best morbidity indicator?

Tumor lysis syndrome, pathogenesis?

Large scale release of uric acid which precipitates renal failure.

Crescent formation on light microscopy of a kidney indicates what medical condition.

Rapidly progressive glomerulonephritis.

Is pyuria associated with malignancy.

Rarely ever!

IV fluids first or catheter first with a patient with acute kidney injury due to obstruction?

Catheter first, or else you will worsen the urinary retention.

Patient drank an unknown substance. Tongue is white, heavy drooling of saliva and unable to swallow. Vitals are normal. Diagnosis?

Caustic ingestion.

White tongue, heavy salivation and dysphagia. Diagnosis?

Caustic ingestion.

What is the the classic "3" sign on x-ray? How does this happen?

Coarctation of the aorta. Indentation of aorta at site of coarctation with pre- and post-stenotic dilation.

What is the diagnosis? a circadian rhythm disorder characterized by inability to fall asleep at normal bedtimes (10pm and midnight)

Delayed sleep phase syndrome.

For complicated cystitis, what drug regimen is given?

Fluoroquinolones and an extended-spectrum antibiotic such as ampicillin/gentamicin for severe cases.

What is the only oral agent approved for empiric treatment of pyelonephritis?

Fluoroquinolones such as ciprofloxacin.

Best class of antibiotics used for treating pyelonephritis?

Fluoroquinolones.

What medicines cause Achilles tendinopathy?

Fluoroquinolones.

Do you give TMP-SMX alone for those with PCP in an HIV patient?

Depends. Steroids as they have been noted to reduce mortality. Indications include a PaO2 < 70 or A-a gradient is > 35 mm Hg on room air.

What immediate acting anticoagulant can be used in the setting of reduced renal clearance?

Unfractionated heparin. (do not use LMW heparin, rivaroxaban or fondaparinux)

Young patient with tinnitus and decrease hearing on one side. Physical exam shows numerous cafe-au-lait spots. Diagnosis and best next step?

Neurofibromatosis type II. MRI with gadolinium.

Pellagra. Put an image in your mind as to what looks like this. What is the mechanism for this disease?

Niacin deficiency

Are long acting beta 2 agonists the answer in acute exacerbations of chronic obstructive pulmonary disease?

No

Does fibromuscular dysplasia cause significant proteinuria?

No

Does multiple myeloma and monoclonal gammopathy of undetermine significance to complement levels?

No

Is anasarca and abnormal urinalysis found in hyperthyroidism?

No

Does ceftriaxone adequately treat streptococcal pneumoniae?

No because of increasing resistance.

What is the goal of management with oxygen therapy?

SaO2 of 90-93% or PaO2 of 60-70 mm Hg. With parameters of significant acidosis or severely reduced LOC requiring mechanical ventilation.

3 most common causes of acute bloody diarrhea? Of the 3 which does not necessarily cause a fever.

1. E coli 2. Shigella. 3. Campylobacter E. coli.

Diagnostic findings of constrictive pericarditis?

1. ECG may be nonspecific or show a. fib. or low-voltage QRS. 2. pericardial thickening & calcifications on imaging. 3. jugular venous pulse tracing shows prominent x & y descents.

What genetic syndromes have atrial septal defects?

1. Eisenmenger syndrome. 2. Holt-Oram (heart-hand) syndrome.

Anti-glomerular basement membrane disease. Pathogenesis? Symptoms?

Anti-GBM antibodies against collagen IV against alpha-5 chain. Leads to rapidly progressive glomerulonephritis or alveolar hemorrhage (pulmonary renal syndrome)

What symptoms should one associated with lacunar infarcts?

Any of the following. 1. pure motor hemiparesis. 2. pure sensory syndrome. 3. ataxic hemiparesis. 4. sensorimotor syndrome. 5. dysarthria-clumsy hand syndrome.

What laboratory findings are associated with alcoholic hepatitis?

1. Elevated AST & ALT (<300). 2. AST:ALT ratio of > 2.0. 3. Elevated GGT, ferritin, bilirubin and or INR. 4. neutrophilic leukocytosis. 5. low albumin. 6. abdominal imaging may show fatty liver.

Signs of right ventricular failure include what?

1. Elevated JVP. 2. Right ventricular 3rd heart sounds. 3. Tricuspid regurgitation murmur. 4. Hepatomegaly with pulsatile liver. 5. Lower-extremity edema, ascites, and or pleural effusions.

Platelet transfusion cut offs?

Anytime less than 10,000 administer or active bleeding with levels below 50,000.

Sudden steady epigastric pain or RUQ pain after large fatty meal. Positive Murphy's sign. Minor to no elevation of ALP.

Acute cholecystitis

Hematemesis in the setting of chronic and excessive aspirin use and heavy drinking. Diagnosis?

Acute erosive gastritis.

Sleeping sickness presentation?

Acute febrile illness with skin lesion. Myocarditis and neurological involvement.

To diagnose cholesterol crystal emboli, what can be done?

1. Elevated serum Cr, eosinophilia, hypocomplementemia. 2. Urinalysis showing eosinophiluria. 3. Skin biopsy showing perivascular inflammation with eosinophils. 4. Renal biopsy showing biconvex, needle-shaped clefts with occluded vessels.

What presentation is concerning for spontaneous intracerebral hemorrhage.

Acute focal neurologic deficit that gradually worsens over minutes to hours with developing symptoms of headache, vomiting and altered mental status due to elevated ICP.

HBV treatment indications?

1. acute liver failure. 2. clinical complications of cirrhosis 3. Advanced cirrhosis with high serum HBV DNA. 4. Positive HBeAG, HBV DNA >20,000 and ALT >2x normal upper limit for those without cirrhosis. 5. Prevent HBV reactivation during chemotherapy or immunosuppression.

What is the likely acid-base disturbance for the following: 1. allergic reaction with stridor? 2. asthma exacerbation?

1. acute respiratory acidosis. 2. acute respiratory alkalosis.

What symptoms of SCD are treated with blood transfusions?

1. acute stroke. 2. acute chest syndrome. 3. multiorgan failure. 4. acute symptomatic anemia. 5. aplastic crisis.

Causes of crystal-induced acute kidney injury

1. acyclovir. 2. sulfonamides. 3. methotrexate. 4. ethylene glycol. 5. protease inhibitor.

Treatment goals for mixed cryoglobulinemia includes what?

1. addressing an underlying hepatitis C infection 2. plasmapheresis to remove cryoglobulins 3. immunosuppressants.

What are the most important factors for improving patient survival in the setting of sudden cardiac arrest?

1. adequate bystander CPR. 2. Prompt rhythm analysis. 3. defibrillation in patients found in v. fib.

What are the 3 most important factors in improving patient survival?

1. adequate bystander CPR. 2. prompt rhythm analysis. 3. defibrillation.

What are the 5 criteria for Lyme disease prophylaxis?

1. adult or nymphal tick. 2. >36 hours attachment or engorged. 3. prophylaxis started within 72 hours of tick removal. 4. Local Borrelia infection rate >20%. 5. no contraindication to doxycycline.

Risks for postoperative urinary retention?

1. advanced age. 2. high fluid intake during surgery. 3. concomitant use of other meds like opiates and anticholinergics.

Risk factors for amniotic fluid embolism?

1. advanced maternal age. 2. gravida 5 or more. 3. c-section. 4. placenta previa or abruption. 5. preeclampsia.

What is the management for massive vatical bleeding?

1. aggressive fluid resuscitation, protection of airway and gastric decompression. 2. Control bleeding with terlipressin (ADH), octreotide, or somatostatin.

Goat milk is deficient in what vitamin?

Folate (associated goat milk with Mexican cooking and how tortillas are not fortified with folate among other vitamins).

Patient with macrocytic anemia and on seizure medicines. Diagnosis? What drug is doing this and how?

Folate deficiency. Phenytoin inhibits folate absorption.

Pathophysiology: who is follicular different than papillary when it comes to pathology?

Follicular is hematogenous while papillary spread via lymphatic dissemination. Also, follicular cancers are encapsulated.

What serious rare renal compilation can occur with cyclosporine use?

Hemolytic uremic syndrome

What is ranolazine and what does it treat?

Late sodium channel blocker. Stable angina patients with recurrent symptoms who are taking a combination of beta blockers, calcium channel blockers or nitrates. Definitely not first line.

Anytime a patient with COPD has findings of hyperinflation and loss of lung markings. What stage is their disease?

Late-stage.

Blastomycosis presentation?

Lungs: acute or chronic pneumonia. Skin: wartlike lesions, violaceous nodules, skin ulcers. Bone: Osteomyelitis. Genitourinary: Prostatitis CNS: Meningitis, epidural or brain abscess.

What is the initial management for patients with atrial fibrillation with rapid ventricular response? Patient is not in shock, confusion nor respiratory distress.

Rate control - beta blocker (metoprolol) or calcium channel blocker (diltiazem).

35 year old with wart like lesions and painless nodules on forearm and back of neck. Dry cough with malaise for 2 months. no other systemic symptoms. Is from Wisconsin. Afebrile, confirmed warty lesions with violaceous hue and demarcated borders. Wet prep of skin scrapping show yeast. Diagnosis? Treatment?

Blastomycosis dermatiditis

50 year old man with cirrhosis of the liver due to chronic HCV and ascites presents with slurred speech, lethargy and confusion. Past 3 days, sleeping more. VSS. Exam shows spider angiomas, asterixis, and shifting dullness. DTR are 3+. Diagnosis?

Hepatic encephalopathy. Serum ammonia concentration.

Transudative pleural effusion in patients with cirrhosis who have no underlying cardiac or pulmonary disease is called what?

Hepatic hydrothorax.

How does one reverse beta blocker and calcium channel blocker toxicity? Why?

IV glucagon as this increases the intracellular levels of cyclic adenosine monophosphate.

Management of acute limb ischemia in someone with an emboli after an MI?

IV heparin immediately upon suspicion. Surgical embolectomy or intra-arterial fibrinolysis or mechanical embolectomy via interventional radiology.

How are viral manifestations of the esophagus different from candida? Between CMV and HSV how are they different in their features in the esophagus?

Viruses cause ulcers in the esophagus. CMV cause larger ulcers than herpes ulcers

Most common adverse drug effect when treating Parkinson's Disease

Visual Hallucinations

How does one differentiate periorbital cellulitis from cavernous sinus thrombosis?

Periorbital cellulitis is an infection that does not extend into the sinus which would cause symptoms of ophthalmoplegia, headache, bilateral face and eye symptoms.

Itching after bathing, H&H elevated twice than normal, 550,000 platelet levels and RBC of 8. History of itching after bathing. Enlarged spleen. Diagnosis? What would you expect EPO levels to be? What are the patients at risk for?

Polycythemia vera. Low. Thrombosis.

What TCA can help with increasing appetite and weight gain in cancer patients?

Mirtazapine (Remeron)

Name all 4 causes of alcohol ingestion and describe how they are similar in laboratory results and make not of any exceptions.

1. alcohol ketoacidosis. 2. methanol ingestion. 3. ethylene glycol ingestion. 4. isopropyl alcohol ingestion. All of these causes have high osmolar gap. All but isopropyl alcohol ingestion have an increased anion gap metabolic acidosis

What drugs are associated with atrial fibrillation?

1. alcohol. 2. amphetamine. 3. cocaine. 4. theophylline.

Pregnant woman comes in with a history that sounds like nephrolithiasis, what procedures are contraindicated?

1. all forms of radiation are contraindicated. 2. shockwave lithotripsy is contraindicated.

Triad for Wernicke's encephalopathy?

1. altered mental status. 2. nystagmus. 3. ataxia

Complications of PEEP?

1. alveolar damage. 2. tension pneumothorax. 3. hypotension when high pressure is applied.

What are the acceptable antibiotics in a pregnant women for a UTI?

1. amoxicillin. 2. amoxicillin-clavulanate. 3. nitrofurantoin. 4. fosfomycin

What are the drug options in treating acute complicated pyelonephritis?

1. ampicillin-sulbactam 2. ticarcillin-clavulanate 3. piperacillin-tazobactam 4. meropenem 5. imipenem 6. aztreonam.

What laboratory values would suggest multiple myeloma?

1. anemia (Hb<12) 2. hypercalcemia. 3. Bence Jones proteins in urine. 4. monoclonal protein peak on electrophoresis. 5. punched out lesions, osteoporosis, or pathologic fractures on x-ray.

How is bronchiectasis different from chronic bronchitis?

Bronchiectasis is associated with recurrent respiratory tract infections and a cough that is mucopurulent in nature. Interstitial lung findings on auscultation are more common in bronchiectasis.

A subtype of adenocarcinoma that grows along the alveolar septa. Name? How does it appear on imaging?

Bronchoalveolar carcinoma. Nodule or lobar consolidation on imaging. (Should be suspected if a treated pneumonia doesn't get better).

34 year old with vague chest discomfort. Recent upper respiratory tract infection. Vitals are normal. Chest x-ray shows enlarged mediastinum. Helical CT scan shows mass in middle mediastinum. Likely diagnosis?

Bronchogenic cyst (these are benign)

Wheezing, murmur that increase with inspiration, and abdominal complaints.

Bronchospasms, right sided valvular complaints and GI complaints make it carcinoid syndrome.

Howell-Jolly bodies are present in what conditions?

Post-splenectomy or functional asplenia.

What features would you be suspicious of an underlying malignancy or granulomatous process for lymphadenopathy on physical exam?

Nodes greater than 2 cm that are firm and immobile. For cervical nodes if they belonged to an older patient or smoker, further workup would be necessary.

What patients will not need any follow up for a solitary pulmonary nodule?

Nodule size of less than 4 mm and low malignancy risk.

35 year smoke with known HIV infection, comes to urgent care with cough over 2 months. He is noncompliant with his anti-retroviral regimen. CD4 is over 200. normal vitals. Chest x-ray shows apical cavitary lesions on the right upper lung.

Reactivation tuberculosis.

Niacin (B3) deficiency causes what?

Remember "3D". 1. diarrhea. 2. dermatitis. 3. dementia.

What situations should you be suspicious with renovascular hypertension?

1. Elevation in serum creatinine > 30% from baseline after starting an ACEi or ARB 2. Severe HTN with recurrent flash pulmonary edema. 3. Severe HTN with diffuse atherosclerosis. 4. Severe HTN after age 55. 5. HTN in pt with asymmetric kidney size or a small atrophic unilateral kidney. 6. presence of abdominal bruit.

Extrahepatic complications of Wilson's disease?

1. Fanconi syndrome. 2. hemolytic anemia. 3. neuropathy

Three pathological stages of alcoholic liver disease?

1. Fatty liver (steatosis). 2. Alcoholic hepatitis. 3. Alcoholic fibrosis/cirrhosis.

Mononucleosis like symptoms but negative heterophile antibody test. Diagnosis?

CMV.

What are the 6 types of immunologic blood transfusion reactions?

1. Febrile nonhemolytic. 2. Acute hemolytic. 3. Delayed hemolytic. 4. anaphylactic. 5. urticarial/allergic. 6. Transfusion-related acute lung injury.

What settings on a mechanical ventilator influence pO2?

1. FiO2. 2. PEEP.

Patient presents with ethylene glycol poisoning. Treatment options and why?

1. Fomepizole - competitive inhibitor of alcohol dehydrogenase. 2. ethanol administration - adding substrate to compete with ethylene glycol for alcohol dehydrogenase. 3. sodium bicarbonate and possibly hemodialysis.

When is hydroxyurea indicated for SCD?

1. Frequent, acute painful episodes. 2. Acute chest syndrome. 3. severe symptomatic anemia.

Major complications of patients not on dialysis due to uremic coagulopathy include what?

1. GI bleeding. 2. hemopericardium. 3. subdural hematoma. 4. bleeding from surgical or invasive sites.

Adverse drug effects of erythromycin include.

1. GI upset. 2. cholestatic jaundice.

Large linear ulcers seen on endoscopy of an AIDS patient. Diagnosis and treatment?

CMV. Ganciclovir.

Pathogenesis of hypokalemic, hypochloremic metabolic alkalosis.

1. Generation phase - HCO3 is elevated from loss of acid, and this is reabsorbed from the GIT. 2. Maintenance phase - loss of intravascular volume leads to aldosterone which maintains alkalosis.

What are ways to distinguish renal transplant rejection versus cyclosporine toxicity?

1. Graft tenderness? 2. Do symptoms change with removal of drug?

HBeAg-negative chronic hepatits (reactivation). What will be present on hepatitis panel and liver biopsy.

1. HBV DNA elevated. 2. HBeAg negative. 3. ALT elevated. 4. chronic inflammation on liver biopsy.

How does excess of CO2 in the brain cause neurologic changes in consciousness. How about seizures?

CO2 increases GABA and glutamine while decreasing glutamate and aspartate. Seizures: CO2 causes cerebral vasodilation.

Neck movements or pressure by things like tight collars causes a syncopal episode. Diagnosis? What exam maneuvers can help confirm this?

Carotid sinus hypersensitivity syndrome. Carotid sinus massage, patient should develop asystole over 3 seconds or a fall in systolic blood pressure of greater than 50.

Immune clearance pattern for HBV on a hepatitis panel would demonstrate. What would show on pathology?

1. HBeAG being positive and gradual clears. 2. HBsAg positive 3. Elevated ALT (inflammation) 4. can have increased HBV DNA & IGM anti-HBc titer Pathology: acute liver inflammation with or without fibrosis on liver biopsy.

Define this, sharp pain localized to the carotid artery distribution of the neck.

Carotidynia

Inactive carrier state for HBV. What hepatitis panel findings are associated with this?

1. HBeAg negative and anti-HBe & HBsAg positive 2. HBV DNA low or undetectable. 3. Need at least 3 normal ALT levels and 2-3 normal HBV DNA tests in a year to confirm.

In what setting would an IVC filter be considered?

1. Hemorrhagic stroke. 2. active bleeding. 3. recurrent DVT despite anticoagulation.

What non-antihypertensive medications act on aldosterone's production or activity, leading to hyperkalemia?

1. Heparin. 2. Cyclosporine.

Other than factors revolving around sex, would other indicators would influence you to screen a patient for HIV annually?

1. Homeless shelter living. 2. correctional facility incarceration. 3. Intravenous drug use.

Sickle cell anemia is associated with what blood smear findings?

1. Howell-Jolly bodies. 2. Sickling

If working up for hypercalcemia and PTH, PTHrP and both forms of vitamin D are normal. What causes of hypercalcemia need to be ruled out? (7)

1. Hyperthyroidism. 2. Multiple myeloma. 3. adrenal tumor. 4. acromegaly. 5. vitamin A toxicity. 6. immobilization. 7. milk-alkali syndrome

40 year old woman with multiple complaints. Fatigue, weakness, anorexia, nausea, abdominal pain, and syncope. Hypotensive vital signs. Hyper pigmentation of the skin in the planar creases? Name 2 likely electrolyte abnormalities and the likely diagnosis.

1. Hyponatremia. 2. Hyperkalemia. Primary adrenal insufficiency (Addison's).

Muscle weakness, make a differential

1. Hypothyroid or hyperthyroid disease. 2. Cushing's syndrome. 3. Dermatomyositis, polymyositis. 4. Myasthena gravis, Lambert-Eaton syndrome. 5. ALS.

Define a hypertensive urgency.

"Severe hypertension" with no symptoms of acute end-organ damage. "Severe hypertension" is defined as a blood pressure above 180/120, with no symptoms of acute end-organ damage.

What is ehrlichiosis? Symptoms? CBC results?

"Spotless rocky mountain spotted fever" - fever, malaise, headache, nausea and vomiting. Not common to have jaundice or hemolysis. Labs will show leukopenia and thrombocytopenia.

Patient presents with dizziness and pulse of 40. What is the management for this patient?

1. ID and treat underlying causes. 2. IV access, cardiac monitoring to identify rhythm. 3. pulse oximetry with oxygen for hypoxemia. 4. 12-lead ECG if possible.

What adjunctive medications are used to provide symptomatic treatment in alcohol withdrawal?

1. Chlordiazepoxide. 2. Clonidine. 3. antiemetics such as promethazine, diphenhydramine 4. loperamide. 5. octreotide (diarrhea and cramps)

How can drug-induced liver disease categorized by pathology findings? What drugs correlate with each subcategory?

1. Cholestasis - chlorpromazine, nitrofurantoin, erythromycin and anabolic steroids. 2. Fatty liver - tetracycline, valproate, and anti-retrovirals. 3. hepatitis - halothane, phenytoin, isoniazid, and alpha-methydopa. 4. toxic of fulminant liver failure - carbon tetrachloride and acetaminophen. 5. Granulomatous - allopurinol and phenylbutazone.

What are risk factors for pigment gallstones?

1. Chronic hemolysis (sickle cell). 2. Chronic biliary tract infection or parasite infestation. 3. Advance age.

Pulmonary side effects of amiodarone include?

1. Chronic interstitial pneumonitis. 2. organizing pneumonia. 3. ARDS.

Ct demonstrates liver metastasis. What are the 3 most common cancers to cause this?

1. Colon cancer. 2. Lung cancer. 3. breast cancer.

What 3 settings leads to cyanide toxicity?

1. Combustion of carbon and nitrogen-containing compounds (wool, silk). 2. industrial exposure (metal extraction/mining) 3. medications (sodium nitroprusside).

In broad categories, what are the causes of pulmonary-renal syndrome?

1. Connective tissue disorders. 2. Goodpasture syndrome. 3. Renal disorders. 4. systemic vasculitis. 5. drugs (PTU). 6. Heart failure.

What ocular side effects are associated with amiodarone?

1. Corneal micro-deposits. 2. optic neuropathy.

Name the 2 classic findings of pancreatic carcinoma and what they are.

1. Courvoisier's sign - nontender but palpable gallbladder at right costal margin in a jaundiced patient. 2. Virchow's node - left supraclavicular adenopathy.

Name 5 pathologies that cause bile salt-related disturbances that lead to steatorrhea

1. Crohn's disease of small bowel. 2. bacterial overgrowth. 3. primary biliary cirrhosis. 4. primary sclerosing cholangitis 5. surgical resection of ileum. *Overall bottom line - obstruction or edema of biliary ducts. Otherwise, bacteria deconjugate bile salts and lack of ileum leads to chronic loss of bile. Crohn's can cause biliary cirrhosis.

What patients are at risk of succinylcholine-induced hyperkalemia?

1. Crush injuries. 2. Burns. 3. Demyelinating diseases such as Guillain-Barre. 4. tumor lysis syndrome.

Name 5 infectious disease etiologies that cause diarrhea for longer than 2 weeks.

1. Cryptosporidium. (i) 2. Cystoispora. (i) 3. Microsporidia. (i) 4. Cyclospora. 5. Giardia Note: those with (i) means it occurs with immunodeficiency.

55-year old present for follow up for high blood pressure noted 3 prior visits over the past 6 months. Pressure of 150 over 95. 25-oack-year smoking history, 3 alcoholic beverages a day. BMI of 22. Unremarkable exam. What are the most appropriate lifestyle modification should this patient implement. Name the top two lifestyle changes that leads to the most significant drop in systolic blood pressure?

1. DASH diet. 2. exercise. 3. dietary sodium restriction. 4. alcohol intake reduction. 5. Weight loss (if over 25 BMI) Blood pressure drops from 5-20 mm Hg for every 10 Kg lost while a DASH diet reduces blood pressure by 8-14 mm Hg. Note: exercise and dietary sodium are next.

What are some well-known side effects of thiazide diuretic therapy?

1. Decreased tolerance to glucose predisposing patients to hyperglycemia. 2. Increased LDL cholesterol and plasma triglycerides. 3. Uric acid retention and risk of acute gout arthritis. 4. Hyponatremia, hypokalemia, and hypercalcemia.

Name 4 common disease that cause polyneuropathy.

1. Diabetes mellitus. 2. vitamin b12 def. 3. GBS. 4. chronic inflammatory demyelinating polyneuropathy.

Precipitating factors of hepatic encephalopathy?

1. Drugs (sedatives, narcotics). 2. Hypovolemia (diarrhea, paracentesis, diuretics). 3. Excessive nitrogen load (GI bleeds, constipation, high protein). 4. Hypokalemia and metabolic alkalosis. 5. hypoxia and hypoglycemia. 6. infection (PNA, UTI, spontaneous bacterial peritonitis). 7. portosystemic shunting.

What are ways to remove potassium out of the body?

1. Duretics. 2. cation exchange resins. 3. hemodialysis.

What autoimmune conditions are associated with autoimmune primary adrenal insufficiency (Addison's disease) ?

1. Thyroid disease. 2. parathyroid disease. 3. ovaries. 4. pernicious anemia. 5. vitiligo.

Causes of high-output cardiac failure include?

1. Thyrotoxicosis. 2. Paget disease. 3. anemia. 4. AV fistula. 5. thiamine deficiency.

What conditions would sodium bicarbonate be useful for?

1. Torsades due to quinidine use. 2. metabolic acidosis. 3. hyperkalemia. 4. tricyclic antidepressant overdose.

What laboratory values are typical for acute liver failure?

1. Transaminases elevated 10 times higher than normal. 2. Elevated bilirubin, PT, and INR. 3. Pattern of worsening PT/INR and bilirubin.

Burr cells are due to what usually?

1. Uremia. 2. artifact of preparation.

What GU symptoms does tolterodine treat?

1. Urge incontinence. 2. overactive bladder.

Causes of acute liver failure?

1. Viral (HAV, HBV) 2. Drugs (acetaminophen) 3. Alcoholic or autoimmune hepatitis. 4. Wilson disease, Budd-Chiari syndrome. 5. Ischemia, malignant infiltration of liver.

What are rare causes of steatorrhea. Hint: I'm not interested in the following 3 causes: pancreatic insufficiency, bile salt disturbances or impaired intestinal surface epithelium.

1. Whipple disease. 2. Zollinger-Ellison Syndrome. 3. Medications

What defines a premature ventricular complex on ECG?

1. Widened QRS. 2. Bizarre morphology. 3. compensatory pause

What parameters ought to be present prior to switching a patient to SQ insulin from IV when correcting DKA?

1. able to eat. 2. glucose less than 200. 3. anion gap closed. 4. bicarb over 15.

What are the reasons for dialysis?

1. acidosis. 2. electrolyte disturbances (mainly potassium) 3. Intoxication (methanol, ethylene glycol, metals, ASA) 4. Overload of fluids (esp. when there is pleural edema) 5. Uremia *Dialysis will fix the electrolyte disturbances of hyperkalemia; however, between the time from symptom to dialysis you need to buffer this with calcium gluconate, sodium bicarbonate, D50, insulin, and kayexalate (sodium polystyrene).

What 2 ways does renal vein thrombosis present as in nephrotic patients?

1. acute abdominal pain, fever and hematuria. 2. gradual worsening renal function and proteinuria in an asymptomatic patient.

What is first line for Myasthenia Gravis? Then second line? How about myasthenia crisis?

1st line: pyridostigmine. 2nd immunotherapy such as glucocorticoids, azathioprine, mycophenolate mofetil & cyclosporine. Crisis: plasmapheresis or IVIG

What is the first line treatment of persistent/moderate to severe symptoms of restless leg syndrome? How about second line?

1st: Dopamine agonists (pramipexole) 2nd: alpha-2-delta calcium channel ligands (gabapentin enacarbil)

How often are placental abruptions visible on ultrasound.

25% of the time, hence this is a clinical diagnosis.

What 3 things are required for a diagnosis of acute pancreatitis?

2 of the following needs to be present... 1. acute epigastric abdominal pain often radiating to the back. 2. Amylase or lipase 3 times higher than normal limit. 3. abdominal imaging showing focal or diffuse pancreatic enlargement on CT or diffusely enlarged and hypoechoic pancreas. Bottom line: pain, enzymes, and imaging

how to calculate serum osmolarity?

2 times sodium plus BUN divided by 2.8 plus glucose divided by 18. For practicality, remember 3 and 20.

What equation calculates serum osmolality?

2 times the sodium plus glucose.

Prothrombin time is elevated what coagulation factors are most likely to be deficient in a patient?

2, 7, 9, 10 Remember we monitor warfarin on the basis of INR (modified PT calculation). Therefore remember what warfarin inhibits.

How much energy is needed to adequately defibrillate a patient?

200-360 joules.

What are common causes of postpartum secondary adrenal insufficiency? How about postpartum primary?

2nd: Sheehan syndrome and lymphocytic hypophysitis. 1st: autoimmune adrenalitis.

Papillary muscle rupture presents how?

3 to 5 days after a MI with an acute onset of hypotension due to severe acute mitral regurgitation. Findings of a holosystolic murmur at the apex. Pulmonary edema. Not likely to cause rapid decompensation in patients.

Seizure and hyponatremia of 115. Treatment?

3% saline at a rate that does not exceed 0.5 mEq/L/hr

What classifies as mild hypothermia? Treatment?

32- 35 C (90-95F) Tachycardia, tachypnea and hyperventilation. Confusion, ataxia, dysarthria, increased shivering. IV fluid support and passive rewarming.

What is the age range of granulomatosis with polyangiitis? What do you treat with?

35-50 years. cyclophosphamide.

Which parts of the patient presentation correlate with the following scores on the Well's criteria: 3, 1.5, and 1?

3: clinical signs of DVT and alternate diagnosis less likely. 1.5: previous PE or DVT, heart rate over 100 and recent surgery or immobilization. 1: hemoptysis and cancer.

What is a normal bladder capacity? What range is a normal post-voidal residual volume?

400-600 mL. Post-voidal of less than 50 mL.

Mean onset of benign essential tremors?

45 years of age.

70 year old man with severe pain in back of neck and upper chest. Difficulty walking due to bilateral leg weakness for past 2 hours. Past medical includes hypertension, bleeding peptic ulcers, and deep vein thrombosis requiring inferior vena lava filter placement. Alcoholic. Blood pressure of 210 over 120 and pulse of 120. X-ray shows right-sided pleural effusion. ECG shows sinus tachycardia.

Aortic dissection -

When do left ventricular aneurysms occur after a myocardial infarction?

5 days to 3 months.

What percentage of patients with a brain abscess will have fever?

50%

What are acceptable FiO2 levels to prevent oxygen toxicity.

50-60%

What exam findings are tested for in patients with brain death?

Apnea. Oculovestibular reaction.

What is the general normal A-a Gradient?

< 15

What triggers are associated with vasovagal syncope in someone younger than 60 versus those older than 60?

< 60 years: emotional or orthostatic stress such as venipuncture, prolonged standing, heat exposure or exertion. > 60 years: micturition, cough and defecation.

What classifies as severe hypothermia? What is the treatment?

<28 C (82F). Symptoms of coma, cardiovascular collapse, ventricular arrhythmias, pulmonary edema. Active external and internal rewarming such as heated blankets and heated iv fluids

Generally speaking, what is the normal QT interval in male and females? How does one measure QT?

<450 ms Start of Q wave to end of T wave.

What abnormal value defines urinary retention?

> 100 mL of urine on bladder scan.

Horses or Egypt in patient history, altered mental status, fever and focal neurologic deficits without nuchal rigidity or signs of meningitis.

Arbovirus which include eastern & western equine virus and West Nile virus.

Presentation of syringomyelia?

Areflexic weakness in the upper extremities and dissociated sensory loss following a cape distribution.

What neurologic disease is associated with syringomyelia?

Arnold Chiari malformation type 1.

Non-vaccinated patient with erythematous blanching maculopapular rash that spread from head to toe and spares the hands or feet, posterior cervical lymphadenopathy and mild joint pains. Rash had spread within 24 hours. Diagnosis?

Rubella

Egophony, bronchophony or whispered pectoriloquy all suggest what?

A consolidative process.

35 year old man from Wisconsin with fever, night sweats productive cough and weight loss. Exam shows multiple, well-circumscribed, verrucous, crusted lesions. Chest x-ray shows left upper lobe consolidation and two lytic lesions in the anterior ribs. Diagnosis? Test to confirm? Treatment?

Blastomycosis. Broa-based budding yeasts on sputum culture. Itraconazole and amphotericin B.

Demeclocycline. What is it?

A treatment for SIADH. It inhibits ADH-mediated aquaporin insertion.

What serum sodium level defines hyponatremia?

A value less than 130 mEq/l

What A-a gradient would indicate a disease not related to hypoventilation or reduced inspired oxygen?

A-a gradient above 30.

30 year old woman presents with occasional headaches and palpitations. 170 over 100 blood pressure. Bilateral flank masses. creatinine of 1.3. 12 red blood cells per high power field and no other abnormalities on urinalysis? Diagnosis?

ADPKD

What renal disease is associated with subarachnoid hemorrhages?

ADPKD.

How is acute fatty liver of pregnancy different that HELLP

AFLP tends to have extrahepatic complications such as leukocytosis, hypoglycemia and acute kidney injury.

What is Wernicke's encephalopathy?

AMS, gait instability, nystagus and gaze palsy due to thiamine deficiency due to long-term alcohol abuse.

What is the range of neuropathy with Diabetes?

Broad... 1. symmetric peripheral neuropathy. 2. mononeuropathy, such as somatic or cranial (most common cranial is CN III). 3. autonomic neuropathy

With regards to rate control, where do beta blockers and calcium channel blockers affect the heart?

AV node. They increase the refractory period which increases the PR interval.

Easiest way to confirm an amyloidosis diagnosis?

Abdominal fat pad biopsy.

When is a combination of ampicillin and gentamicin used in the clinical setting?

Abdominal infections especially in combination with metronidazole.

Presentation of an interventricular wall rupture?

About 5 days after a MI, findings of sudden hypotension, congestive heart failure (predominantly right), and a loud holosystolic murmur heard best at the lower left sternal border.

What ESR level would suggest temporal arteritis?

Above 50.

Flow cytometry of a patient with paroxysmal nocturnal hemoglobinuria would show what?

Absence of CD55 and CD59.

Sulfur granules or granular yellow pus

Actinomyces

Patient is suspected of having cyanide poisoning from ingesting the toxin. What do you do?

Activated charcoal. Hydroxocobalamin or sodium thiosulphate. Nitrites if unavailable.

How does heparin work?

Activates antithrombin III with inactivates Factors IIa (thrombin), IXa and Xa.

60 year old with long history of hypertension collapses at church. Severe chest pain that radiates to her back. Pressure of 90/50 and pulse of 110. JVD present. Widening of mediastinum on x-ray. Intra-arterial catheter shows significant variation in systolic blood pressure related to respiratory cycle. Diagnosis?

Acute aortic dissection complicated by cardiac tamponade.

young nonsmoker developed cough productive of purulent sputum after an upper respiratory infection. Mild wheezing on exam. Rhonchi that clear with cough. likely diagnosis?

Acute bronchitis.

Presentation of right ventricular infarction?

Acute cariogenic shock during the MI due to occlusion of the RCA. Exam findings of JVD, and clear lung fields. ECG findings of inferior MI and or ST elevation in V4R to V6R.

Patient has ulcerative colitis. What is his colonoscopy screening recommendations?

After having a diagnosis for 8 years, patient should get a colonoscopy now and every year afterwards.

For vaccinated adults with pneumococcal vaccine. What age categories do you divide them into? Which of these patients get both vaccines or just PPSV23 alone?

Age recommendations are broken down into 2 categories. Of the two those 19-64 are broken down by diseases. The following are the categories and conditions that warrant certain vaccine combinations. Ages 19-64 who receive PPSV23 alone Chronic heart, lung or liver disease. Diabetes, Smokers, alcoholics. Ages 19-64 who receive PCV13 and PPSV23 CSF leaks, cochlear implants. Sickle cell, asplenia. Immunocompromised (HIV, cancer) Chronic kidney disease. Ages > 65 receive PCV13 & PPSV23

For any medication that can crystalline nephropathy what is the best preventative measure in avoiding this complication?

Aggressive IV hydration.

What is the pathogenesis for isolated systolic hypertension? What is the treatment for it?

Aging causes loss of elasticity in the arteries. 1. low dose thiazide. 2. ACEi. 3. long-acting calcium channel blocker.

Neurocysticercosis, treatment.

Albendazole.

What pattern of LP finding is associated with GBS?

Albuminocytologic dissociation, elevated protein with nL WBC.

Slurred speech, unsteady gait, altered mentation in the setting of alcohol? What laboratory results are expected from this diagnosis?

Alcohol Ketoacidosis. 1. High osmolar gap. 2. increased anion gap metabolic acidosis due to ketosis.

AST of 120 and ALT of 90. Goto diagnosis?

Alcoholic hepatitis as AST is usually 1.5 or 2 times more than ALT.

Alcoholic patient presents with fever, ascites, abdominal pain and peripheral leukocytosis. Paracentesis shows PMN of 100. Diagnosis? What would the diagnosis be if PMN was 300?

Alcoholic hepatitis. Spontaneous bacterial peritonitis.

50 year old presents with jaundice and malaise. Smoker and drinker. Bilirubin of 3, direct of 1.2. AST of 150 and ALT of 50. ALP normal. Diagnosis?

Alcoholic liver disease. AST to ALT ratio of great than 2:1)

Patient has evidence of dilated cardiomyopathy, thrombocytopenia, macrocytosis and elevated transaminases. Likely etiology of cardiomyopathy?

Alcoholism.

Any time a question asks about what would happen with electrolytes and pH due to vomiting, what hormone must you think about as to answer the question correctly?

Aldosterone

What is spironolactone? What does it treat?

Aldosterone antagonist. 1. Ascites from cirrhosis. 2. symptomatic patients with moderate-to-severe systolic congestive heart failure.

Does ventricular fibrillation spontaneously terminate?

Almost never.

Patient presents with kidney stone of 5mm. Other than hydration and analgesics what else can be done?

Alpha blockers.

Panacinar emphysema and elevated AST and ALT. Diagnosis? Best next step.

Alpha-1 anti-trypsin deficiency. Biopsy and staining with PAS reagent.

Which thalassemia is associated with a normal hemoglobin electrophoresis study?

Alpha. Beta has an increased hemoglobin A2.

Hearing loss, ocular abnormalities, hematuria, and progressive renal insufficiency. Diagnosis and pathogenesis?

Alport syndrome. X-linked defect in collagen-IV formation.

How is Alzheimer's different from Lewy bodies dementia?

Alzheimer's presents mainly with problems with memory issues initially while Lewy bodies presents with issues in alertness and hallucinations initially.

Parkinson's patient with livedo reticularis. What drug causes this side effect?

Amantadine.

Gait imbalance and decreased hearing after being given antibiotics for an enterococcal wound infection. What is the drug toxicity?

Aminoglycoside toxicity, like gentamicin.

What anti arrhythmic should not be used in patients with lung disease? Why?

Amiodarone for it causes the following. 1. chronic interstitial pneumonitis. 2. organizing pneumonia. 3. ARDS.

Pregnant woman with Lyme disease, treatment and why? Any other patient populations that need a special treatment regimen?

Amoxicillin should be given to pregnant women with Lyme Disease. Doxycycline (think tetracycline) causes teeth discoloration and skeletal retardation. Lactating women and Children under 8 years old who have Lyme disease should receive amoxicillin instead of doxycycline.

60 year old with progressively worsening exertional dyspnea and new-onset ankle swelling. Recently evaluated for proteinuria and easy bruising. JVD. Scattered bibasilar crackles. Small pericardial effusion. Echo shows small pericardial effusion, concentric thickening of ventricular walls, normal ventricular chamber dimensions and diastolic dysfunction. Likely diagnosis.

Amyloidosis. (Pay attention to the proteinuria history).

50 year old with excessive wasting of extremity muscles which began distally and asymmetrically. Recent weakness with swallowing, chewing and speaking. Patient has muscle stiffness. Physical exam reveals excessive wasting of muscles which is more prominent in lower extremities. Fasciculations and hyperreflexia of all extremities are noted. Bulbar reflexes are decreased. Diagnosis and why?

Amyotrophic lateral sclerosis. Lower and upper motor neuron involvement.

Progressive motor deficits. Hyperreflexia, fasciculations, spasticity and weakness. What sign is muscular finding is not associated with this disease?

Amyotrophic lateral sclerosis. Not associated with myoclonus.

In the setting of someone less than 50 years of age who presents with bright red blood per rectum and has no history of colon cancer. Next best step?

Anoscopy.

Pralidoxime, what is it?

Another antidote that works like atropine for the treatment of organophosphate poisoning.

How does methotrexate and trimethoprim cause folate deficiency?

Antagonizes it's physiologic effects.

For GBS what history is typical for these patients?

Antecedent respiratory or gastrointestinal infection.

Locate the lesion: a lacunar infarct that causes ataxic-hemiparesis.

Anterior limb of the internal capsule.

What antibodies are associated with Hashimotos thyroiditis?

Anti

What type of bacteria is Actinomyces? Treatment?

Anaerobic Gram-positive and filamentous branching. Penicillin for 12 weeks.

What is phenazopyridine used for? What urinary complication is associated with this?

Analgesia for urinary tract mucosa, used to treat dysuria cystitis after infection or instrumentation. Renal stone formation.

25 year old woman undergoing cholecystectomy. Procedure is converted into an open surgery due to difficult visualization. Patient is hypotensive now and has a diffuse rash. 2 months ago, she developed hives after having protected sex with her partner fort he first time. Likely diagnosis?

Anaphylactic shock due to latex allergy.

Rapidly enlarging thyroid mass in an old patient. Type of cancer histology? What is the prognosis.

Anaplastic thyroid carcinoma. Poor.

Patient has a carotid stenosis of 45% what do you do?

Annual Duplex ultrasound.

What are symptoms of hypercalcemia?

Anorexia, nausea, constipation, polyuria, polydipsia and dehydration.

Autoimmune hemolysis. Pathogenesis? Diagnostic test? What would be present on blood smear?

Anti-RBC membrane IgG antibodies. Postive direct Coombs test. Spherocytes.

"red as a beet, dry as a bone, hot as a hare, blind as a bat, mad as a hatter, and full as a flask"

Anti-muscarinic effects and associated symptoms. Flushing, anhidrosis, hyperthermia, mydriasis/vision changes, delirium/confusion and urinary retention/constipation.

What antibodies are associated with Hashimoto's thyroiditis?

Anti-thyroid peroxidase.

How does infliximab work? What diseases does it treat?

Anti-tumor necrosis factor agent. 1. rheumatoid arthritis. 2. psoriasis. 3. ankylosing spondylitis. 4. Crohn's disease.

Treatment for acalculous cholecystitis in a critically ill patient.

Antibiotics and percutaneous cholecystostomy followed by cholecystectomy when the patient is stabilized.

What is the pathophysiology of Lambert-Eaton syndrome? How does one diagnose this condition? What is the treatment?

Antibodies to voltage gated calcium channels. Electrophysiology studies. Plasmapheresis and immunosuppressive drug therapy.

TCA overdose. Symptoms? Treatment?

Anticholinergic toxicity (think excess sympathetic response) 1. hyperthermia. 2. dilated pupils. 3. intestinal ileus. 4. urinary retention. 5. cardiac toxicity. Treat with sodium bicarbonate when QRS complex is greater than 100 msec.

What is flucytosine used for?

Antifungal for Cryptococcus and Candida.

Name some antihistamines and 5HT3 antagonist that help control nausea.

Antihistamines - diphenhydramine, promethazine. 5HT3 antagonists - ondansetron, granisetron.

Recurrent arterial or venous thrombosis, neurologic findings, microangiopathic hemolytic anemia, and loss of pregnancy.

Antiphospholipid antibody syndrome.

What lung diseases are beta blockers contraindicated in?

Asthma and COPD.

With regards to COPD and Asthma how are the disease different in terms of DLCO?

Asthma can present with normal or increased DLCO. Chronic bronchitis is normal. Emphysema is low (b/c of alveolar damage)

Pt has wheezing, coughing and SOB. PFTs show FEV1 of 65% and FEV1/FVC of 65%. With a bronchodilator FEV1 increases to 80% and FEV1/FVC of 80% Diagnosis and why?

Asthma. Obstructive PFTs and > 12% increase in FEV1 after bronchodilator use.

What are the indications of carotid artery endarterectomy for both men and women?

Asymptomatic men > 60% stenosis or men. Symptomatic men > 50% Women (symptomatic or asymptomatic) > 70% stenosis.

What patients have a low pretest probability of coronary artery disease.

Asymptomatic people of all ages. Atypical chest pain in women less than 50 years old.

60 year old presents with 4 hours of chest pain that is substernal and radiates to the left arm. History of hypertension, diabetes and high cholesterol. STEMI on ECG with several ventricular premature beats. Patient is accidentally given lidocaine. What is the patient at risk for?

Asystole. Lidocaine was used as prophylaxis against ventricular fibrillation however evidence shows no change in prognosis.

What is the ADE of lidocaine? what class of antiarrhythmics does it belong to?

Asystole. Class IB

Treatment regimen for confirmed brain abscess requires what?

At least 4-8 weeks of antibiotic therapy and aspiration and drainage of the abscess.

How does lithium cause nephrogenic diabetes inspidus? What symptoms do these patients experience?

At the collecting duct, lithium induces ADH resistance by impairing water reabsorption. 1. acute-onset nocturne. 2. polyuria. 3. polydipsia.

What medicine can help prevent the muscarinic side effects of anticholinesterase therapy?

Atropine (anticholinergic agent).

Other than seizures what can carbamazepine be used for?

Atypical bipolar depression and trigeminal neuralgia

What type of diseases are associated with crescentic glomerulonephritis

Autoimmune disorders.

Other than splenic rupture, what other complication is associated with infectious mononucleosis? Describe it's pathogenesis.

Autoimmune hemolytic anemia and thrombocytopenia which is due to cross reactivity of EBV antibodies against RBCs and PLTs. IgM cold-agglutinin antibodies (anti-i antibodies) cause complement-mediated destruction of red blood cells (Coombs' test positive)

Patient has infectious mononucleosis, what is management for the teenager in question?

Avoid contact sports for at least 3 weeks until all symptoms resolve.(not the size of the spleen but the symptoms).

Treatment of disseminated mycobacterium avid is?

Azithromycin.

For patients with cirrhosis who are resistant to diuretic use and unable to mobilize ascites what are these patients at risk for?

Azotemia and electrolyte disturbances (hypokalemia) which leads to hepatic encephalopathy

locate the lesion: a lacunar infarct that causes dysarthria-clumsy hand syndrome.

Basis pontis

Why is it hypomagnesemia causes the same symptoms as hypocalcemia and vice versa?

Because magnesium decreases PTH secretion and decreases referral responsiveness to PTH.

Why is it important to treat systemic infections while a woman is pregnant?

Because the inflammatory response induces preterm labor.

Patient opts to receive BRCA testing and is positive. What are acceptable screening options to do for ovarian cancer?

Because the patient is at increased risk she could consider

What exam finding with regards to location of crackles and wheezes would suggest CHF over COPD?

Bibasilar crackles and wheezes rather than diffuse.

What is significant about a bicarbonate level of less than 8?

Bicarbonate of less than 8 s profound anion gap metabolic acidosis. Few conditions cause this. 1. ethylene glycol ingestion. 2. diabetic ketoacidosis. 3. lactic acidosis.

30 year old man comes for a 2 week follow up visit after an ER visit for epistaxis requiring anterior nasal packing. Pressure at the time was 170/110. Pressure today is 180/110 and pulse of 80. No bruits on abdomen. ECG shows high-voltage QRS complexes, down-sloping ST-segment depression and T-wave inversion in leads V5 and V6. Hemoglobin, platelets and creatinine are normal. Next best step and why?

Bilateral arm and leg blood pressure measurements. Assessing for aortic coarctation.

Late signs of spinal cord compression?

Bilateral babinski reflex, decreased rectal sphincter tone, paraparesis or paraplegia with increased deep-tendon reflexes and sensory loss.

Define cauda equina syndrome.

Bilateral, severe radicular pain. Saddle anesthesia, asymmetric motor weakness, hyporeflexia or areflexia and late-onset bowel and bladder dysfunction.

Pulsus bisferiens what is this? What patients have this?

Biphasic pulse on exam. 1. aortic regurgitation. 2. hypertrophic obstructive cardiomyopathy. 3. large patent ductus arteriosus.

Presentation of psittacosis?

Bird exposure and symptoms of fever, dry cough and headache.

The most common cancer cause of hematuria in a smoker at 60 year old is?

Bladder cancer. 1. > 35 years of age. 2. smoking history. 3. occupational history (chemicals, dyes). 4. drug exposure (cyclophosphamide).

40 year old presents with hematuria at the end of voiding and is otherwise asymptomatic. What organ is likely the source of blood? How about if blood started at the beginning of voiding? How about if it was red throughout start to finish? If blood clots are visible what can be ruled out?

Bladder or prostate (terminal hematuria). Urethra (initial hematuria). Kidney or ureter (total hematuria). Blood clots rule out a renal cause. When comparing bladder and urethra take notice of the distal vs proximal relation of in relation to timing of the urine.

In terms of disseminated disease in a non-immunocompromised patient how is blastomycosis different from histoplasmosis?

Blastomycosis can cause disseminated disease in healthy individuals

How does one differentiate nonallergic rhinitis from allergic rhinitis?

Non allergic rhinitis has no identified triggers with symptoms of nasal congestion, stuffiness or postnasal drip. Allergic rhinitis has specific triggers and as well as eye symptoms, itching and sneezing.

How does shigella usually present?

Bloody diarrhea with fever and bacteremia. Often due to contaminated food or water or travel outside the USA.

Smooth-surfaced, dome-shaped melanocytic papules that develop from macules and tend to be less than 1 cm. Diagnosis?

Blue nevi.

Severe symptoms of bradyarrhythmia are treated with what?

Bolus IV atropine (0.5 mg), repeat every 3-5 minutes up to 3 mg max.

Is hyper or hypothyroidism associated with myopathy?

Both cause proximal muscle weakness.

How are COPD and pneumothorax different and alike on exam findings?

Both have decreased breath sounds and tactile fremitus and can present as hyperresonant on percussion. Mediastinal shift is associated with pneumothorax.

What diseases are associated with low renin and low aldosterone in the setting of a patient with HTN and hypokalemia?

Bottom line: non-aldosterone causes. 1. CAH. 2. Deoxycorticosterone-producing adrenal tumor. 3. Cushing syndrome. 4. exogenous mineralocorticoids.

Headaches, focal neurologic deficit, solitary ring-enhancing lesion on brain CT scan and fluid collection in an ethmoid sinus. Diagnosis?

Brain abscess.

What is trastuzumab used for? What adverse drug effect is associated with it and needs monitoring how?

Breast cancer positive for HER2. Cardiotoxicity (do an echocardiogram to monitor).

Presentation of benign paroxysmal positional vertigo?

Brief and recurrent episodes of feeling of the room spinning when turning the head in various directions. Nystagmus and nausea (rarely vomiting) without significant ear pain, tinnitus or hearing loss.

Clostridium perfringens presentation?

Brief watery diarrhea, cramps and fever associated with undercooked or unrefrigerated food.

CHOP regimen is used for what cancer?

Non-hodgkin Lymphoma.

Fomepizole vs flumazenil what the heck do these similar sounding drugs do?

Fomepizole treats methylene and ethylene glycol poisoning. Flumazenil treats benzodiazepine overdose.

When should a physician order a urine culture in the outpatient setting when the given history is sounds like a cystitis diagnosis?

For patients who fail initial therapy.

Of the leukemias, which does chlorambucil and prednisone treat?

CLL

Smudge cells.

CLL

BCR-ABL protein. Diagnosis?

CML

What pathogen is usually responsible for retinitis in an HIV patient?

CMV

Pt presents with constipation. Distended abdomen and tenderness in RLQ. Upright x-ray shows gas throughout large and small bowel with some fluid levels. NGT works with decompression. Urinalysis shows red blood cells 15/HPF and needle shaped crystals. Next appropriate step?

CT of abdomen or IV pyelography. Note: although uric acid stones are not seen on x-ray the combination of 3D x-ray will show the uric acid stones.

Thought question: patient is suspected to have a massive PE. What options are on the table for diagnosing this patient?

CT pulmonary angiography if there is time. Echocardiogram for bedsided diagnosis.

What is the best imaging to detect renal stones?

CT scan

Patient presents with uncomplicated pyelonephritis and is treated appropriately with IV antibiotics. Patient does not get better on the third day. Next best step?

CT scan of abdomen to evaluate for pathologies such as abscess, renal corticomedullary abscess, emphysematous pyelonephritis, or papillary necrosis.

Best next step in a patient with apparent unprovoked first seizure?

CT without contrast or MRI

If an unprovoked first seizure and a fall, would MRI or CT be better?

CT, as it can help rule out hemorrhage.

Pt has had recurrent pneumonias in the same lobe confirmed by X-rays. Next best step. If negative than whats the next step after that?

CT. Bronchoscopy.

Cyclosporine-induced hypertension. Treatment?

Calcium channel blockers.

After a thyroidectomy what monitoring is typical?

Calcium levels.

Rectangular, enveloped-shaped crystals. Likely diagnosis?

Calcium oxalate crystals. Ethylene glycol poisoning.

Child or young adult with abdominal pain and blood diarrhea. Raw or undercooked meat. Likely diagnosis?

Campylobacter

White plaques seen on endoscopy of an AIDS patient. Diagnosis and treatment?

Candida. Fluconazole.

Knife-like pain of the face that is paroxysmal. Occurs 10-20 times and last a few seconds. First line therapy for this disease?

Carbamazepine is first line for trigeminal neuralgia.

70 year old female pt with short-bowel syndrome who developed symptoms of ataxia, confusion and dysarthria shortly after lunch. What is the dx and what happened?

Carbohydrate loading led to a patient having D-lactic acidosis.

Group of teenagers attend an indoor barbecue on a cold winter night. All of them present to the ER with headache, nausea, vomiting, and vague abdominal discomfort and confusion. Exams show tachycardia, tachypnea and a pinkish-skin hue.

Carbon monoxide poisoning.

40 year old patient presents with a 1-year history of diarrhea. Stools are watery and accompanied by abdominal cramps. No blood or fat in stools. Symptoms of dizziness, flushing, wheezing and feeling warm. 2/6 systolic murmur over the left lower sternal border that increases with respiration. Hepatomegaly present and no abdominal tenderness. Diagnosis? and Vitamin deficiency at risk?

Carcinoid syndrome. Niacin (B3).

What is the purpose of IV calcium gluconate?

Cardiac membrane.

60 year old with left-sided weakness. Increased fatigue, low-grade fevers, and occasional palpations over last 3 months. Exam show normal S1 and S2 and a mid-diastolic rumble at the apex. Transthoracic echo shows a mass in left atrium. Diagnosis? Treatment?

Cardiac myxoma. Prompt surgical resection.

What disease cause Beck's Triad? Define the triad.

Cardiac tamponade. 1. hypotension. 2. distended neck veins. 3. muffled heart sounds.

Most common cause of death in dialysis patients is due to? What is noteworthy about the clinical problem in relation to this outcome?

Cardiovascular disease notably due to myocardial infarction or sudden cardiac death. These patients have CV risk factors including HTN, diabetes, low HDL, ventricular hypertrophy, CAD and old age.

What does vibrio vulnificus cause?

Cellulitis

Where do carcinoid tumors usually present in the lungs?

Centrally located and easily visible with a bronchoscope.

Titubation. what is this and what structure is involved?

Cerebellar dysfunction which causes a forward and backward movement of the trunk.

70 year old man with frequent falls. Past 3 months of headaches and nausea. Tends to balance on left, feels as if weak on that side. PMH includes DM and HTN. Sways to the left with eyes open. Decreased resistance to passive flexion. Diagnosis?

Cerebellar tumor on the left hemisphere causing obstructive CSF symptoms.

What is the mechanism for hypertensive encephalopathy?

Cerebral edema due to break through vasodilation from failure of autoregulation.

When is mannitol used? What needs to be monitored with it's use?

Cerebral edema with a patient with severely elevated intracranial pressures (often obtunded patients). Renal function and electrolytes.

50 year old with increasing neck swelling. Scant yellowish discharge on same side of face. Needle aspirate shows gram-positive, anaerobic branching bacteria. Treatment? Complications?

Cervical actinomycosis. Penicillin. Complications include: 1. abscess. 2. fistulas. 3. draining sinus tracts.

HSV in a woman who is in labor. treatment?

Cesarean.

Painless genital ulcer. Name the lesion. Diagnosis? (be specific)

Chancre. Primary syphilis

Ascites and rigid abdomen in an alcoholic patient with a history of peptic ulcer disease. What 2 diagnosis need to be teased out? What is the best test to distinguish the two?

Chemical peritonitis due to peptic ulcer perforation versus spontaneous bacterial peritonitis. Chest X-ray looking for pneumoperitoneum. *SBP will always be on the differential of ascites with pain.

Stocking-glove pattern, symmetrical paresthesia in a cancer patient. Likely diagnosis?

Chemotherapy-induced peripheral neuropathy from a vinca alkaloid, platinum-based medication and or taxanes.

Every time someone gets a central venous catheterization what needs to be done?

Chest x-ray to confirm placement.

Wilson disease presentation?

Child or adolescent with liver disease Young adult with neuropsychiatric symptoms like tremors, rigidity, depression, paranoia and catatonia with hepatomegaly or abnormal liver function tests.

What is the prognosis of those with postinfectious glomerulonephritis? State the general age groups too.

Children have good prognosis. Adults may possibly progress to chronic kidney disease.

30 year old patient brought to the ER. Excruciating left-sided chest pain. Temp of 99, Pressure of 160/90, pulse of 125 regular and respiration of 20. Anxious, agitated and diaphoretic. Dilated pupils and atrophic nasal mucosa. EKG shows ST depression and T wave inversion on leads V 4 through 6. No abnormality on chest x-ray. Likely diagnosis and next appropriate step and why? What other medications or procedures are appropriate and why?

Cocaine use. Best next appropriate step: Benzodiazepine for blood pressure and anxiety. Aspirin. Nitroglycerin and calcium channel blockers for pain. Cardiac Cath with repercussion when indicated.

Erythema nodosum and erythema multiform and pulmonary symptoms "Valley Fever", Diagnosis?

Coccidiomycosis

Name a commonly used long-acting benzodiazepine used for treating alcohol withdrawal.

Chlordiazepoxide.

Gallbladder wall thickening

Cholecystitis.

What complication do patients on TPN run a risk of developing while on this feeding protocol for at least 2 to 3 weeks? What liver pathology findings may be observed because of this?

Cholestasis. Fibrosis.

60 year old man with complaints of nausea and abdominal pain. Coronary angiogram and stent placed 5 days ago for chest pain. Painless, purple mottling of skin of both feet. Eosinophils at 12%. Diagnosis?

Cholesterol crystal emboli (atheroembolism)

How do you differentiate a patient with acute hypercarbia from a chronic CO2 retainer with COPD?

Chronic CO2 retainers have normal pH and high serum bicarbonate.

60 year old presents with intermittent arthralgias for the past 8 years. During those times he has transiently elevated hepatic transaminase levels. He had a mild episode of jaundice 2 years after a mitral valve replacement. Exam shows vesicles and erosions of dorm of both hands. Diagnosis?

Chronic hepatitis C infection because of intermittent elevations of transaminases and porphyria cutanea tarda.

How does one diagnose chronic hypertension with superimposed preeclampsia?

Chronic hypertension with 1 of the following: 1. new onset proteinuria or worsening proteinuria after 20 weeks. 2. sudden worsening of HTN. 3. signs of end organ damage.

Chronic dull epigastric and left upper quadrant pain, diarrhea, weight loss, and daily alcohol use. Pain not alleviated with antacids. Polydipsia and polyuria are his other complaints. Diagnosis and next best step?

Chronic pancreatitis. CT scan.

Cold vs warm agglutinin in terms of IgM and IgG?

Cold = IgM; Warm= IgG.

What is the goto fluoroquinolone for treatment of urinary tract infections?

Ciprofloxacin because it has goo oral absorption and is excreted into the urine.

What liver disease is associated with Dupuytren's contracture? How about clubbing?

Cirrhosis.

Treatment for leukemic reticuloendotheliosis (hairy cell leukemia)?

Cladibrine, a purine analog which is toxic to bone marrow. Neurological and kidney damage.

40 year old is on medication for paroxysmal atrial fibrillation. While stress testing his QRS complex duration increases from 90 milliseconds to 130 when his heart rate goes from 75 to 165. What medicine is he likely on? Why does this happen?

Class IC antiarrhythmic drugs such as flecainide and propafenone. "Use dependence" as the heart is pumping faster there is less time to dissociate from the sodium channels, leading to a higher number of blocked channels, leading to a progressive decrease in impulse conduction and widening QRS complex.

Postpartum endometritis, initial antibiotics?

Clindamycin and gentamicin.

Pt was treated for CAP with azithromycin for 5 days. Still has no improvement of her symptoms. 8 days ago she had an upper GI endoscopy. CXR shows RUL infiltrate. Next best step?

Clindamycin for anaerobic coverage.

What antibiotic treats cellulitis due to MRSA and can also be used as prophylaxis for infective endocarditis in high-risk patients undergoing invasive procedures.

Clindamycin.

What is the hepatojugular reflux used for?

Clinically differentiate between cardiac and liver disease related causes of lower-extremity edema. Those with peripheral edema from primary hepatic disease and cirrhosis have reduced or normal JVP and negative hepatojugular reflex. Heart failure causes will demonstrate a positive reflex.

Best medication for treatment resistant schizophrenia or those at high risk suicidality?

Clozapine.

Severe hypertension, rib notching on chest x-ray, brachial-femoral pulse delay and continuous murmur over the back.

Coarctation of the aorta.

Streptococcus gallolyticus, what disease association does this have? What is indicated in this patient's health management?

Colon cancer. Colonoscopy. (s. gallolyticus = bovis biotype 1, these are considered as Group D strep)

What is the presentation of myasthenia gravis?

Common in women between 18-3os or men older than 60 with intermittent dysarthria, dysphagia, ptosis and diplopia as well as generalized muscle weakness within a year onset. Pivot: resolution of muscular weakness with rest.

Why are survivors of Hodgkin lymphoma at an increased risk for cardiac disease?

Complications due to mediastinal irradiation and or anthracycline therapy (usually 10-20 years later)

Patient presents with hyponatremia, low serum osmolarity, hypervolemia. Name 3 causes.

Conditions which excess fluid is allocated elsewhere. 1. CHF. 2. Hepatic failure. 3. Nephrotic syndrome

When evaluating hyperbilirubinemia. What is the diagnostic approach to consider?

Conjugated or unconjugated? If conjugated, then is it mostly a Liver issue, Biliary issue or metabolic problem. Note metabolic problem as in Dubin-Johnson and Rotor's syndrome as ALT, AST, ALP are normal.

How do patients with sickle cell disease end up with folate deficiency?

Constant turnover of RBC uses folate.

Clostridium botulinum causes what symptoms?

Constipation and not vomiting.

45 year old immigrant from China presents with dyspnea, fatigue and abdominal distension for 2 months. Normal vital signs. Pedal edema, increased abdominal girth with free fluid and elevated JVP without inspiratory decline. Chest auscultation reveals decreased heart sounds and accentuated sounds after second heart sound in early diastole. Chest x-ray shows ring of calcification around heart. JVP tracings show prominent x and y descents. What is the likely cause of this patients symptoms? Likely reason for physical exam findings?

Constrictive pericarditis likely due to tuberculosis. Pericardial fibrosis and obliteration of the pericardial space.

Progressive peripheral edema, ascites, elevated JVP, pericardial knock (middiastolic sound) and pericardial calcifications on chest radiograph and a history of cardiac surgery. Diagnosis?

Constrictive pericarditis.

Progressive peripheral edema, ascites, elevated jugular venous pressure and pericardial knock. Diagnosis?

Constrictive pericarditis.

Sharp x and y descents on central venous tracing. Likely diagnosis?

Constrictive pericarditis.

Patient began RIPE therapy because of a Tb infection. AST and ALT are elevated in the 80s. Continue therapy and why?

Continue therapy. Subclinical hepatic injury due to isoniazid is expected in 10-20% of patients. Studies show condition is self-limited.

Afebrile man with irritative voiding symptoms.

Non-inflammatory chronic prostatitis.

What cardiac arrhythmias are contraindicated with adenosine? Which arrhythmias are terminated with this drug?

Contraindicated in second or third degree AV block. Used in termination of AV node-dependent reentrant tachycardia.

Hemorrhagic thalamus. Exam findings?

Contralateral hemiparesis and hemisensory loss. Upgaze palsy and eyes deviated towards hemiparesis. Nonreactive miotic pupils.

What neurological findings characterize an anterior cerebral artery stroke?

Contralateral motor and sensory deficits more pronounced in the lower limb than the upper limb with possible urinary incontinence.

Best diagnostic test to confirm aortic dissection? Is mediastinal widening on chest x-ray always present?

Contrast CT or TEE. No, do not rely on this finding.

What form of syncope can mimic a seizure?

Convulsive syncope. Prolonged syncope causing cerebral hypoperfusion, causing clonic jerks and urinary incontinence.

What defines heat stroke?

Core temperature above 40 C and altered mental status.

Any seizure that lasts more than 5 minutes puts a patient at risk for what?

Cortical laminar necrosis due to excitatory cytotoxicity.

The vagus nerve provides what part of the cough reflex? How about the gag reflex?

Cough: afferent limb. Gag: efferent limb.

Generally to get amebiasis what countries would someone come from and how long do they need to stay to get infected?

Countries like SE Asia, Africa and Central & South America. Patients usually have to stay a month in endemic areas to be exposed.

More than half of all cases of epigastric pain with no other symptoms such as weight loss, nausea and vomiting is due to what?

Non-ulcer dyspepsia (something that is not admitted to the hospital)

Fever, headache and papilledema in a HIV patient. Diagnosis and test? Treatment?

Cryptococcal meningoencephalitis. Antigen testing of CSF. Amphotericin B with flucytosine initially followed by maintenance fluconazole.

Dry cough and systemic symptoms that last for months. Chest x-ray shows bilateral ground glass infiltrates. Diagnosis?

Cryptogenic organizing pneumonia.

Diarrhea in an AIDS patient with acid-fast stained oocysts from stool sample. Diagnosis?

Cryptosporidium parvum.

What cause of metabolic alkalosis are associated with high urine chloride, hypovolemia/euvolemia and is saline responsive.

Current diuretic use.

Patient with years of bronchial asthma requiring multiple medications complains of weight gain despite no change in appetite, diet or activity. Exam shows supraclavicular fullness, cataracts, bruising and acne, and decreased proximal muscle weakness. Diagnosis and likely electrolyte finding?

Cushings syndrome. Hypokalemia (steroids work like aldosterone to an extent).

Children's sandbox and serpiginous, reddish brown, elevated lesions which are bilaterally on both upper extremities. Diagnosis? Pathogenesis?

Cutaneous larva migrans. Soil contaminated with dog and cat feces.

Pathophysiology of cyanide toxicity?

Cyanide binds to cytochrome oxidase and inhibits oxidative phosphorylation.

Pathogenesis of cyanide toxicity?

Cyanide is a potent inhibitor of cytochrome oxidase A3 in the electron transport chain. (It binds to ferric iron (well whammy know?) blocking ATP production. Hence it reduces oxygen utilization by tissues.

25 year old found unconscious in a house fire. Soot present in nares and mouth. Capillary refill time is 4 seconds. Heart rate of 125 and respiration of 25. Patient is on a non-rebreather mask. Blood gas shows pH of 7.15, PaCO2 of 33 and bicarbonate of 12. Lactic acid is 20. Patient should be empirically treated for what and why?

Cyanide poisoning because it is a potent and fast-acting poison that cannot be measured rapidly to confirm diagnosis prior to treatment.

Sodium thiosulfate is used for what?

Cyanide poisoning in the setting of fires or occupational hazards such as mining or pesticides.

Bitter almond breath?

Cyanide poisoning.

What cardiovascular diseases are associated with digital clubbing?

Cyanotic congenital heart disease.

Malaria presentation?

Cyclical fevers, systemic symptoms, anemia and thrombocytopenia.

Granulomatosis with polyangiitis. Treatment?

Cyclophosphamide and steroids.

Nephrotoxicity, hyperkalemia, hypertension, gum hypertrophy, hirsutism and tremor. All are adverse drug effects of?

Cyclosporine.

Adverse drug effect of clindamycin?

Diarrhea, nausea, vomiting, anorexia and metallic taste.

What is amitriptyline used for?

Depression, insomnia, and neuropathic pain.

Diffuse ST-segment elevations weeks after an acute myocardial infarction. Diagnosis?

Dressler's syndrome.

How do nitrates relieve angina?

Decrease left ventricular volume, which reduces wall stress and myocardial oxygen demand.

How do benzodiazepines treat cocaine related cardiovascular compromise?

Decrease the sympathomimetic actions of cocaine by reducing anxiety, and agitation, improving blood pressure and heart rate and alleviating cardiovascular symptoms.

If right heart strain progresses rapidly to right ventricular dysfunction what happens to the left heart?

Decreased return to the left side and decreased cardiac output. Left heart pump failure and bradycardia.

How does abnormal lung architecture cause decreased diffusion capacity of carbon monoxide?

Decreased surface area.

What finding maybe still present in patients with brain death?

Deep tendon reflexes (intact spinal cord).

What is the defect in chronic granulomatous disease?

Defect in NADPH oxidase which leads to ineffective intracellular killing.

Why are CT angiography useful with subarachnoid hemorrhages?

Detection of vasospasms which can help with the decision to initiate nimodipine.

What is a RigiScan?

Device that assess penile tumescence.

Craniofacial anomalies, hypocalcemia, thymic hypoplasia and congenital heart disease. Diagnosis?

DiGeorge syndrome.

Thymic aplasia.

DiGeorge syndrome.

On a chemistry panel what would be a dead give away when ruling out diabetes insipidus from psychogenic polydipsia? What features do these two disease share?

Diabetes insipidus would demonstrate hypernatremia while psychogenic polydipsia would demonstrate hyponatremia. These 2 disease share clinical features of euvolemia and polyuria.

Bipolar patient presents with complaints of confusion and needing to drink more water. Sodium is 155 and glucose is 95. Urine osmolarity is 250 and plasma osmolarity is 325. Diagnosis?

Diabetes insipidus, nephrogenic because of lithium.

Nodular glomerulosclerosis (Kimmelstiel-Wilson Nodules).

Diabetes membranous nephropathy.

Pathogenesis of fibromuscular dysplasia?

Noninflammatory and non-atherosclerotic condition due to abnormal cell development in the arterial wall that leads to vessel stenosis, aneurysm or dissection.

If IV fluids is not sufficient enough to fix hypotension in someone experiencing hypothermia what else can be done medically?

Dopamine.

Teenager presents to ER for sudden-onset redness and swelling of her skin over exposed areas. Acne history treated with an antibiotic. Exam shows erythema, edema and vesicles on her face, neck, dorsal hands and upper chest. Diagnosis?

Doxycycline-induced phototoxicity.

What antibiotic treats ehrlichiosis?

Doxycycline.

With regards to treating syphilis, if a patient is allergic to penicillin what medication does one prescribe? In what situation would one deviate from this medication?

Doxycycline. If treating tertiary syphilis administer ceftriaxone. If treating a pregnant woman, desensitize the patient and administer penicillin

HIV needlestick in healthcare worker. Protocol?

Draw blood now and start 3 drug antiretrovirals

Pleuritic chest pain and pericardial friction rub after weeks to months after an MI. Likely diagnosis? Treatment?

Dressler syndrome. NSAIDs.

For patients with a stroke history, advanced age, Parkinson's disease, dementia or sensory impairment what are the most likely causes of delirium?

Drugs Infections Electrolyte disturbances Metabolic derangements Systemic illness Central Nervous system

How does pneumocystis pneumonia present?

Dry cough and dyspnea. Chest x-ray demonstrates diffuse infiltrates; pleural effusion is not a common finding.

Serum sickness. Name the symptoms and pathophys.

Due to an immune complex reaction against heterologous proteins patients will experience the following: 1. fever. 2. urticaria. 3. arthritis. 4. nephritis. Bottom line: Serum sickness = immune complex = joints, kidneys with allergy-like symptoms (rash & fever)

How does one definitely diagnose infective endocarditis?

Duke Criteria (2 major or 1 major and 3 minor) Major - Positive blood culture for typical organisms (staph & strep) Echocardiogram for valvular lesions. Minor - Predisposing cardiac lesion IV drug use > 38 C Embolic phenomena Immunologic phenomena (glomerulonephritis) Positive blood culture for nontypical organisms

Bleeding in renal failure patients is most often due to what?

Dysfunctional platelets.

What is the purpose of cerebellar function testing (define this with a medical term!)

Dysmetria and rapidly alternating movements.

40 year old man with complaints of periodic involuntary head turning and fixation to the left side. Exam shows hypertrophied right sternocleidomastoid muscle. Diagnosis?

Dystonia (torticollis)

30 year old male with large mediastinal mass with associated elevations of AFP and B-hCG. Diagnosis?

Nonseminomatous germ cell tumor

HIV patient with difficulty swallowing is not responding to fluconazole treatment. Next best step?

Endoscopy to see lesions in esophagus to verify if candida or a viral etiology such as CMV or herpes.

ARDS pathophysiology?

Endothelial damage, cytokines and diffuse alveolar damage.

Eggshell calcification of a hepatic cyst, amebiasis or E. granulosus? Do you aspirate this?

E. Granulosus. Do not aspirate this, risk of anaphylaxis. E in E. granulosus for IgE mediated anaphylaxis.

Early manifestations of analgesic nephropathy? Later manifestations?

Early 1. polyuria. 2. sterile pyuria Later (papillary necrosis) 1. hematuria 2. renal colic Severe manifestations (chronic tubulointerstitial nephritis) 1. HTN. 2. Proteinuria. 3 impaired urinary concentration.

Recent upper respiratory tract infection, dyspnea, elevated JVP, clear lung fields and increased cardiac silhouette on x-ray. Diagnosis?

Early cardiac tamponade due to a large pericardial effusion.

Encapsulated and calcified cyst that contain fluid and budding cells is observed from liver pathology. Diagnosis?

Echinococcus

What is the etiology for aseptic meningitis usually? What months does this occur?

Echovirus. Summer months.

How do you differentiate myasthenic crisis from MG cholinergic crisis?

Edrophonium (Tensilon) test. This strengthens pts with crisis while worsens weakness in those with cholinergic crisis.

What iron study results would be typical of thalassemia and why?

Elevated iron, ferritin and transferring saturation. Total iron binding capacity would be low. This is due to the frequent transfusions associated with iron overload.

Why is there a persistent cough or hoarse voice with mitral stenosis?

Elevated left atrial and pulmonary vascular pressures causes left atrial enlargement which causes compression of the recurrent laryngeal nerve.

Air fluid levels or gas shadowing in the gallbladder

Emphysematous cholecystitis.

IV drug user who presents with infective endocarditis stigmata. Blood cultures drawn. Best next step?

Empiric treatment with vancomycin.

Fever, SOB and abnormal contour opacity (effusion) on CXR. Diagnosis?

Empyema.

What are the typical exam findings between viral upper respiratory syndrome versus streptococcal pharyngitis?

For viral upper respiratory syndrome, expect nasal edema with normal or slightly erythematous pharynx. With strep, expect pharyngeal edema, tonsillar hypertrophy & exudates and tender cervical lymph nodes.

Patient presents with shock and chest pain after 5 days from a treated left anterior descending occlusion MI. Exam shows jugular venous distention and distant heart sounds. Likely diagnosis? Treatment?

Free wall rupture of heart and cardiac tamponade resulting from hemopericardium. Pericardial effusion with tamponade.

Cirrhosis patient is bleeding at IV sites. Treatment and why?

Fresh frozen plasma because liver disease has led to decrease coagulation factors.

Bacillus cereus, presentation?

Fried rice, diarrhea, and cramps.

Paradoxical splitting, what conditions is this seen with?

Fixed left ventricular outflow tract obstruction such as the following: 1. aortic valve or sub-aortic stenosis. 2. left bundle branch block. 3. right ventricular paced rhythm.

Name 2 class IC antiarrhythmic drugs used for treating atrial fibrillation for maintenance of sinus rhythm. How doe these drugs work?

Flecainide and propafenone. These drugs block sodium channels and inhibit the initial depolarization phase of an action potential.

How do you diagnose paroxysmal nocturnal hemoglobinuria?

Flow cytometry for CD55 and CD59.

What happens to serum calcium levels when acidosis or alkalosis occurs?

Extra cations (hydrogen) will displace calcium off of albumin increasing serum calcium. While the opposite is true of alkalosis.

How does succinylcholine cause hyperkalemia?

Extracellular leakage of potassium through acetylcholine receptors.

How is isoniazid drug-induced hepatotoxicity different from typical drug induced hepatotoxicity?

Extrahepatic sensitivity manifestations like rash, arthralgias, fever, leukocytosis and eosinophilia are absent in isoniazid-induced hepatic cell injury.

Manifestations of Q-fever?

Flu-like syndrome, hepatitis and pneumonia.

Feigning physical or psychological symptoms to meet emotional needs. Obvious external rewards are absent. Diagnosis?

Factitious disorder

What is the most common inheritable cause of DVT. What is it's pathophysiology?

Factor V Leiden. Factor V is resistant to inactivation by protein C.

30 year old male admitted for severe epigastric pain and vomiting. Smoking history. Pressure of 140/90 and heart rate of 110. Prominent epigastric tenderness. Crops of yellow-red papule on arms and shoulders. Lipase is 3,500. ALT, AST, ALP and bilirubin is normal. Next best step and diagnosis being pursued? Name of physical exam finding?

Fasting lipid profile. This patient has symptoms of acute pancreatitis. Although pancreatitis is most commonly associated with gallstones and alcoholism other causes include hypertriglyceridemia. Eruptive Xanthomas are the crops of yellow-red papule described.

How does adrenal insufficiency present?

Fatigue, weakness, anorexia, GI disturbances and weight loss.

What medicine would be useful as prophylaxis for cryptococcus and coccndioides in an HIV patient who's had these disease previously?

Fluconazole.

25 year old undergoing chemo for metastatic cancer is in the ER for fever and chills for the past 24 hours. Exam is unremarkable. Leukocytes are 700 with 20% neutrophils. Urinalysis and x-ray is unremarkable. Clinical problem and best step in management.

Febrile neutropenia (this is a medical emergency). Piperacillin-tazobactam.

Define postpartum endometritis.

Fever after the 1st 24 hours postpartum. Exam is characterized by fever, uterine tenderness and foul-smelling lochia.

Presentation of Legionella pneumonia?

Fever over 39 C (over 102.2 F), bradycardia, neurologic and GI symptoms with no response to beta-lactams & aminoglycoside antibiotics. Recent history of travel to cruise or hotel within 2 weeks or exposure to contaminated potable water in hospitals or nursing homes.

Presentation of acute hemolytic transfusion reaction? Testing to confirm?

Fever, flank pain, hemoglobinuria, renal failure & DIC within 1 hour of transfusion. Positive direct Coombs test, pink plasma (ABO incompatibility)

Dengue fever, how does it present? Complication of this disease?

Fever, headache, retro-orbital pain, rash and myalgia and arthralgias. Complication may include hemorrhage in skin or nose.

How does TTP present?

Fever, microangiopathic hemolytic anemia, thrombocytopenia with possible purport, renal failure and neurological findings.

30 year old presents with occipital headaches that does not get better with acetaminophen. Blood pressure of 160/90. Right-sided renal bruit. Diagnosis and treatment? What would the likely diagnosis be in a 60 year old?

Fibromuscular dysplasia causing renovascular hypertension. Percutaneous angioplasty with stent placement. Surgery if stent placement fails. Atheromatous plaque in a 60 year old.

Drug option for patients who cannot tolerate oral vancomycin for C. diff?

Fidaxomicin.

For a solitary pulmonary nodule what must not be present in terms of imaging?

Findings of pleural effusion, adenopathy and atelectasis.

What medicines are used to treat CMV?

Ganciclovir and foscarnet

Where is the left sternal border? What heart murmurs can you hear there?

Generally at the third intercostal space. 1. HOCM. 2. aortic regurgitation. 3. pulmonic regurgitation.

For prosthetic joint infections how do delayed onset infections start and present

Greater than 3 month, symptoms present as persistent joint pain, implant loosening, and sinus tract formation

How does autosomal dominant polycystic kidney disease present?

Gross hematuria in a patient less than 60 years old with a history of recurrent flank pain, UTIs, pyelonephritis, and nephrolithiasis. Ultrasound shows kidney cysts.

Tubercle and associated lymphadenopathy is known as a what?

Ghon complex.

Patient is from the middle east and has icterus. It is currently Ramadan. Unconjugated bilirubin is elevated. Otherwise history and exam is normal. Diagnosis? Pathogenesis?

Gilbert's disease. Reduced numbers of UDP glucuronyl transferase.

Schizophrenia patient presents with a temperature of 101.6 F, pressure of 160/100, pulse of 120 and respirations of 25. Exam shows rigid muscles and increased tone. Creatine kinase is 50000. CSF fluid shows total white blood cell count of 5. Next best step and why?

Give dantrolene for neuroleptic malignant syndrome likely due to antipsychotics. Other options include bromocriptine (dopamine agonist) and amantadine (antiviral drug with dopaminergic properties).

40 year old woman with mitral regurgitation and palpitations. No other symptoms. Likely cause of mitral regurgitation?

Given her age, myxomatous degeneration of the mitral valve. If older, mitral annular calcification.

What is timolol eye drops used for?

Glaucoma

Necrotic migratory erythema

Glucagonoma

13 year old patient with recently diagnosed androgen insensitivity presents to office with cryptorchid gonads. Next best step and why?

Gonadectomy after puberty because of 1-5% risk of developing a dysgerminoma or gonadoblastoma after puberty. Estrogen therapy if there is insufficient hormone production (setting of bilateral gonadectomy)

25 year old with 2 week history of hemoptysis, shortness of breath, ankle edema and dark urine. creatinine of 3. Urinalysis shows dysmorphic red blood cells per high power field, mild proteinuria and red cell casts. Chest x-ray reveals bilateral alveolar infiltrates. Diagnosis? Treatment?

Goodpasture's syndrome. Emergency plasmapheresis.

Actinomyces and Nocardia are similar structurally how?

Gram-positive rods.

35 year old male with increasing SOB. 2 years of recurrent sinusitis. Temp of 101.5. Patchy rales on auscultation. CXR shows multiple nodular densities bilaterally. Cr of 2.5 and urinalysis shows RBC casts. Diagnosis? Test?

Granulomatosis with polyangiitis (Wegener's) c-ANCA and possibly biopsy at nasopharyngeal.

Why does good in the chest need to be evacuated?

Growth medium for bacteria.

What uremic toxin is associated with platelet dysfunction seen in chronic renal failure.

Guanidinosuccinic acid.

Cellulitis on the face of a child, etiology?

H. influenza.

What organisms that cause infective endocarditis are treated with ampicillin-sulbactam?

HACEK and penicillin-resistant enterococcus.

Porphyria cutanea tarda. What viral illness do you associate with this?

HCV

How does cryptococcal meningoencephalitis present?

HIV patient with headache, fever and malaise that develops in less than 2 weeks.

How is primary HIV infection different than mononucleosis?

HIV will more likely have a rash and diarrhea than mononucleosis, at the same time, tonsillar exudates are uncommon in primary HIV

What infectious causes of meningitis does ceftriaxone treat?

Haemophilus influenzae and Neisseria meningitidis.

What symptoms are associated with cerebellar hemorrhage?

Headache, facial weakness, gaze palsy or nystagmus, neck stiffness (blood in 4th ventricle), vomiting and gait ataxia.

What is pulseless electrical activity in the setting of cardiac tamponade?

Heart conduction is detected on ECG however due to the mechanical problem with tamponade patients will be pulseless.

All valvular heart disease are similar to what disease presentation?

Heart failure.

Symptomatic sinus bradycardia is defined how?

Heart rate less than 60 beats per minute and a constant PR interval

Altered mental status and muscle rigidity in a young patient outside on a hot summer day during football practice.

Heat stroke.

Iopanoic acid. What is it for and how does it work?

Helps control severe symptoms of thyrotoxicosis via decrease release of thyroid hormone by decreasing the amount of T4 converted into T3 (active form) in the body.

What is the value of an EEG with regards to seizures?

Helps stratify patient risk in the need of maintenance antiepileptic agents.

Patient has been in the hospital for acute renal failure secondary to PSGN. Patient develops retrosternal, non-radiating chest pain relieved by leaning forward. 2-component diastolic sound with squeaking quality heard at the left sternal border. EKG shows non-specific T wave abnormalities. Echo show trivial pericardial effusion and normal left and right ventricles. Urea nitrogen is 60 and creatinine is 4. Best next step?

Hemodialysis. Patient is experiencing uremic pericarditis.

At what hemoglobin concentration do patients with chronic renal failure receive erythropoietin?

Hemoglobin of less than 10.

Elderly patient who's on warfarin, indomethacin and aspirin with stroke like symptoms. What is the concern? Best treatment order to make?

Hemorrhagic stroke because all three of those medicines prevent clotting. Fresh frozen plasma and vitamin K.

Warfarin use, hypertension history and focal occipital headache is worrisome for what (be specific too)

Hemorrhagic stroke.

40 year old is admitted for acute alcoholic pancreatitis. Prior to admission he was noted to vomit small amounts of bright red blood a passing of black stool. 2 days during hospitalization, he is confused and disoriented. Exam demonstrates abdominal dissension with shifting dullness. When patient holds his arms in from of him with wrists extended, he repeatedly jerks his hands down and up. Diagnosis? Treatment? What if patient doesn't get better?

Hepatic encephalopathy. Administer lactulose. Rifaximin.

What virus is associated with polyarteritis nodosa.

Hepatitis B.

What are the liver pathology findings of HBV?

Hepatocellular injury, sinusoidal cell reactive changes, and inflammation and fibrosis of the portal tracts.

Cirrhosis patient due to HCV is admitted with belly pain and confusion. VS are unstable. He has all the cirrhosis stigmata on exam. Hyponatremia, hypokalemia and worsening kidney status (creatinine). Urinalysis is normal. Ultrasound shows post-void residual urinary volume less than 50. IV albumin and saline do not improve his renal output. Clinical problem and definition?

Hepatorenal syndrome - decreased glomerular filtration in the absence of shock, proteinuria, or other clear case of renal dysfunction and a failure to respond to 1.5 L bolus of normal saline.

Vesicle and round or avoid ulcers seen on endoscopy of HIV patient. Diagnosis and treatment?

Herpes simplex virus. Acyclovir.

Describe what is seen on MRI of a Glioblastoma multiforme.

Heterogenous and serpiginous contrast enhancement.

Define secondary hyperaldosteronism. What are the causes of secondary hyperaldosteronism? (name 4)

High aldosterone and high renin. 1. renovascular hypertension. 2. malignant hypertension. 3. renin-secreting tumor. 4. diuretic use. Bottom line: associate high renin levels with secondary hyperaldosteronism

Define primary aldosteronism and list the 2 causes of this illness.

High aldosterone, low renin. 1. Aldosterone producing tumor. 2. bilateral adrenal hyperplasia. Bottom line: with primary endocrine abnormalities, think tumor or hyperactive

80 year old patient presents with severe hypovolemia. Exam shows cold clammy skin wit poor skin turgor. Patient has a sodium level of 120. Cause of hyponatremia?

High antidiuretic hormone.

What iron study results would be typical of hemochromatosis?

High ferritin, transferrin saturation, and serum iron. Low total iron binding capacity.

What does HBeAg positivity indicate?

High level of viral replication even if patient has normal ALT.

Patient presents to you after a motor vehicle accident with gross deformity of the leg and is in severe pain. He has a history of substance abuse with narcotics. Most appropriate pain medicine?

IV morphine or similar. Doesn't matter about his past. Acute injury must be treated with IV medicines. Oral pain medications will always be wrong.

Hypotension, bradycardia and respiratory difficulty after the onset of an epidural injection for a woman in labor.

High spinal.

30 year old stab 6 months ago in the leg. Experiencing progressive weakness and exertional dyspnea since. Vital signs are afebrile, pressure of 160 over 60 and pulse of 100. Examination of the previously traumatized leg is warmer and more flushed. Soft systolic murmur is heard over the cardiac apex that does not change with Valsalva. Likely diagnosis?

High-output cardiac failure due to symptomatic arteriovenous fistula due to trauma.

Why is there decreased glucose concentration in cases of exudative pleural effusions?

Higher metabolic activities of leukocytes and possibly organisms in the fluid.

What clinical findings differentiate histoplasmosis from blastomycosis?

Histoplasmosis is associated with hilar adenopathy. Blastomycosis may present with osteolytic bone lesions or prostate involvement.

What fungal disease mimics sarcoidosis in terms of presentation?

Histoplasmosis.

30 year old with pain in right calf for the last 24 hours. Temp of 103F. Linear streaks of erythema noted on the calf with generalized swelling. Scaling is found in the toe webs. KOH shows hyphae. WBC of 14,000 with 6% bands. Best next step in management?

IV nafcillin or cefazolin. Note: chronic fungal infections can serve as a nidus for bacterial cellulitis.

Endocrine side effects of amiodarone include?

Hypothyroidism and hyperthyroidism.

Other than vitamin B12 deficiency what endocrine disorder can cause macrocytic anemia and polyneuropathy?

Hypothyroidism.

Under what conditions should antihypertensives be used in a patient with an ischemic stroke? What BP medication is recommended?

IFF BP >185/110, then either of these 2 settings: The use of thrombolytics or pt has a comorbid condition which requires lowering of BP such as heart failure, ACS or an aortic dissection. Labetalol or nicardipine

What is a stage III and IV ulcer?

III: full-thickness skin loss with possible visible subcutaneous fat. IV: full-thickness skin loss with exposed bone, tendon or muscle.

HIV with diagnosed candida infection after an endoscopy of the esophagus is not responding to fluconazole. Next best step?

IV Amphotericin

What medication should be given to a patient suspected of cocaine-associated chest pain and why?

IV BZDs because... 1. improve psychomotor agitation. 2. reduce myocardial O2 demand. 3. alleviate CV symptoms.

Patient presents with ventricular tachycardia and is stable. Treatment?

IV amiodarone, lidocaine or procainamide.

What well known HIV drug produces crystal-induced nephropathy? What class of drug does this belong to?

Indinavir. Protease inhibitor.

Why do patients with aortic regurgitation feel a pounding sensation on their chest? What position is the even more pronounced?

Increase LV size brings heart apex closer to chest wall. Left lateral decubitus.

What changes in the relationship between protein and albumin would you notice with someone with multiple myeloma

Increase protein levels and and an increase in the gap between them.

What coagulation study abnormalities is associated with a vitamin K deficiency?

Increase prothrombin time and aPTT to a lesser extent.

Why is increasing sodium intake a bad idea for those with renal stones?

Increase sodium increases calcium excretion.

How does lymphoma cause hypercalcemia?

Increased 1,25-dihydroxy vitamin D production.

Atrial septal defects cause murmurs in the pulmonic area and tricuspid area. Why?

Increased blood flow to both valves.

What is the most sensitive indicator for orthostatic hypotension from labs?

Increased blood urea nitrogen to creatinine ratio.

What dietary changes help reduce the risk of nephrolithiasis?

Increased fluids, reduce sodium and protein, normal calcium intake, increased citrate from fruits and vegetables and reduced-oxalate diet for oxalate stones (dark roughage, vitamin C).

Blind loop syndrome, what is this?

Increased gut absorption of lactic acid which causes hypochloremic metabolic acidosis.

How does positive pressure mechanical ventilation increase the risk for acute circulatory failure in a patient who is hemorrhage internally?

Increased intrathoracic pressure, increasing right atrial pressure and decreasing systemic venous return.

A hemoglobin A1C control of less than 6% is associated with what?

Increased mortality.

Why is restriction of dietary calcium a bad idea for those with recurrent nephrolithiasis?

Increased oxalate absorption from the gut.

What is the mechanism that explains why a patient lying on his left side where his pneumonia is would cause a decrease in oxygen saturation versus lying on his right side?

Increased physiologic shunting.

25 year old at 28 weeks gestation is concerned because fetal movements have not happened for 2 weeks. Fetal heart tones on doppler and cardiac activity on ultrasound confirms fetal demise. Fibrinogen and platelets at low normal and normal Prothrombin and partial thromboplastin time. There is no signs of active bleeding. Next best step and why?

Induction of labor. Fibrinogen and Platelets are low normal and may indicate an ongoing DIC, so induce labor and definitely do not cut the patient with procedures like a C-section. Also, options such as platelet transfusion, fibrinogen replacement and transfusion of fresh frozen plasma are unnecessary as patient is not bleeding.

Coxiella what is the predisposing factor for this infection? What disease does it cause?

Infected livestock or consumption of unpasteurized milk. Q fever.

Patient is admitted for worsening confusion, high fever and hypotension. 10 days ago he was had an episode of pancreatitis that was treated. Likely diagnosis?

Infected pancreatic necrosis.

40 year old HIV patient complains of worsening pain with swallowing over the past few weeks. Not taking treatments and CD4 of less than 100. Likely diagnosis? Best next step?

Infectious esophagitis, candida. 1. oral fluconazole.

Atypical lymphocytosis, transient hepatitis. and flu-like syndrome. Diagnosis and Test?

Infectious mononucleosis. Heterophile antibody (Monospot)

Septic emboli on CT of lungs in an IV drug user. Presentation of fever, cough and chest pain. No murmur on exam. Diagnosis?

Infective endocarditis, likely staph aureus. Note: due to low pressure gradient a murmur may not be heard.

40 year old man with 2 weeks of weakness, low-grade fevers, and exertional shortness of breath. Pain at fingertips and dark and cloudy urine. Proximal and distal interphalangeal joints are swollen. Diagnosis?

Infective endocarditis.

Fever, chills, myalgia, cough and coryza during the winter that developed suddenly over a period of 2 days. X-rays may show no findings or interstitial or alveolar patterns.

Influenza

Trichinellosis, what are its presentation?

Ingestion of under cooked meats, endemic in Mexico, Argentina, China, Thailand and central Europe. Gastric symptoms within 1 week of ingestion. Myositis, fever, subungual splinter hemorrhages and periorbital edema 4 weeks after.

Lesions of the frontal lobe cause with eye findings.

Ipsilateral deviation of the eyes.

Pathogenesis of hemochromatosis on the heart? Heart complications?

Iron deposition in cardiac tissue. 1. dilated cardiomyopathy. 2. heart failure. 3. conduction abnormalities.

How does cyclosporine work? Name the 8 side effects associated with this drug.

Inhibits transcription of IL-2. 1. Nephrotoxicity - electrolyte imbalances and possibly HUS. 2. HTN. 3. Neurotoxicity - vision changes, seizures, tremors. 4. Glucose intolerance. 5. Infection. 6. Malignancy - squamous cell of skin and lymphoproliferative diseases. 7. gingival hypertrophy & hirsutism. 8. Gi manifestations: n/v/d

Cyclosporine, how does it work?

Inhibits transcription of interleukin-2, to target T-helper lymphocytes.

Treatment for cryptococcal meningoencephalitis?

Initially administer amphotericin B with flu cytosine, then maintain the patient o fluconazole.

Patient has a defect in leukocyte adhesion, what is the defective molecule?

Integrin B2.

Men with atypical angina are at what pretest probability of coronary artery disease?

Intermediate (20-80%) at all ages.

What is an effective way to reduce UTIs in a patient with neurogenic bladder?

Intermittent catheterization.

Schatzki ring. Patient history? Treatment?

Intermittent dysphagia. Pneumatic dilation.

progressive dyspnea, persistent cough, atrial arrhythmias causing palpitations and x-ray showing increased reticular or nodular interstitial markings without left main stem bronchus displacement. Diagnosis?

Interstitial lung disease.

What is the risk of giving cation exchange resins?

Intestinal necrosis.

What are the complications of hypertensive encephalopathy?

Intracranial and subarachnoid hemorrhage.

What laboratory values help distinguish intravascular hemolysis from extravascular hemolysis?

Intravascular is associated with the following... 1. markedly reduced serum haptoglobin (almost undetectable). 2. Elevated indirect bilirubin levels. 3. Raised serum lactate dehydrogenase.

Patient with myasthenia gravis has declining vital capacity, maximal inspiratory force and oxygen saturation? Next best step?

Intubate for airway protection and withhold AChE inhibitors to avoid excess excretion.

70 year old presents to ER found unresponsive. History of PNA and GI bleed 2 months ago. In ER today, BP of 70/40, HR of 120 and RR of 32. AST and ALT are in the 3000 range and ALP is 150. Diagnosis?

Ischemic hepatic injury.

75 year old man with a pressure of 165 over 75. What is the clinical term for this?

Isolated systolic hypertension likely due to decreased elasticity of arterial wall. Notice that is just the systolic blood pressure that is elevated, this would rule out hypertension associated with RAAS.

HIV patient with PPD test of 6mm, management and explain.

Isoniazid and pyridoxine. Patient is at risk for active Tb. The pyridoxine is there to prevent possible neuropathy from pyridoxine, but it will not help with isoniazid-induced hepatitis.

What is the pathogenesis for paroxysmal nocturnal hemoglobinuria?

Issues with CD55 and CD59 protein.

What does adenosine do?

It acts as a very short acting AV-nodal blocker.

What does herpes simplex virus cause in the brain and explain the symptoms.

It causes encephalitis, leading to cognitive and personality changes as well as focal neurological deficits and or seizures. Usually the temporal lobe is involved. If these symptoms are not present in the workup of a patient with fever and headache or other nonspecific neurological findings, put this lower on the differential.

True or false, papilledema is a contraindication to lumbar puncture.

It depends on the clinical setting! It is not a contraindication iff there is an absence of obstructive/noncommunicating hydrocephalus or mass lesion on imaging.

What is tacrolimus?

It is a macrolide produced by fungi that has the same actions as cyclosporine.

What is aminophylline and what applications does it have?

It is a methylxanthine which blocks phosphodiesterase, increasing cyclic AMP. It causes bronchodilation and has positive inotropic/chronotropic effects.

When working up metabolic alkalosis in patients with high urine chloride, what is the purpose of evaluating volume status?

It is a quick way to rule in diseases of excess mineral corticoid activity such as primary hyperaldosteronism, Cushing's disease, black licorice, and ectopic ACTH production.

How does mycophenolate work? What is a major adverse drug effect of this?

It is a reversible inhibitor of inosine monophosphate dehydrogenase which inhibits de novo purine synthesis. Major toxicity is bone marrow suppression. Note: Think M for marrow suppression.

Why is negative pressure wound therapy never the right answer with an infected or necrotic process?

It is a treatment plan that helps accelerate a healing wound. Active inflammatory processes are not cured by this method and will not address underlying disease processes.

What is fluphenazine. What is a complication unique to this medication within it's own class?

It is a typical antipsychotic. Hypothermia, hence patients are advised to avoid prolonged exposure to cold temperatures.

What is adenosine and how does it work?

It is an L-type calcium channel inhibitor that decreases conduction velocity in the AV node.

How is type 2 membranoproliferative glomerulonephritis unique among glomerulopathies?

It is caused by IgG antibodies called "C3 nephritic factor" that is directed against C3 converts of the alternative complement pathway.

Is a positive toxoplasma serology helpful?

It is common for people to have this.

Where does the spinal cord end in adults?

L1 and L2.

What do parapneumonic effusions (complicated nor uncomplicated) and empyema all have in common?

LDH ratio > 0.6

How is Lambert-Eaton myasthenic syndrome different from myasthenia gravis in terms of presentation?

LEMS: 40 years old and muscle strength with use. MG: 18-25 year old women with strength with rest.

Locate the lesion: mild dysarthria with unilateral motor deficit of face, arms and legs. No sensory, visual or higher cortical dysfunction.

Lacunar infarct in the posterior limb of the internal capsule.

55 year old man comes to ED with severe dizziness, inability to walk, and stabbing pain on the right side of his face that started in the AM. Pt is at risk for atherosclerosis. Pt topples to the right when sitting without support. Left pupil is larger than the right. Reduced corneal reflex on the right directly but not consensually. Partial ptosis of the right eye. Horizontal and rotational nystagmus present. Gag reflex is diminished. Loss of pain and temperature sensation in the right face and the left trunk and limbs. Location of the lesion?

Lateral medulla

Locate the lesion! Loss of pain and temperature in the ipsilateral face and contralateral trunk and limbs ~PLUS~ vestibulocerebellar findings such as nystagmus (horizontal or rotational), vertigo with falling to one side (side of lesion) and difficulty sitting upright without support.

Lateral medullary infarction due to occlusion of the intracranial vertebral artery.

Complications of chronic mitral regurgitation?

Left atrial and ventricular enlargement. Left ventricular dysfunction. CHF. Atrial fibrillation

What chamber of the heart should you associate with inferior myocardial infarction?

Left atrium

Pulseless electrical activity after a recent first myocardial infarction without signs of heart failure? Likely diagnosis? Workup?

Left ventricular free wall rupture. Confirm with echocardiogram.

Round or oval macule with even pigmentation.

Lentigo simplex.

With a patient who just received fibrinolytic therapy for a stroke, what is the goal for blood pressure control in these patients? Best medicine for this purpose?

Less than 185/110 in 24 hours after administration. Labetalol (alpha-1 and beta adrenergic blocker)

For prosthetic joint infection, what is considered "early-onset infection?"

Less than 3 months since arthroplasty. Presentation is wound drainage, erythema and fever.

What is the D-dimer cutoff used in excluding a pulmonary embolism?

Less than 500 ng/mL

Ideally when should percutaneous coronary angiography be done?

Less than 90 minutes.

Decompensated cirrhosis and ascites patient is admitted to the hospital. He is given loop diuretics. How does this change his ABGs and CMP profile?

Loop diuretics inhibit the Na-K-2Cl carrier in loop of Henle. By doing so there is a loss of those ions. In addition, volume contraction from this diuretic stimulates aldosterone secretion. The effects of all this lead to a net loss of potassium and chloride, improvement in BUN and Cr, and increase in pH.

Patient with acute kidney injury in setting of heart failure, best treatment strategy?

Loop diuretics to get the heart pumping to the renal arteries.

Mechanisms of metabolic acidosis due to diarrhea?

Loss of HCO3 in feces.

What is the pathological basis of thrombotic thrombocytopenic purpura?

Low ADAMTS13 activity

How does scleroderma renal crisis present? Pathogenesis?

Low H&H and platelets. Increased total bilirubin and creatinine. Increased permeability, activation of coagulation cascade and increased renin secretion.

What complications are also associated cirrhosis that do not involve portal hypertension.

Low protein synthesis which leads to the following: 1. hypogonadism. 2. decrease T3 and T4 via decreased production of thyroxine-binding globulin. 3. hyperlipidemia due to decreased lipoprotein.

In the workup of hyponatremia what is considered low serum osmolarity? How about high?

Low serum osmolality would be less than 280 while high would be greater than 295.

How does a large pericardial effusion present on ECG?

Low voltage of QRS complexes on ECG.

Elderly patient with severe agitation in the setting of dementia and delirium, what is the best medicine to prescribe such a patient?

Low-dose haloperidol. Other options: quetiapine and risperidone.

What do fasciculations indicate? Lower or upper motor neuron lesions.

Lower motor neuron damage.

Fever, confusion, seizure and focal deficits like positive Babinski. CT is unremarkable. Next best step? Likely diagnosis?

Lumbar puncture then PCR. Herpes encephalitis.

If you are suspicious of a subarachnoid hemorrhage but CT is negative what would test can you pursue?

Lumbar puncture.

Thick-walled cavitary lesions on chest x-ray in the setting of cough, hemoptysis, fever and purulent sputum.

Lung abscess.

In terms of fever and spread of rash how is measles different from rubella?

Measles is associated with high fevers (over 104) and the rash spreads over a period of days not hours within a day.

Workup of hypercalcemia shows PTH levels are suppressed. Next best step followed by the next.

Measure PTHrP and both 25 and 1, 25 vitamin D.

What do the guidelines include when evaluating a young patient with systemic hypertension?

Measure brachial and femoral pulses to assess for brachial-femoral delay and blood pressure measurements.

70 year old woman admitted for increasing shortness of breath, weight gain and lower extremity edema. History of hypertension, hyperlipidemia, non-ischemic cardiomyopathy and ejection fraction of less than 30%. She is on appropriate medicines including digoxin. Started on IV furosemide for the past 3 days. On telemetry she has had 6 beats of wide complex tachycardia. Nothing has been ordered yet. Best next step?

Measure serum electrolytes and digoxin Hypokalemia and hypomagnesemia can lead to ventricular tachycardia. Note: hypokalemia potentiates the side effects of digoxin.

Contralateral paralysis of the arm and leg to the right and deviation of the tongue towards the left. Loss of tactile and position sense on the right. Where is the lesion?

Medial medullary syndrome

Contralateral ataxia and hemiparesis of the face, trunk and limbs with loss of contralateral tactile and position sense. Where is the infarct?

Medial mid-pontine (infarction)

What is the pathophysiologic mechanism that cause clubbing?

Megakaryocytes trapped in the distal fingertips as the have skipped the normal route of fragmentation within the pulmonary circulation and have entered the systemic circulation. Note: VEGF and PGDF promote growth at the site of the fingers.

Hepatitis C infection causes what kind of renal disease?

Membranoproliferative glomerulonephritis

Of all the nephritic syndromes, which etiology is the most commonly associated with renal vein thrombosis?

Membranous glomerulopathy.

What cause of nephrotic syndrome is most associated with renal vein thrombosis?

Membranous glomerulopathy.

30 year old woman with 2 months of edema and abdominal distention. Ascites and lower extremity edema. Proteinuria of 4 grams per day. Albumin of 2.5. Normal ultrasound. Patient is placed on diuretics and salt and protein restriction which improves edema. Suddenly patient develops right-sided abdominal pain, fever, and gross hematuria. What is the likely diagnosis? What is the clinical problem?

Membranous glomerulopathy. Complicated renal vein thrombosis because of loss of antithrombin III.

How does emphysematous cholecystitis present? Complications?

Men of the age 50 to 70 who present with RUQ abdominal pain, fever, nausea, and vomiting. Exam may demonstrate crepitus in the abdominal wall adacent to the gallbladder, but peritoneal signs are absent. 1. gangrene. 2. perforation.

Purpuric skin lesions, fever, myalgia in a young patient with meninigismus. Likely diagnosis?

Meningococcemia.

35 year old with Hodgkin lymphoma diagnosis comes to physician with fatigue and generalized edema. Chemistry is normal for sodium, potassium, creatinine and total bilirubin except albumin is 2 and globulin is 5. Urinalysis shows 4+ protein with no hematuria. Diagnosis?

Minimal change disease. This condition is associated with minimal change disease.

What are the equations for appropriate compensation of metabolic acidosis and alkalosis?

Metabolic acidosis - arterial PaCO2 = 1.5 times serum HCO3 + 8 +/-2 Metabolic alkalosis - arterial PaCO2 increases by 0.7 mm Hg fore every 1 mEq/L rise in serum HCO3.

30 year old woman presents with weakness, tingling, and numbness of extremities. Normal vitals. Sodium of 135, potassium of 3, chloride of 85, bicarbonate 35 and pH of 7.5. Urine chloride and sodium are normal. Diagnosis?

Metabolic alkalosis due to self-induced vomiting.

Visual blurring, central scotomata, afferent pupillary defect and altered mentation in the setting of substance overdose. Diagnosis?

Methanol ingestion 1. High osmolar gap 2. increased anion gap metabolic acidosis

Homeless man presents with headache, nausea, vomiting and epigastric pain. Noted to have vision loss and decreased awareness. Hyperemic optic discs. Anion-gap acidosis present. Diagnosis and treatment?

Methanol ingestion and fomepizole.

Of the three inhaled toxins which causes patients to present with cyanosis and bluish discoloration of the skin and mucous membranes.

Methemoglobinemia.

"Snowfield vision"

Methyl alcohol poisoning

Decreased appetite, nausea, abdominal bloating and early satiety for the past months in a 65 year old with uncontrolled diabetes. What would best help treat his condition and why?

Metoclopramide because it has antiemetic effects as well as prokinetic ones.

What is the first-line medication for controlling symptoms and improving exercise tolerance in patients with stable angina pectoris?

Metoprolol or a similar beta blocker.

60 year old with complaints of nocturnal urinary frequency, occasional dribbling and a weak urinary stream for past 3 months. Diagnosed with diabetes 15 years ago and has recently controlled sugars.Blood pressure is 160 over 100. Post-void bladder residual volume is 50. Dipstick shows 3+ protein and no blood. Creatinine is 2. What is likely the cause of his chronic kidney disease?

Microangiopathy due to diabetes. Ignore the BPH symptoms.

Delayed hemolytic transfusion reaction. Presentation? Pathophys? Diagnosis?

Mild fever and hemolytic anemia. Symptoms appear 2-10 days after transfusion because of an anamnestic antibody response. Positive direct Coombs test, positive new antibody screen.

Delayed hemolytic anemia, how does it present? Mechanism of disease?

Mild fever and signs of hemolysis 2 to 10 days after transfusion. Positive direct Coombs test. Due to anamnestic antibody response.

What FEV1 or PEF value correlates to mild-to-moderate asthma exacerbation? How about Moderate-to-severe?

Mild-to-moderate: >40% Moderate-to-severe: <40%

With regards to types of tumor associations, what associations are studied with minimal change disease versus membranous glomerulopathy. Is there an easy way to remember this?

Minimal change disease is associated with lymphoma. Membranous glomerulopathy is associated with solid cancers like lung, colon, prostate and breast cancers (basically the most common cancers of all time...)

Patient with lymphoma has proteinuria of 4+. What is the etiology of his nephrotic syndrome?

Minimal change disease.

What valve dysfunctions cause chronic left ventricular volume overload? How about non-valvular dysfunctions?

Mitral and aortic regurgitation. Impaired myocardial wall contractility.

Exertional dyspnea, dry cough and holosystolic murmur best heard on apex. Likely diagnosis?

Mitral regurgitation.

20 year old woman complaints of intermittent chest pain for past 3 weeks. Sharp pain located to the left of the sternum and lasting 5-10 seconds. No associated symptoms. Systolic murmur murmur at apex that shortens with squatting. Likely diagnosis?

Mitral valve prolapse.

What type of valvular disorder has the highest association with infectious endocarditis?

Mitral valve prolapse.

Aspiring toxicity is what kind of acid base disturbance and what is it's pathogenesis?

Mixed. Aspirin stimulates the medullary respiratory center to cause tachypnea and respiratory alkalosis. Aspirin itself causes anion gap metabolic acidosis due to increased product and decreased elimination of organic acids (lactic, ketoacids).

Which Mobitz AV block is associated with increasing PR intervals?

Mobitz type I.

What lactate levels is cyanide poisoning associated with?

More than 10

For the regular patient with renal stones what advice should be given with regards to treatment? Should the patient limit calcium intake and why?

Most common renal stones are calcium stones, avoid dietary protein and oxalate. No, limiting calcium has been proven to increase stone formation. Limit vitamin C intake if patient forms oxalate stones.

Inappropriately normal PTH or elevated PTH with hypercalcemia. Diagnosis? Why is this not chronic kidney disease?

Most likely, primary hyperparathyroidism. Calcium would be low in chronic kidney disease.

What is mitral valve prolapse syndrome?

Nonspecific symptoms associated with mitral valve prolapse. 1. atypical chest pain. 2. dyspnea. 3. palpitations. 4. dizziness. 5. anxiety. 6. panic disorder. 7. nonspecific electrocardiographic changes.

70 year old with 3 month history worsening fatigue and back pain. Diabetes and hypertension for 10 years. Hemoglobin of 9, calcium of 11, phosphorus of 3.5, urea nitrogen of 40 and creatinine of 2. Diagnosis? Pathogenesis?

Multiple myeloma. Clonal proliferation of plasma cell resulting in excessive production of single immunoglobulin type (monoclonal paraproteins).

35 year old white woman with bilateral, lightning-like pain on her face. Prior to this she had left arm weakness. Neurologic exam shows no focal deficits. Likely diagnosis?

Multiple sclerosis.

Erythema multiforme and pneumonia in a young patient. Diagnosis?

Mycoplasma.

Mnemonic for Papillary necrosis?

NSAID Non-steroidal anti-inflammatories Sickle cell Analgesic abuse Infection (pyelonephritis) Diabetes.

What drug is a common cause of SIADH?

NSAIDs potentiate the effects of ADH. Watch out for this rheumatology patients.

What drugs tend to precipitate delirium?

Narcotics. Sedatives. Antihistamines. muscle relaxers.

How does one confirm an influenza diagnosis? What is the treatment and parameters for it?

Nasal swab for influenza antigens. Treat with oseltamivir and zanagmivir (neuraminidase inhibitors) within 48 hours of symptom onset.

Damaged to what nerve will cause cornel anesthesia?

Nasociliary nerve.

What is habituation in regards to psychiatry?

Natural decrease in anxiety that occurs when patients remain exposed to anxious thoughts without engaging in a compulsion response.

Dishwater drainage in a post-op wound.

Necrotizing surgical site infection

Why is it in severe asthma or COPD that the drop in intrathoracic pressure can be exaggerated up to 40 mm Hg?

Negative pressure from these disease causes pooling of blood in the pulmonary vasculature. Also, marked expansion of the lungs in these disease can impinge upon the outward expansion of the heart.

Is ceftriaxone associated with cholestasis. What patient age groups is this contraindicated in?

Neonates at high risk of developing cholestasis (risk of kernicterus).

30 year old admitted to the hospital for hallucinations and discharged in stable condition. Temperature of 101, pressure of 150 over 100, pulse of 110 and respirations of 25. Profuse diaphoresis. Abdominal muscles are rigid. Muscle tone is increased. Lead pipe rigidity.

Neuroleptic malignant syndrome, "drug induced idiosyncratic reaction"

What is the presentation of conversion disorder (functional neurologic symptom disorder)

Neurologic symptom incompatible with any known neurologic disease; often acute onset associated with stress.

What are the clinical manifestations of cyanide poisoning?

Neurologic, respiratory and cardiovascular depression. 1. coma, seizure. 2. bradycardia, hypotension. 3. cardiorespiratory arrest.

What consult must be put into place when intracranial hemorrhage is seen on CT?

Neurosurgery.

Dermatitis, diarrhea, dementia. Which vitamin is deficient?

Niacin (B3)

Aphthous ulcer seen on endoscopy of HIV patient. Diagnosis and treatment?

No diagnosis, symptomatic treatment

Would you treat with streptokinase or urokinase use in a patient with empyema in the setting of a motor vehicle trauma?

No it is contraindicated.

Does progressive multifocal leukoencephalopathy lesions present as enhancing lesions or mass effects on MRI?

No they are non-enhancing and have no mass effect.

Are patients with acute limb ischemia treated with IV infusion of fibrinolytic agents?

No they are treated with direct intra-arterial administration of fibrinolytic agents on the clot using an angiographic catheter.

Is giving benzodiazepines to the elderly for agitation a great idea?

No! Give BZD to the young.

Should doctors rely on transaminases to correlate with disease activity with HCV? What is the best predictor of HCV progression?

No! transaminases have a poor correlation. Best predictor of disease progression is liver biopsy.

Do patients with hyperaldosteronism have peripheral edema?

No, hence why its called aldosterone escape.

Do acetylcholinesterase inhibitors induce remission of myasthenia gravis?

No, they just reduce symptoms.

Do antipyretic medications have a role in management of exertional heat stroke?

No.

Is there an association with hypoalbuminemia and pulmonary edema?

No.

Is mixed cryoglobulinemia likely to present with arthralgia alone as the main complaint? If not then what is expected?

No. From the history, one would expect, chronic vasculitic syndrome characterized by palpable purpura, lymphadenopathy, nephropathy and neuropathy.

Mega doses of vitamin C in someone with renal calculi. Is this recommended? why or why not?

No. Increased formation of oxalate stones.

Is right ventricular preload affected by reduced effective forward flow of the heart and cardiac output in patients with acute MI and MR?

No. right ventricular preload is determined by systemic venous return.

What is an aerobic, Gram-positive and partially acid-fast filamentous rod that affects the brain, soft tissue, skin and lungs of immunocompromised patients?

Nocardia.

Immunocompromised patient with brain abscess, lung nodules and and fever. Cultures show gram positive branching rods. Diagnosis and treatment?

Nocardia. TMP-SMX.

What is the pathophysiology for patients who experience distal ischemia such as necrotic digits, mesenteric ischemia or renal failure?

Norepinephrine-induced vasospasm.

What BMP findings are typical of renal tubular acidosis?

Normal anion gap metabolic acidosis. Bicarb below 20.

Antepartum bleeding. What are the 4 things on the differential? Generally who do you distinguish them?

Normal labor, placental abruption, placenta previa, and uterine rupture. Any ongoing hemorrhage or passing of clots rules out normal labor. Presence of pain rules out placenta previa. Uterine contractions rules out uterine rupture.

Dementia, gait disturbance and incontinence, diagnosis?

Normal pressure hydrocephalus.

70 year old complains of urinary incontinence for the past 6 months. Denies dysuria, nocturia and penile discharge. gait is wide-based, slow and shuffling. MMSE is 24/30. Diagnosis? and pathophysiology?

Normal pressure hydrocephalus. Increased ventricular size without persistent elevations in ICP, leading to distortions of the periventricular brain matter.

Cirrhotic patient due to alcoholism and HCV is admitted for severe vomiting. Vitals are normal. ABGs show pH of 7.5, PaCO2 of 41 and HCO3 of 30. Sodium of 138, potassium of 3 and chloride of 95. Treatment and why?

Normal saline and potassium because the severe intravascular volume loss needs to be addressed to terminate the maintenance phase (kidney produces aldosterone when under perfused).

60 year old returns to office 10 days after an MI. Multiple premature ventricular complexes are seen on ECG. Treatment?

Observation.

Scooped out pattern on flow-volume loop.

Obstructie lung diseases.

What is the main mechanism for chylothorax?

Obstruction of the thoracic duct or its tributaries.

Schatzki ring, pathophysiology and associations?

Occurs because of acid reflex. Associated with hiatal hernia.

What are Hollenhorst plaques?

Ocular involvement from cholesterol crystal emboli.

Of the following which are antibiotic options to treat pyelonephritis if an organism is susceptible to it? 1. gentamicin. 2. ceftriaxone. 3. nitrofurantoin.

Of the 3, gentamicin and ceftriaxone.

What is the best next step for newly diagnosed HCV in a patient.

Offer a liver biopsy!

What is HER2 and what test will determine its expression on a breast cancer tissue sample?

Oncogene. FISH or IHC staining.

What is the first branch of the internal carotid artery?

Ophthalmic artery.

HIV patient presents with dysphagia and substernal burning for a week. CD4 is 90. Mucosa shows white plaques that are easily removable. Vitals are stable. Other aspects of physical exam are unremarkable. next best step?

Oral fluconazole. Iff there is a failure to respond to empiric therapy or a HIV patient does not have thrush would warrant an endoscopy. If it was strep pneumoniae, it would have been continued until day 4 of treatment.

What are the treatment options for blastomycosis?

Oral itraconazole for mild to moderate disease. IV amphotericin B for severe disease.

Pt suspected to have C. diff has the following labs: WBC is 17,000, a creatinine of 1.9 (previous Cr is 1), and serum albumin of less than 2.5. Management? How about if patient experiences ileus?

Oral vancomycin. If presenting with ileus too, add metronidazole and switch to rectal vancomycin instead of oral.

Infertility in a 30 year old body builder with small testes. Test and diagnosis?

Order LH. Exogenous steroid use.

Farmer attempts suicide and is found soiled with his own vomitus. Pulse is 50 and respirations are 25. Oxygen saturation is 85. Watering of the eyes and 1 mm pupils, bilaterally, widespread rhonchi with prolonged expiration, increased bowel sounds and muscle fasciculations. Diagnosis and best first step in treatment

Organophosphate poisoning ("think fast and wet?") 1. Administer atropine. 2. remove patient's clothing to prevent continued absorption of organophosphate through skin.

Why is there low phosphorus in patients who present with HHS or DKA?

Osmotic diuresis.

Disadvantages of renal transplantation over dialysis?

Other than finding a donor and surgical risk and cost, it is immunosuppression.

Ramsay Hunt Syndrome, what is it?

Otherwise known as Herpes zoster oticus, VZV causes a triad of ipsilateral facial paralysis, ear pain and vesicles in the auditory canal and external auricle.

For pyelonephritis what drug regimen is prescribed outpatient? Inpatient?

Outpatient: fluoroquinolones such as ciprofloxacin or levofloxacin. Inpatient: IV fluoroquinolone, aminoglycoside plus or minus ampicillin.

What levels of triglycerides are associated with acute pancreatitis?

Over 1000!

Usual etiology of malignant otitis externa?

P. aeruginosa.

What is the Pneumococcal vaccine guidelines for an HIV patient?

PCV13 once followed by PPSV23 8 weeks later and every 5 years.

Which pneumovaccine is recommended in patients with chronic liver disease?

PPSV23 then revaccinate at age 65 with PCV13 and PPSV23

When would steroid use be indicated in an HIV being treated for pneumocystis pneumoniae?

PaO2 of less than 70 or A-a gradient greater than 35.

Pathology findings associated with obstructive nephropathy?

Parenchymal atrophy due to calyceal dilation.

40 year old woman presents with abdominal pain and dark urine. Right upper quadrant tenderness. Hemoglobin 9, platelets 125,000, total bilirubin 6, LDH of 8000, and haptoglobin of 25. Hepatic vein thrombosis. Diagnosis?

Paroxysmal nocturnal hemoglobinuria.

hemolytic anemia, cytopenias and hypercoagulable state. diagnosis? Treatment?

Paroxysmal nocturnal hemoglobinuria. Iron & folate supplementation. Eculizumab (inhibitor of complement activation).

Patient presents with palpitations with a pulse rate of 180 and no evidence of structural heart disease. Most common arrhythmia to cause this?

Paroxysmal supraventricular tachycardia.

What clinical history would make a partial seizure with secondary generalization more likely than simple partial and complex partial?

Partial with secondary generalization is associated with loss of urine, tongue biting and tonic clonic seizure with elevated CPK. Simple is not associated with loss of consciousness. Complex is associated with automatisms like chewing, picking movements or lip smacking.

What findings can help differentiate arthritis due to parvovirus from rheumatoid arthritis?

Parvovirus is not associated with joint swelling nor joint stiffness of over an hour in the morning. Parvovirus is also acute (within 7 days) while rheumatoid symptoms last for weeks at a time.

Worsening insomnia, confusion and memory loss over the past 3 weeks in a 60 year old. Poorly groomed and disoriented to date and time. Nystagmus, hypokinesia and positive extensor plantar response bilaterally. CT is normal. What is expected on EEG?

Periodic sharp waves (history is concerning for Creutzfieldt-Jakob)

60 year old with lower GI bleeding. She has COPD with for pulmonale, hypertension and diverticulosis. She takes albuterol, furosemide and lisinopril. On admission her pressure was 80 over 50 and hemoglobin of 6.5. She was given fluids, blood transfusions and had her home meds withheld. She was stabilized with good urine output. On day 4, she was 110 over 70. Leukocytosis, hemoglobin of 10, urea of 60, creatinine of 3, sodium of 132, potassium of 5, chloride of 100 and bicarbonate of 18. Blood gases are 7.15 for pH and 80 and 60 for oxygen and carbon dioxide. What is the clinical problem and why?

Patient has a mixed acid-base disorder. CO2 is elevated indicating respiratory acidosis and her bicarbonate levels are lower indicating metabolic acidosis. This patient likely has CO2 retention due to underlying COPD leading to CO2 narcosis (seen at PaCO2 of greater than 60 mm Hg). She definitely does not have acute hypercarbia because you would expect chronic CO2 retainer stop have normal pH and high serum bicarbonate. Patient has AKI, see the Cr and BUN. Normal anion gap which helps exclude causes like uremia as a metabolic acidosis.

70 year old with inclusion body myositis has become less active. Blood pressure is 90/60 and heart rate is 100. Orthostatic vital signs. Exam shows frail man with dry oral mucosa and decreased axillary perspiration. Chemistry panel shows sodium of 165. Best next step and why?

Patient has hypernatremia due to severe hypovolemia likely because of dehydration. Due to his hypovolemic exam findings, best step is to give the patient Normal saline infusion first to treat his intravascular volume. After correcting this, a half normal saline infusion with 5% dextrose can address his body dehydration.

Potassium of 6 in an otherwise asymptomatic patient complaining of fatigue. History includes hypertension, diabetes, hyperlipidemia, chronic kidney disease and ischemic cardiomyopathy. No ECG abnormalities. Management?

Patient is asymptomatic with no ECG abnormalities proceed to review medications for cause of hyperkalemia.

60 year old with alcoholic cirrhosis is admitted for confusion and lethargy. Poor intake. Patient has been on furosemide and spironolactone. Cirrhosis stigmata present. Potassium of 3, bicarb of 30 and otherwise normal chemistries. What is the clinical problem?

Patient is experiencing hepatic encephalopathy due to hypovolemia and electrolyte disturbances.

Explain what the results mean for a patient from China being worked up for HBV: Positive: HBsAG, Anti-HBc and HBeAg Negative: Anti-HBs and Anti-HBe LFTs: Normal

Patient is in the immune tolerance phase of HBV infection. Such patients like this one can have this for up to 10-30 years. Likely perinatal infection. There is an active infectious process occurring and the immune system is not inflaming the liver.

Fever, anorexia, and right upper quadrant pain. Pain radiates to right shoulder upon inspiration. Bloody diarrhea 6 months ago when returning from Mexico. Ultrasound shows single thin wall uniform cyst on superior surface of right lobe of liver. Explain the radiating pain. Diagnosis?

Patient likely has an amebic liver abscess that is involving the pleura hence why the pain sensation radiates to the shoulder because of the phrenic nerve.

How does adrenal hemorrhage present as? Why does this presentation happen?

Patient on warfarin who has an acute stressor such as sepsis or a child suffering from meningococcemia. Stress increases the ACTH levels which increases adrenal blood flow.

Patient presents with complains of epigastric fullness or discomfort after eating. What are alarm symptoms that need to be asked?

Patient presents with complaint of dyspepsia, assess the following alarm symptoms. 1. weight loss. 2. gross or occult bleeding. 3. anemia. 4. dysphagia. 5. persistent vomiting. 6. early satiety. *Bottom line - associate GI cancers with bleeding and complications of a growing mass. If any of these are present consider endoscopy.

70 year old woman with diabetes returns with a creatinine of 5. Recently admitted one week ago for pyelonephritis. Her creatinine then was 2. Urinalysis today shows rare epithelial casts and no white blood cells. Fractional excretion sodium greater than 2%. What likely happened and reason out why?

Patient was on a macrolide such as amikacin. Macrolides are used to treat patients with pyelonephritis due to multidrug-resistant organisms. Patient is lacking eosinophils or white blood cell casts to suggest acute interstitial nephritis. Elevated fractional excretion of sodium would suggest that there is not a pre-renal cause.

Non-inflammatory chronic prostatitis. Presentation?

Patient who is afebrile with irritative voiding symptoms. Examination and urinalysis is unremarkable. Prostatic secretions demonstrate no leukocytes and is negative on cultures. Unremarkable PSA.

What are the typical clinical presentations of Toxic Shock Syndrome?

Patient with confluent erythematous macules with signs of severe sepsis such as skin desquamation, GI disturbances, myalgias, renal involvement, altered mentation and liver damage. the following are settings that predispose patients to this. 1. Tampons. 2. Septorhinoplasty. 3. surgical wound infections. 4. sinusitis (nasal packing).

What are the features of analgesic nephropathy?

Patient with long term multiple analgesics for chronic headaches and somatic complaints with laboratory findings of elevated creatinine with urinalysis showing hematuria or sterile pyuria. Mild proteinuria maybe represent. CT scan may show small kidneys with bilateral renal papillary calcifications.

What is febrile neutropenia?

Patient with no obvious focus of infection and an ANC of less than 1500 (severe would be < 500).

Describe how does pneumoconiosis presents?

Patient with prolonged occupational exposure to inhaled inorganic dust and develops dyspnea on exertion, pulmonary hypertension and cor pulmonale.

How does nocardia infect people? Histology?

Patients inhale saprophytes or they get into open wounds. These bacteria are Gram positive and beaded or branching in appearance that also stain partially acid-fast.

In what setting is digoxin often used for patients with supraventricular arrhythmias such as flutter, atrial fibrillation and atrial tachycardia?

Patients with hypotension and or heart failure who are unable to tolerate beta blockers or calcium channel blockers.

What is a characteristic difference between patients presenting to a PCP with Alzheimer's dementia vs pseudodementia?

Patients with pseudodementia will be overly concerned about their memory loss and often seek help. Those with Alzheimer's do not.

What 5 findings do you need to think about with cauda equina syndrome or conus medullaris syndrome?

Pattern of pain, area of anesthesia, presence of symmetry in motor weakness, quality of reflex, and early or late onset bowel and bladder dysfunction.

What diseases are associated with a normal plateau pressure and increased peak pressure? Why is this?

Peak pressure = resistive pressure and plateau pressure. Bronchospasm, mucus plug and biting ET tube cause an increase in resistive pressure and peak pressure as a result.

What diseases are associated with an increase peak pressure and plateau pressure? Why is this the case?

Peak pressure = resistive pressure and plateau pressure. Pneumothorax, pulmonary edema, pneumonia, atelectasis and right mainstem intubation as all theses disease decrease compliance (ability to accommodate a volume of air) which increases plateau pressure (a measure of lung compliance).

A nonpalpable point of maximal impulse is a clinical finding with what disease?

Pericardial effusion.

What seizure medication is known to cause cerebellar atrophy?

Phenytoin.

35 year old who is anxious with symptoms of palpations, throbbing headache and diaphoresis. Vitals show severely elevated BP of 230/110 and pulse of 110. Diagnosis?

Pheochromocytoma. *Note the young age. Associate headache from pressure, diaphoresis from sympathetics.

Causes of concentric hypertrophy?

Physiologically due to chronic pressure overload

when a non-complex empyema fails to drain with a single chest tube, what else can you do?

Place a second one just in case the first one is block.

Beefy red tongue, dysphagia and anemia. Diagnosis and treatment?

Plummer vinson syndrome. Iron supplement.

HIV patient with acute onset of high-grade fever and pleural effusion. CD4 is 300. Etiology?

Pneumonia due to streptococcus pneumoniae.

How does nocardiosis present?

Pneumonia in a HIV patient with or without extra pulmonary involvement such as cutaneous and systemic findings or a brain abscess.

Other than meningitis what rare presentation of cryptococcal infection can occur in HIV patients?

Pneumonia.

What does a SAAG of greater than 1.1 usually mean in a cirrhosis patient?

Portal hypertension

Pathology findings with PBC are?

Portal tracts infiltrated by lymphocytes, macrophages and eosinophils. Noncaseating granulomatous inflammation and progressive destruction are seen in the terminal and conducting bile ducts.

40 year old patient had a seizure. Has stopped taking his anti-seizure medicines for a year. Confused and lethargic on exam, otherwise unremarkable exam. Labs show sodium of 140, chloride of 103, bicarbonate of 17, bun of 20, and creatinine of 1. Diagnosis and management?

Post-ictal lactic acidosis. Observe for another 2 hours and repeat laboratories because the lactic acid is elevated only transiently for 60 to 90 minutes.

60 year old on day 4 post-op develops waxing and waning fever, tachypnea and shortness of breath. Exam shows fever of 40, pressure of 90 over 60, pulse of 110 and respirations of 22. Urine output is 100 milliliters. Blood gas shows ph of 7.2, PaO2 of 60, PaCO2 of 30 and HCO3 of 15. Likely diagnosis and management?

Post-operative pneumonia which has become septic considering signs of SIRS, hypotension and infectious process. IV antibiotics and fluids.

Amiloride, what is this?

Potassium-sparing diuretic.

Does prednisone have an application for contrast-induced nephropathy?

Prednisone does not prevent contrast-induced nephropathy. It can raise fluid retention and worsen blood pressure elevations. It can, however, prevent hypersensitivity reactions to contrast media though...

Name 3 common immunosuppressive agents used in treating myasthenia gravis?

Prednisone. Azathioprine. Cyclosporine.

What is the mechanism of disease with premature atrial contraction?

Premature activation of the atria originating from a site other than the SA node.

Greatest risk factor for cerebral palsy?

Prematurity (Do you remember the young Caribbean child at Athens Neuro?)

Why is it important to know if someone had a history of prior radiation, caustic injury, complex stricture or surgery for laryngeal or esophageal cancer when evaluating dysphagia?

Presence of the information would warrant doing a barium swallow first prior to an endoscopy as these patients are at an increased risk of esophageal perforation.

Abdominal pain, neurologic and psychiatric abnormalities in the setting of someone with essential tremors. Likely rationale for patient presentation? What needs to be checked to establish diagnosis?

Presentation is acute intermittent porphyria due to primidone use (an anticonvulsant which converts into phenylethylmalonamide and phenobarbital). Urine porphobilinogen

What is the presentation of renovascular hypertension? What is the treatment?

Presentation of headache, elevated blood pressure and renal bruit. Angioplasty with stent placement.

21 year old presents with dizziness and palpitations that started abruptly. Usually provoked by strong emotions and stopped by squatting and taking a deep breath, but this isn't working. Pressure is 65/40 and pulse is 240. Extremities are cool. ECG shows regular, narrow-complex tachycardia. Next best step?

Sedation and direct current cardioversion.

Define status epilepticus.

Seizure lasting greater than 5 minutes or a cluster of seizures with the patient not recovering a normal status in between.

Patient presents with the following. pH of 7.31, PaCO2 of 29, HCO3 of 14. Chemistry panel shows 132 and chloride of 85. What type of acid-base disturbance does this person have? Show your work

Primary metabolic acidosis with respiratory compensation. Anion gap acidosis present. Using winter's equation for metabolic acidosis to solve for a PaCO2 of 29 (which is the same as the number given).

Right upper quadrant pain in otherwise asymptomatic individual. Findings of hepatic steatosis on imaging and biopsy. What things need to be ruled before making the most likely diagnosis.

Rule out disease other organic disease such as significant alcohol use and other causes of fatty liver 1. hepatitis titers. 2. antimitochondrial antibodies. 3. ANA antibodies. 4. ceruloplasmin. 5. transferrin saturation.

What defines prolonged rupture of membranes?

Rupture over 18 hours

What is the most common cause of subarachnoid hemorrhage? What is the second most?

Ruptured saccular or berry aneurysm. Amyloid angiopathy.

Most common cause of pneumonia in nursing home residents?

S. pneumoniae

40 year old is in the ER with sudden onset of fever of 103, nausea, abdominal pain and dark urine. She has had a history of IV drug use and alcohol use and has completed rehab 2 years ago. AST and ALT are in the 2000 range. Positive HBV and undergoing treatment. On day 3 she becomes agitated and confused. ALT is now in the 4000s with other liver studies elevated. Best next step?

Put her on emergent liver transplant list. Do not waste time treating HBV, this woman is dying.

What is the pathophysiology with E. Coli infections an uterine contractions?

Pyrogens released from infection increases prostaglandins in the uterus leading to increased uterine contractions.

What adverse drug effect does stall have and what type of drug is this?

QT prolongation and Torsades. Class III antiarrhythmic.

Side effects of macrolide antibiotics include what heart and gastrointestinal manifestations?

QT prolongation and cholestasis.

Honey moon UTI in young women. Etiology?

S. saprophyticus.

Exertional dyspnea of 10 days. Vaginal delivery 3 months ago with frequent episodes of dark bloody vaginal discharge. Most recent bleeding was 6 days ago. VS WNL. Clear lungs on exam. Enlarged uterus on pelvic. Multiple bilateral infiltrates of various shapes on CXR. Next best step to establish diagnosis?

Quantitative beta HCG because of possible choriocarcinoma (a type of gestational trophoblastic disease). Note: all GTDs present with irregular vaginal bleeds, enlarged uterus and pelvic pain. Irregular vaginal bleeding 8 weeks (2 months) beyond postpartum is abnormal. Unlike choriocarcinoma, gestational trophoblastic neoplasia is localized.

30 year old female with 4 episodes of pneumonia in the past year presents again with SOB and cough. CXR shows RLL infiltrates and LUL fibrosis. Best next step?

Quantitative measurement of serum Ig levels as this patient may have an immunodeficiency.

What is the Bohr effect? How about the Haldane effect?

RBCs travel with oxygen to areas of increased PCO2. Exchange occurs because of relative acidity causing oxygen loading. The opposite effect with Haldane.

What germline mutations are involved with MEN2A and MEN2B?

RET proto-oncogene.

Crigler-Najjar syndrome type 1, how does it present? Treatment management?

Presents only in infants as this disease is fatal without liver transplantation. Sever jaundice and kernicterus. Indirect bilirubin levels above 20. IV phenobarbital is administered, serum

Other than TMP-SMX in the treatment of pneumocystis pneumoniae, what other drug is combined with clindamycin as an alternative form of treatment? Adverse drug effect?

Primaquine. Methemoglobinemia and hemolytic anemia (check for G6PD deficiency).

HIV patient with CD4 count less than 50 presents with altered mental status. CT exam shows solitary, irregular, weakly ring-enhancing mass in the periventricular area. Patient is on antiretroviral therapy, TMP-SMX and azithromycin. PCR shows EBV DNA in CSF and serology is positive for toxoplasma. Diagnosis and why?

Primary CNS lymphoma because of the presence of EBV DNA is specific in this condition.

Solitary mass, HIV patient and EBV positive in CSF. diagnosis?

Primary CNS lymphoma.

Hyperpigmentation, hypotension and hyperkalemia. Diagnosis?

Primary adrenal insufficiency.

50 year old with nocturnal substernal chest pain that wakes her up. Episodes last for 15 minutes. No drug use. Normal vitals. ECG shows transient ST element elevations in leads I, aVL and V4 though V6. Likely diagnosis? Treatment?

Prinzmetal's angina. Calcium channel blockers.

Behavioral changes, and myoclonus and or seizures that is rapidly progressive within the past month in a 50 year old patient. Likely diagnosis?

Prion disease.

myoclonus with rapidly progressive dementia in a patient between 50-70 years of age. What should you be thinking?

Prion disease.

Presentation of vasovagal syncope?

Prodrome of pallor, dizziness and nausea & vomiting in response to a trigger leading to short term syncope.

What medication is useful for palliation of anorexia and in improving appetite and weight gain with patients cancer-related anorexia or cachexia syndrome?

Progesterone analogs such as (megestrol acetate and medroxyprogesterone acetate).

What symptoms precede a syncopal episode?

Program of nausea, warmth, lightheadedness or diaphoresis.

JC virus causes what? How is this diagnosed?

Progressive multifocal leukoencephalopthy. PCR.

What laboratory abnormalities area associated with hemophilia?

Prolonged activated partial thromboplastin time. Decreased or absent VIII or IX activity. Normal platelets, bleeding time and prothrombin time.

Why do clonic jerks occasionally happen during a syncopal episode?

Prolonged brain hypoxia.

Leukocytoclastic vasculitis. Presentation?

Prominent cutaneous manifestations in the setting of pulmonary infiltrates and pleural effusions that came from a slowly evolving process.

What's the purpose of IV saline hydration for symptomatic hypercalcemia?

Promote urinary calcium excretion and restore intravascular volume.

Best medication for benign essential tremor?

Propranolol.

What is the mechanism which niacin causes flushing? What medicine can reduce the effect of this?

Prostaglandin-related reaction. Low-dose aspirin.

What key defining features differentiate prostate cancer from BPH from a digital rectal exam?

Prostate cancer usually involves... 1. lateral lobes of the prostate (BPH involves the central portion known as the "transitional zone") 2. asymmetrically enlarged, nodules and firm prostate.

Pathogenesis of osmotic demyelination syndrome? Management guidelines?

Rapid correction of hyponatremia with a more concentrated crystalloids will move fluid rapidly out of the neurons and into the extracellular compartment. This disrupts the cellular metabolic activity and damages the cell. Do not infuse pass 0.5 mEq/L/hr.

30 year old returns from Eastern Europe after swimming and hiking in the local lakes and streams. Diarrhea for 3 weeks. High-volume watery stools. No blood or mucus. No fever. Stool is negative for WBCs and RBCs. Generally, what type of infection is causing this patient's illness?

Protozoan parasite

How can thyroid disease mimic neuropathy and myopathy?

Proximal muscle weakness and diplopia. Note: thyroid-related myopathy rarely cause dysphagia or bilateral ptosis.

Palpable mass in epigastrium weeks after episode of acute pancreatitis? Pathology report of this diagnosis would read as?

Pseudocyst. Thick fibrous capsule of non-epithelial origin containing inflammatory fluid, tissue, and debris.

Cellulitis after a puncture wound, etiology?

Pseudomonas.

History of nail puncture wound and presentation of recent osteomyelitis. Etiology?

Pseudomonas.

What is the most common cause of pneumonia in adult CF patients?

Pseudomonas.

What is never a good sign on ABGs of an acute asthma exacerbation?

Pt with elevated PCO2 with hyperventilation indicates impending or actual respiratory arrest.

How is normal pressure hydrocephalus different from Alzheimer's in terms of presentation?

Pts with NPH will complain of urinary incontinence, slow, broad-based, shuffling gait and memory loss without focal neurologic changes.

What symptoms are associated with ascariasis?

Pulmonary and gastrointestinal complaints and possibly hepatobiliary involvement.

How is pulmonary hypertension defined as?

Pulmonary artery pressure above 25 mm Hg.

Elevated jugular venous pressure, hepatomegaly and peripheral edema in the presence of clear lungs in a patient with COPD. Diagnosis?

Pulmonary hypertension and cor pulmonale.

Describe the pulse findings of aortic stenosis.

Pulse with delayed upstroke, delayed peak and small pulse amplitude.

Azathioprine, mechanism of action and adverse drug effect?

Purine analog that is converted to 6-mercaptopurine which acts to inhibit purine synthesis. Toxicity causes diarrhea, leukopenia and hepatotoxicity.

Frost bite injury, treatment plan?

Rapid rewarming with warm water because there is more damage with slow rewarming at room temperature. Extremity should be immersed in warm water which is continuously circulated.

Tick-borne paralysis, presentation?

Rapidly progressive ascending paralysis, absence of fever and sensory abnormalities and normal CSF exam.

Describe the presentation of adult Still's disease.

Recurrent high fevers over 39 C, rash and arthritis. Rash is often maculopapular and nonpruritic affecting the trunk and extremities during febrile episodes. Note* renal disease and painful skin nodules are not seen.

What is the most common mechanism for ventricular fibrillation after a myocardial infarction.

Reentry

Why are dihydropyridine calcium channel blockers a bad idea in patients with an acute MI or CHF?

Reflex tachycardia along with increased myocardial oxygen demand.

How does desmopressin work and what is it used for?

Releases factor VIII:von Willebrand factor multimers from endothelial storage sites. This is a treatment option for the platelet dysfunction seen in chronic renal failure.

What is cryoprecipitate used for?

Replacement therapy for patients with fibrinogen, von willebrand factor or factor VIII deficiency.

In Paget's disease what are the markers of bone resorption and bone formation that are elevated?

Resorption - C-telopeptide and N-telopeptide. Formation - ALP, osteocalcin.

Why would a patient experience symptoms of hypocalcemia in someone who is having a pulmonary embolism?

Respiratory alkalosis causes an increase of free calcium to bind to albumin.

What spirometry findings are associated with musculoskeletal deformities and neuromuscular disease?

Restrictive pattern (FEV1/FVC >70%)

What pattern of FEV1/FVC and DLCO does heart failure present with?

Restrictive pattern and low DLCO.

What are the clinical features of histoplasmosis? Best means to diagnose this condition?

Reticuloendothelial system - pancytopenia, hepatosplenomegaly and adenopathy. Pneumonia - diffuse reticulonodular or cavitary. Mucocutaneous lesions. Urine antibody.

Intensive glycemic control of hemoglobin A1C <6.5% has the greatest benefit of reducing the risk of what complication.

Retinopathy or nephropathy (these are microvascular complications) Note: no effect on macrovascular complications

Why does exophthalmos happen in hyperthyroidism?

Retro-orbital tissue expansion and lymphocytic infiltration due to stimulation of orbital fibroblasts by the same anti-TSH receptor autoantibodies.

60 year old alcoholic hospitalized for generalized tonic-clonic seizures. Lab studies on second day show hyperkalemia, elevated urea nitrogen and creatinine. Urinalysis detects large amounts of blood. Sediment microscopy shows 0-1 RBCs with epithelial cells. Diagnosis? What are you suppose to key in on?

Rhabdomyolysis. Notice the discrepancy with urinalysis and urine sediment microscopy? Detected blood must have been from myoglobin.

Woman has sudden onset of severe shortness of breath and inability to lie flat. She is an immigrant from Eastern Europe and is 26 weeks pregnant. She had a history of recurrent sore throats. Heart rate of 12o. EKG suggest atrial fibrillation. Diagnosis? How does the pathophysiology relate to her illness?

Rheumatic Fever. Patient likely has mitral stenosis and as the typical features of volume overload occur due to pregnancy, she is now symptomatic because of her condition.

Etiology for mucormycosis? Treatment?

Rhizopus. Surgical debridement and systemic amphotericin B

Red urine while on TB medicines. What happened?

Rifampin ("R" for red?)

What medication can be used if lactulose isn't helping with hepatic encephalopathy due to ammonia?

Rifaximin (a non-absorbable antibiotic)

Patient has hemi-neglect syndrome with symptoms of anosognosia. Where is the lesion?

Right (non-dominant) parietal lobe (the area responsible for spatial organization... of that side?)

Someone suspected of having a massive PE would have what worrisome finding on ECG?

Right axis deviation.

How does a restrictive lung disease affect the pulmonary flow-volume curve?

Right shift of graph, indicating decrease in lung volume and also a decrease in flow rate on both inspiration and expiration.

30 year old on warfarin because of DVT 3 weeks ago. INR is not in range because of diet. Presents with worsening pain and swelling in leg. Venous Doppler ultrasound shows popliteal vein thrombus extending to femoral vein. Best next step and why?

Rivaroxaban because patient needs immediate anticoagulation. Clots in the proximal veins like the proximal popliteal, femoral or iliac vein are at high risk of pulmonary embolism.

Hypertrophic obstructive cardiomyopathy, pathogenesis?

Sarcomere gene mutation.

How are ventricular aneurysms characterized on pathology?

Scarred or fibrotic myocardial wall from the healed transmural MI.

Patient with mechanical valve and palpitations while exercising. HCT is 30%. Best thing to evaluate for? Diagnosis?

Schistocytes on blood smears and elevated LDH. Traumatic hemolysis.

Marked hypretension and AKI in the setting of systemic sclerosis. Diagnosis?

Scleroderma renal crisis. Note 1/5 of pts with diffuse cutaneous systemic sclerosis.

Condyloma lata, hepatitis, diffuse rash, oral lesions and lymphadenopathy. Diagnosis? (be specific)

Secondary syphilis.

Rash in palms and soles, fever and sore throat and international travel. Diagnosis in consideration? Initial tests than most accurate test?

Secondary syphilis. RPR or VDRL. FTA antibody

Presentation of hyperemesis gravidarum

Severe and persistent nausea and vomiting with laboratory derangements such as hypokalemia and ketonuria and more than 5% loss of pre-pregnancy weight.

What is the correlation of severe aortic regurgitation to length of murmur when compared to mild?

Severe aortic regurgitation is associated with a holodiastolic murmur.

Calcium levels > 14 mg/dL, What do you do?

Short-term - normal saline hydration plus calcitonin and avoid loop diuretics unless volume overload is present. Long-term - bisphosphonates.

Presentation of amniotic fluid embolism?

Shortness of breath after delivery or c-section or within a 1 week period after delivery.

How does erysipelas present?

Sharply demarcated, erythematous, edematous, tender skin lesion with raised borders while having systemic signs like fevers and chills.

35 year old woman gave birth 2 months ago complains of lethargy, weight gain and fatigue. Denise headache or visual disturbances. Delivery was complicated by vaginal bleeding requiring blood transfusion and postpartum endometritis. Exam shows sparse pubic hair, dry skin and decreased DTRs. diagnosis?

Sheehan's syndrome (hypopituitarism).

Pathogenesis of hemolytic uremic syndrome with EHEC?

Shiga toxin activates endothelial cell which leads to excess thrombosis.

Patient has had bloody diarrhea for the past 3 days and does not have a fever. Diagnostic test? Treatment is being pursued but what is a possible complication with treatment?

Shiga toxin in stool. Hemolytic Uremic Syndrome.

What is the lung pathology that does not correct with supplemental oxygen?

Shunts.

Patient presents with a MI complicated by cardiogenic shock and has complications of renal failure and GI bleeding from anticoagulation. T3 levels drop and T4 and TSH are normal. Diagnosis? Pathophysiology?

Sick euthyroid syndrome (low serum thyroid hormones in clinically euthyroid patients with nonthyroid systemic illness). Caloric deprivation and an increase in cytokine levels.

Pappenheimer bodies. Dz association?

Sideroblastic anemia

What is the most common infectious disease cause of a brain abscess? How about other the other ones?

Sinusitis (50%) Hematogenous (25%), like pneumonia or endocarditis

What type of gait is neurosyphilis associated with?

Slapping sounds as feet are lifted higher than usual.

What is athetosis? What disease is associated with this?

Slow, writhing movements that affect the hands and feet. Huntington's disease has both features of athetosis and chorea.

How does glioblastoma multiforme present?

Solitary mass with central necrosis and extensive vasogenic edema.

Mechanism for neurogenic fever.

Someone who has a stroke at the hypothalamus which disrupts the thermoregulatory mechanism.

How do you reason out a chronic CO2 retainer?

Someone who has physiologically adapted to their disease. Hence, their pH should be normal and their bicarbonate should be elevated to compensate for CO2 retention.

How long can post concussive syndrome last?

Sometimes they can be persistent and last over 6 months.

45 year old continued RUQ pain despite elective cholecystectomy. Labs show highly elevated ALP and moderate ALT and AST elevations. Bilirubin is 1.5. ERCP shows normal bile ducts and pancreas. Diagnosis? Test to confirm? Treatment?

Sphincter of Oddi dysfunction. Sphincter of Oddi manometry. ERCP sphincterotomy.

GBS is suspected, what testing needs to be done to assess the patients functional status? If this test can't be done what else can be done?

Spirometry to assess forced vital capacity, the gold standard for assessing ventilation. Peak flow meter to assess maximal speed of expiration, not accurate but an option. Note, decline of < 20 ml/kg indicates respiratory distress.

What is the clinical problem? Fever, chills, left upper-quadrant pain, leukocytosis and splenic fluid collection. What etiologies are associated with this problem?

Splenic abscess. 1. Staph. 2. Strep. 3. Salmonella.

Alcoholic patient with a history of ascites presents with diffuse abdominal pain and confusion. Serum albumin is 2.2 and paracentesis shows an albumin of 0.8 and absolute neutrophil count of 315. Diagnosis and the criteria to fit that? Treatment?

Spontaneous bacterial peritonitis. 1. > 250 PMN count. 2. SAAG >1.1. 3. ascites protein < 1 g/dL & glucose > 50 mg/dL. Culture and Antibiotics - 3rd gen cephalosporin such as cefotaxime. *note: spontaneous bacterial peritonitis does not necessarily inflame and disrupt the capillary beds of the peritoneum. Also, most of these patients have some form of underlying cirrhosis. Hence low ascites protein.

What finding differentiates syringomyelia from anterior spinal cord syndrome?

Syringomyelia has no involvement with pain and temperature loss below the lesion.

Greatest risk factor for acute aortic dissection?

Systemic hypertension.

Most common cause of diastolic dysfunction?

Systemic hypertension.

Pustular and ulcerated lesions localized to site of wound or associated lymphatic channels. Name the infection.

Sporotrichosis

What parameters would rule out the possibility of a malignancy when a patient has a solitary pulmonary nodule.

Stable size of for 2 years.

NSAIDS mainly affect which organ in the GIT?

Stomach, hence gastric ulcers and gastritis.

If you suspect C. diff what do you order?

Stool antigen.

What should you do with evaluating a HIV patient presenting with acute watery diarrhea, cramps and fever other than provide fluids and electrolyte support?

Stool culture to examine for ova and parasites and test for C. difficile toxin.

2o year old complains of heat intolerance, sweating and palpitations as well as menstrual irregularities. She has increased appetite and diarrhea. Exam shows diffuse enlarged, non-tender thyroid gland. TSH is depressed and T4 and T3 are elevated. Thyroid stimulating IgGs are present. She is on propylthiouracil. Her physical exam is 101.5 F Her soft palate, pharynx and tonsils are red and swollen next appropriate step in management? What are the guidelines?

Stop PTU because of agranulocytosis. Any patient taking PTU complaining of fever and sore throat need to be taken off of PTU. Note: usually in the first 90 days of the medication is when this side effect occurs.

Patient presents with signs and symptoms of small bowel obstruction; however, vital signs show fever and tachycardia while labs show leukocytosis and metabolic alkalosis. Concerns? Treatment?

Strangulation. Urgent surgical exploration.

How can effect modification be distinguished from confounding?

Stratified analysis centering on the variable of interest. Basically another variable is put into play and whether or not there is a strong association or not dictates if there is confounding or effect modification. With confounding the presence or absence of the variable has no effect on the outcomes.

What is the treatment for cluster headaches?

Sumatriptan and high-flow oxygen.

30 year old male suffered serious chest trauma from a motor vehicle collision. 1 liter of blood evacuated from chest. 10 days later discharged home. Returns with low-grade fever and dyspnea. CT scan shows complex located effusion with a thick surrounding peel. Chest tube is placed. Still has low-grade fever. Next best step and why?

Surgery. Patient has empyema, an infection in the pleural space.

Patients with idiopathic intracranial hypertension that is refractory to medical treatment or those with progressive vision loss need what? For patients with worsening symptoms what can be given to bridge the patient to surgery?

Surgical intervention with optic nerve sheath decompression or lumboperitoneal shunting. Steroids.

What treatment option is recommended for a 65 year old man with a solitary brain metastasis at the right hemisphere at the grey-white matter junction?

Surgical resection of the mass.

What is the most common cause of out-of-hospital sudden cardiac arrest in adults?

Sustained ventricular tachycardia or fibrillation due to acute myocardial ischemia or infarction. Note: about 10% are discharged out of the hospital.

Patient had pyelonephritis and was treated with IV ceftriaxone. 2 days after, patient felt better with good appetite and is afebrile. Culture studies available. Next management plan?

Switch patient to oral antibiotics.

Early sings of spinal cord compression?

Symmetric lower extremity weakness, hypoactive or absent deep-tendon reflexes.

What is vasovagal syncope?

Syncope characterized by nausea, diaphoresis and pallor prior to syncope, often triggered by a stress, pain and certain actions.

What cause of syncope may not be associated with prodromal symptoms?

Syncope due to cardiac arrhythmias.

What is Dejerine-Roussy syndrome?

Syndrome due to a stroke involving ventral postero-lateral nucleus of the thalamus. Lesions to this area will cause contralateral hemianesthesia accompanied by transient hemiparesis, athetosis or ballistic movements. Dysesthesia of the area affected by the sensory loss is known as thalamic pain phenomenon.

Unexplained hemolytic anemia and thrombocytopenia in a patient with renal failure and neurologic symptoms. Hemoglobin of 8. Platelet of 80,000. Fragmented RBCs Diagnosis? Treatment?

TTP-HUS. Plasmapheresis.

In terms of understanding mild vs moderate hypothermia what happens?

Tachycardia, tachypnea, and hyperventilation as well as confusion, ataxia, dysarthria and increased shivering become the following: Bradycardia, lethargy, hypoventilation, decreased shivering, loss of pupillary reflexes and atrial arrhythmias. Bottom line: body is less able to respond at (28-32C or 82-90F)

Progressive dyspnea, decreased exercise tolerance, atrial fibrillation with rapid ventricular response and left ventricular systolic dysfunction. Diagnosis?

Tachycardia-mediated cardiomyopathy.

55 year old with 4-week history of fatigue and decreased exercise tolerance. Occasional palpitations without chest pain. Routine check-up 6 months ago normal. No medicines. Pressure of 150/90 and pulse of 13o irregular. Clear lungs. No clear P waves on ECG. Echo cardiography shows Ejection fraction of 35% with moderate mitral regard and left atrial and ventricular dilation with global hypokinesia. Diagnosis?

Tachycardia-mediated cardiomyopathy. Note: mitral regurgitation in this patient is due to mitral annular enlargement secondary to LV dilation.

What cytochemical feature is associated with Leukemic reticuloendotheliosis (hairy cell)? Why is it that marrow aspirations are frequently "dry tap"?

Tartate-resistant acid phosphatase. Extensive fibrosis.

Clinical presentation of M. avium complex? What other diseases presents like MAC?

Tb-like picture of nausea, vomiting, abdominal pain, weight loss, cough, fever, and night sweats with exam findings of splenomegaly in a HIV patient with CD4 counts less than 50. Labs demonstrate elevated ALP. Tb and CMV.

More of a note: Pay attention to the variable axis and understanding what it means to move it from left to right (Bioethics questions)

Test 1 number 17.

40 year old man presents with fatigue. Hemoglobin of 8 and MCV of 70. Iron and ferritin levels are low. Best next step?

Test for occult blood in stool. Note: knowing why the patient is iron deficient is more important that administering iron and seeing if he improves.

Why is there normal breast development in patients with complete androgen insensitivity?

Testosterone is converted into estrogen peripherally.

A hemorrhagic lesion to what part of the brain would cause contralateral gaze deviation, hemiparesis and hemisensory loss?

Thalamus.

Of the causes of microcytic anemia, what diseases cause an increase of iron and ferritin and why?

Thalassemia due to RBC turnover.

When evaluating for causes of hypercalcemia, what does one need to be cautious not to assume about with interpreting PTH values and exogenous sources?

That PTHrP can cause the same effects of PTH but will not be seen as an increase in PTH because they are different.

When evaluating blood urea nitrogen in the setting of GI bleeds and possible acute kidney injury, what needs to be considered?

That the blood urea nitrogen elevation can be due to byproducts from the GIT. Calculate anion gap to evaluate for presence of uremia.

Unilateral flank pain, poor urine output, intermittent episodes of high volume urination and occasional RBCs and WBCs but no casts. Diagnosis and why?

Unilateral obstructive uropathy due to renal calculi. Unilateral flank pain correlates with this diagnosis and the high volume urination describes a pattern of dislodge stone. Lack of casts support this is not a glomerular issue.

Hypertension, abdominal bruit and flash pulmonary edema.

Unilateral renal artery stenosis.

How is it that radioactive iodine can worsen Grave's opthalmopathy?

The damaging effects from the treatment can transiently elevate the levels of circulating thyroid hormone, worsening symptoms of opthalmopathy.

Why is it advisable to do a diagnostic peritoneal lavage instead of immediate laparotomy in unstable patients, who are suspicious for abdominal bleeding with an inconclusive FAST?

The laparotomy itself may lead to clinical deterioration.

40 year old presents with 6 months of profound fatigue, 4 months of fleeting joint pains and an itchy rash on his legs 2 months ago. Exam shows palpable rash on bilateral lower extremities that do not blanch with pressure. Labs significant for creatinine of 2, AST & ALT mildly elevated, C3 and C4 low, Rheumatoid factor elevated, negative ANA and HIV. Urine studies show 3+ blood and protein with few dysmorphic red blood cells. What is the clinical problem? What is the next best diagnostic step and what diagnosis are you chasing?

This patient has acute moderate-to-severe GN with low complement levels and nephritic urinary sediment. Viral hepatitis serology because this patient has mild elevations in transaminases and rashes suggestive of cryoglobulinemia which HCV is associated with. Things that are less likely are lupus and postinfectious GN.

Pregnant woman presents with severe RLQ pain, fever, and nausea. Next best step?

Ultrasound (remember she's pregnant).

What is the purpose of classifying patients as uncomplicated or complicated with regards to a cystitis presentation?

The need for ordering urinary cultures prior to therapy.

Pulsatile mass on abdominal exam. Next best step.

Ultrasound not CT.

On exam, how are consolidations such as lobar pneumonia different from pleural effusions and pneumothorax.

Unlike pleural effusions, consolidations amplify breath sounds and tactile fremitus (no decreased breath sounds either) Unlike pneumothorax, consolidations are dull to percuss and not hyperresonant.

What is anatomically damaged in cauda equina syndrome?

The spinal nerve roots Note: do not put conus medullaris or lumbar spinal column.

What do fluconazole and moxifloxacin have in common with regards to the heart?

Their side effects cause QT prolongation.

Why are beta blockers, calcium channel blockers and nitrates withheld 48 hours prior to stress testing?

These drugs reduce the extent and severity of ischemia during exercise stress testing and limits the accuracy of the test itself.

Patient presents with hyponatremia, low serum osmolarity, hypovolemia, and low urine sodium. Name 3 causes.

These findings are due to non-renal salt loss such as... 1. diarrhea. 2. vomiting. 3. dehydration.

Patient presents with hyponatremia, low serum osmolarity, hypovolemia, and high urine sodium. Name 3 causes.

These findings are due to renal salt losses such as... 1. diuretics. 2. ACE-inhibitors. 3. Mineralocorticoid deficiency.

What is the clinical significance of an S4 and elevated electrocardiogram voltages in a stroke patient?

These findings correlate to left ventricular hypertrophy, most commonly due to long-term hypertension. The greatest risk factor for a stroke.

Why is there right upper quadrant pain in HELLP syndrome?

These occurrences happen, centrilobular necrosis, hematoma, and thrombi in the portal capillary system which cause liver swelling with dissension of the hepatic capsule thus leading to pain.

Why is hypoxemia in ARDS assessed by PaO2/FiO2?

These patients need to be on a ventilator. By assessing the arterial blood oxygen to the change in FiO2 shows progress or worsening respiratory status.

With any wide-complex tachycardia what would warrant synchronized cardioversion? and define the parameters.

Unstable patient with findings as follows: 1. hypotension. 2. altered mental status. 3. respiratory distress (pulmonary edema).

How is opioid intoxication different from cocaine?

They are complete opposites. Opioid - bradycardia, hypotension, decreased respiratory rate, and miosis. Cocaine - tachycardia, hypertension, tachypnea, and mydriasis.

How are Chlamydia and Salmonella similar in terms of symptoms?

They both are associated with reactive arthritis.

How does a nerve conduction study help with the diagnosis of peripheral neurologic disorders?

They help differentiate wether or not there is a neuromuscular junction, muscle or nerve pathology.

How are vancomycin and gentamicin similar? Different?

They mainly work against aerobic organisms. Vancomycin: gram positive. Gentamicin: gram negative. If using these 2 together it has no effect on anaerobic organisms.

What is typical of streptococcal pharyngitis with regards to upper respiratory symptoms?

They mostly are pharyngeal symptoms while viral conditions are mostly rhinorrhea, coryza, sneezing and mild pharyngitis.

How are ethylene glycol and methanol intoxication similar and different?

They present similar in symptoms such as headache, nausea, vomiting and epigastric pain; however they are different as follows. 1. methanol intoxication involves the eyes. 2 ethylene intoxication involves the kidneys.

What class of drugs does ticlopidine belong to? What other drug is in this class.

Thienopyridines. Clopidogrel

DiGeorge syndrome, define it.

Think "CATCH" Conotruncal cardiac defects. Abnormal facies. Thymic aplasia or hypoplasia. Cleft palate. Hypocalcemia. Note: the hypocalcemia can trigger seizures, arrhythmia and tetany

Lesch-Nyhan syndrome define it.

Think HGPRT (the protein that is messed up. Hyperuricemia. Gout. Pissed (self-mutilation). Retarded. Tone (dystonia, extrapyramidal symptoms including choreoathetosis)

Double duct sign

Think pancreatic head cancer or ampullary cancer

Escape rhythm, constant R-R interval, and constant P-P interval.

Third degree heart block

Nocardia presents how?

This infection presents in immunocompromised patients and mimics findings similar to Tb.

Which population of patients would HCV testing be warranted for those who received blood transfusion in their lifetime?

Those before 1992, when HCV was screened for.

What age group is associated with amyloid angiopathy?

Those older than 75 years of age.

What is the pathophysiology of malignant hyperthermia.

Those with the autosomal dominant disorder are susceptible to anesthetic drugs halothane and succinylcholine will have uncontrolled efflux of calcium from the sarcoplasmic reticulum.

Hepatorenal syndrome, pathogenesis? Treatment.

Thought to be due to renal vasoconstriction in response to decreased total renal blood flow and vasodilatory substance synthesis. Liver transplantation.

60 year old man presents with severe right upper quadrant pain, nausea and fever of 38.8 C. Past medical includes hypertension, coronary artery disease, diabetes, and venous stasis ulcers. Gallbladder is palpable. Liver studies only show 2 times more than normal AST and ALT values otherwise normal. Leukocytes of 16,500. Potassium is 5.1 and otherwise normal chemistry panel. Diagnosis? and next best step to confirm?

Ultrasound. Emphysematous cholecystitis. Ultrasound findings will show curvilinear gaseous shadows.

25 yo with 3 month history of weight loss, irritability, insomnia and palpitations. BP is 155/70 and pulse is 110. Lid retraction, dry skin and fine tremors. Diagnosis? Cause of Hypertension?

Thyrotoxicosis. Increased myocardial contraction.

35 year old brought to ED with progressive ascending paralysis that began 18 hours earlier. Initial symptoms included paresthesia in the lower extremities followed by a sense of fatigue and weakness. Hiker in Colorado. VS are WNL. Absent DTR in lower extremities with normal sensation and cranial nerves. No abnormalities on CSF nor CBC. Diagnosis?

Tick-Borne paralysis.

focal neurologic deficit following a seizure. What is this?

Todd's paralysis.

Patient was noted to be disoriented for an hour after observed convulsions. Patient is noted to have facial drop. What is the disease being described? How does one diagnose this?

Todd's paralysis. EEG.

What drug can help with significant hypervolemic hyponatremia (heart failure) or euvolemic hyponatremia (SIADH) that does not improve with standard therapy (water restriction).

Tolvaptan

Involuntary, stereotyped, repetitive movement and vocalizations such as shrugging, blinking, grimacing and coprolalia. Diagnosis?

Tourette syndrome.

CT showing multiple ring-enhancing lesions and upper motor neuron stigmata in an HIV patient. Diagnosis and treatment?

Toxoplasmosis. Sulfadiazine and pyrimethamine.

What is a finding of the extremities associated with amlodipine?

Trace edema.

What is the normal pH of transudative fluid? What is normal pleural fluid pH?

Trans: 7.4-7.55 Norm: 7.6

What should a physician due prior to attempting cardioversion in a (stable?) patient with atrial fibrillation?

Transesophageal echocardiogram

Lower left abdominal pain that radiates to groin. Extreme discomfort lying on exam table. Best test to diagnose condition? Diagnosis?

Ultrasound. Urolithiasis. *Lying on table worsens pain via less gravity?

Vasa previa, what is it?

Umbilical cords lying above the cervical os.

Pathophysiology of D-lactic acidosis?

Unabsorbed carbohydrates are metabolized by intestinal bacteria producing D-lactic acid, which is absorbed.

30 minutes after a transfusion patient experiences a fever of 39 C, has chills and severe flank pain. Diagnosis? What is causing the flank pain? Diagnostic test to confirm disease.

Transfusion reaction due to acute hemolytic reaction due to ABO incompatibility. Immune complexe deposition in the kidney. Positive direct Coombs test

Respiratory distress and signs of noncardiogenic pulmonary edema within 6 hours of transfusion. Diagnosis and pathogenesis?

Transfusion-related acute lung injury. Donor anti-leukocyte antibodies.

What is the typical scenario with contrast-induced nephropathy?

Transient spike creatinine within 24 hours of contrast administration. Return to normal renal function within 5 to 7 days.

Patient presents with hepatic hydrothorax. What is hepatic hydrothorax? Treatment?

Transudative pleural effusion in cirrhosis patients with cardiopulmonary disease. Therapeutic thoracentesis followed by salt restricted diet and diuretics.

Patient with renal insufficiency has a ventilation/perfusion scan done and there is an absence of abnormality. What do you do next?

Treat for PE if suspicion is high.

HIV patient presents with dysphagia history and thrush on exam. Treat or do esophagoscopy?

Treat with fluconazole, oral. Perform endoscopy if there is no thrush or failure to respond to therapy.

Patient presents with CHF findings, S4 and an aortic stenosis. What age groups are bicuspid aortic valves more likely the cause for aortic stenosis?

Under 70 years of age. Otherwise, patients older than 70 would likely have senile calcific aortic stenosis.

What holosystolic murmurs can be heard at the tricuspid area?

Tricuspid regurgitation and ventricular septal defect.

What parts of the history would help you cue you in on neurocardiogenic syncope?

Triggers from prolonged standing, emotional distress or painful stimuli. Dizziness, nausea, pallor, diaphoresis, abdominal pain or generalized sense of warmth prior to episodes.

50 year old. History of heartburn and upper abdominal pain. Epigastric tenderness. Tender, erythematous and palpable cord-like views on arm and chest. Finding? Diagnosis? Test?

Trousseau's syndrome (migratory superficial thrombophlebitis) Pancreatic cancer. CT

True or false, communication failure between physicians during patient handoffs are a large contributor to medical error and adverse patient outcomes.

True.

Mexican 40 year old with weight loss, fever, cough with sputum, nausea, abdominal pain, and postural dizziness. Patient is diagnosed with adrenal insufficiency with cosyntropin stimulation test to verify. CT scan shows calcification of adrenal glands. What is the cause of this patients disease?

Tuberculosis.

When would you a patient ever need a surgical excision of a lung nodule?

Two conditions. 1. high malignancy risk. 2. patient of low risk with a nodule that is equal to 8 mm or greater in size that is suspicious for malignancy after biopsy or FDG-PET.

TTP-HUS, presentation? Enzyme related to this disease hence treatment?

Unexplained hemolytic anemia and thrombocytopenia in a patient with renal failure and neurologic symptoms. Autoantibody against ADAMTS-13 (vWF-cleaving protease). Plasmapheresis removes the offending antibodies and repletes the deficiency enzyme.

What are the types of hepatorenal syndrome?

Type 1 and Type 2. Differences is one is rapid the other is not (death within 10 weeks vs 3-6 months). Cause of death is due to hemorrhage or infection.

Halothane is not used in the USA anymore. Why?

Type 2 halothane reactions caused severe or fulminant hepatitis.

Dense intramembranous deposits that stain for C3.

Type 2 membranoproliferative glomerulonephritis.

What patients have a high pretest probability of coronary artery disease?

Typical angina in men over 40 years old. Typical angina in women over 60 years old.

What is esmolol used for?

Ultras-short acting beta blocker used for rapid rate control in atrial flutter or fibrillation.

30 year old man gasping for air and is agitated. shortness of breath and difficulty swallowing. Exam shows excessive accessory respiratory muscle use, retraction of subclavicular fuss during inspiration, and scattered urticaria over the body. Mechanism of disease?

Upper airway obstruction due to laryngeal edema.

What does a positive pronator drift exam finding indicate? How can this be helpful?

Upper motor neuron disease. The exam is assessing for weakness in supination. Patients who feign upper-extremity weakness will typically drop the "affected" arm without pronating.

How does one establish a diagnosis of vasovagal syncope?

Upright tilt table test.

How does one confirm a diagnosis of vasovagal (neurocardiogenic) syncope?

Upright tilt table testing. Note: this is a clinical diagnosis, this test is unnecessary unless a case is uncertain.

How does uremia cause an anion gap acidosis?

Uremia is another way of saying renal failure hence the failure to excrete acids as NH4+ causes acidosis.

Patient has a 1 year history of non-STEMI and CKD, and presents with pericarditis. What diagnosis is more likely and why?

Uremic pericarditis over Dressler's syndrome as the later usually occurs 1 to 6 weeks after a myocardial infarction.

What's the workup for an epidural abscess?

Urgent MRI. Blood cultures. CRP and ESR. CT guided aspiration and culture.

What does nitoprusside test measure? Limitations?

Urinary acetoacetate and acetone as to evaluate for ketones. It does not measure beta-hydroxybutyrate which needs to be assayed directly in DKA.

For the workup of hypercalcemia. If PTH levels are normal or elevated, what is the best next step? What are you trying to tease out?

Urinary calcium. High would be a primary or tertiary cause of hyperparathyroidism. Low would be a familial hypercalcemic hypocalciuria (this is due to a calcium sensing receptor issue).

What characteristics are associated with saline-unresponsive metabolic alkalosis

Urinary chloride of greater than 20.

"Urticarial" or allergic transfusion response. Presentation?

Urticaria, flushing, angioedema, and pruritus within 2-3 hours after transfusion.

40 year old man comes for recurrent palpitations. Diagnosed with atrial fibrillation. Echo is normal. He's placed on new medicines. Weeks later goes on a stress test. QRS complex increases from 0.09 to 0.13. What is this phenomenon? What types of medicine do this?

Use dependence. Class I (esp. IC) and class IV antiarrythmic drugs.

Lorazepam or midazolam for acute panic attacks?

Use lorazepam. Midazolam is used for sedation prior to a medical procedure.

How does a deep lacunar stroke present? What is the pathophysiology?

Usually a pure motor stroke involving both upper and lower extremities. Microatheroma and lipohyalinosis in small penetrating arteries of the brain.

Acute interstitial nephritis. Presentation ?

Usually due to exposures of beta lamas or proton pump inhibitors which symptoms occurs 1 week after the drug exposure. Findings include skin rash, eosinophilia, eosinophiluria, and pyuria.

For patients with IgA nephropathy, is the prognosis good?

Usually prognosis is benign. However, it is possible that rapidly progressive glomerulonephritis can develop or nephrotic syndrome can happen.

How is the presentation of uterine rupture different than that of abrupt placentae?

Uterine rupture is associated with cessation of uterine contractions and palpable fetal parts.

Tolvaptan. What is it? What ADE?

V2 vasopressin receptor antagonist that causes a selective water loss in the kidney without affecting sodium or potassium excretion. Increased liver enzymes.

Patient presents with chronic diarrhea that is tea colored. She admits to having muscle weakness, cramps, flushing, nausea and vomiting. BMP show low potassium. CT scan shows mass in pancreatic tail. What is the syndrome? What other electrolyte abnormality can present and why? Treatment?

VIPoma syndrome a.k.a. pancreatic cholrea. Hypocalcemia due to elevated PTH, MEN type 1. Treatment includes IV fluids, octreotide and possibly hepatic resection for metastasis.

Which cranial nerve innervates the baroreceptors and chemoreceptors of the aortic arch.

Vagus.

What is the anatomic reason why cavernous sinus thrombosis.

Valveless veins of the facial and ophthalmic venous system.

Red man syndrome?

Vancomycin.

20 year old man who underwent bone marrow transplantation for acute myelogenous leukemia complains of fever, severe diarrhea, pruritus and jaundice. Liver biopsy shows decreased quantity of bile ducts. What is the syndrome? Diagnosis?

Vanishing bile duct syndrome. Primary biliary cirrhosis. Note: biopsy is describing something called ductopenia.

Describe atrial fibrillation on ECG.

Varying R-R intervals with no clear discernible p waves and narrow-complex tachycardia.

What is electrical alternans?

Varying amplitude of the QRS complexes. Note: electrical alternans with sinus tach is highly specific for pericardial effusion.

Stepwise decline, objective neurologic deficits, and early executive dysfunction in someone suspected of having dementia. Diagnosis?

Vascular dementia.

What are the most likely organisms to cause a brain abscess from sinusitis?

Viridans streptococci and anaerobes such as Prevotella, Peptostreptococcus and Bacteroides.

What is the major cause of death with subarachnoid hemorrhage on days 3 to 10 after the initial stroke?

Vasospasms (occurs in 30% of patients)

Symptoms of type 2 heparin-induced thrombocytopenia?

Venous thrombosis - DVT, PE or skin necrosis. Arterial thrombosis - limb ischemia, organ infarction (mesenteric or kidney) and stroke.

60 year old, progressive exertional dyspnea and fatigue. 2 months ago hospitalized for an anterior wall myocardial infarction. 3/6 holosystolic murmur at apex with radiation to the axilla. Likely diagnosis?

Ventricular aneurysm. Do not be tempted to pick papillary muscle rupture. That would be more of an acute event (2 days to 1 week).

Treatment for NPH?

Ventriculoperitoneal shunt.

Locate the lesion: a lacunar infarct that causes a pure sensory stroke.

Ventroposterolateral nucleus of the thalamus.

What artery is involved with medial medullary syndrome?

Vertebral or anterior spinal artery.

Fever, back pain and focal spinal tenderness after a recent infection. Diagnosis? Test?

Vertebral osteomyelitis. MRI

Patient presents with acute onset of dizziness that has lasted for several days. There is no hearing loss and patient falls to one side. Recently the patient has had a cold. Diagnosis? Exam maneuvers?

Vestibular neuritis. Abnormal head thrust test.

Shellfish and vomiting, diarrhea and abdominal pain. Diagnosis? What patients are at risk of life-treatening complications with this disease?

Vibrio vulnificus. Immunocompromised patients or those with liver disease.

severe odynophagia without dysphagia in a patient with HIV. Diagnosis?

Viral esophagitis.

What can incite digoxin toxicity? Why is this?

Viral illness or excessive diuretic use can lead to volume depletion or renal injury elevating the digoxin level. Renal excretion.

What vitamin deficiency is associated with Tb medications?

Vitamin B6 deficiency due to isoniazid use.

Abdominal pain, constipation, polyuria and polydipsia.

Vitamin D toxicity. Note: this is hard to distinguish from diabetes

If a patient is not adequately treated for compartment syndrome in the arms, what can happen?

Volkmann contracture

Pregnant woman has hyperemesis gravidarum. How is it she can compensate for her metabolic alkalosis?

Volume depletion from vomiting causes contraction metabolic alkalosis.

Staph aureus poisoning, presentation

Vomiting and abdominal pain with rapid onset of symptoms. Think potato salad.

What is normal CSF?

WBC and RBC < 5. Protein <40 mg/dL Glucose at 40-70 mg/dL (or 2/3 serum)

What anticoagulation drug causes a low protein S

Warfarin.

How is a water stress test done? What results do you expect for someone with primary polydipsia? Why?

Water deprivation for 2-3 hours. If urine OSM is over 600, it is primary polydipsia. It's basically self induced over drink of water to a point the patient is surpassing his/her kidney's ability to excrete the water. However, these patients still have intact ADH responses like anyone else.

Best test to discern the various causes of polydipsia

Water stress test helps distinguish causes such as primary polydipsia, central diabetes insipidus and nephrogenic diabetes insipidus.

How do ventricular aneurysms present?

Weeks to months after an acute STEMI, patients may experience heart failure, refractory angina, ventricular arrhythmias or systemic arterial embolism due to mural thrombus formed within aneurysm.

Patient is a 40 year old man with type 2 diabetes. Present for health exam. BMI is 32. Drinks and smokes. Loves salty and greasy foods. Pressure is 150/95. What is the most effective means of controlling his pressure in this case?

Weight loss is the most effective means.

Mnemonic for normal-pressure hydrocephalus?

Wet, wacky and wobbly.

How are rashes from disseminated gonococcemia different from allergic interstitial nephritis?

With gonococcemia, the rashes include both apples and pustules with a hemorrhagic component (get picture)

How is is hyperhemolytic crisis different from aplastic crisis?

With hyperhemolytic crisis there is still appropriate reticulocytosis..

How does hepatic disease such as severe cirrhosis lead to renal failure?

With severe liver cirrhosis, increased nitric oxide in the splanchnic circulation causes vasodilation, reducing the peripheral vascular resistance and blood pressure, thus causing renal hypoperfusion.

An acute symmetric arthritis of the hands, knees and ankle joints in a 25 year old woman with a few minor complaints such as loose bowel movements associated with mild skin itching and patchy redness. No history of fever, weight loss or lymphadenopathy. Works as a school teacher. Diagnostic test? Treatment?

anti-parvovirus B19 IgM. Observe as it is self-limited.

What is the best treatment option for a 65 year old with multiple metastatic lesions on MRI of the brain?

Whole brain radiation therapy.

For histoplasmosis are you testing for antibody or antigens in the serum or urine?

antigen

Describe the presentation of idiopathic intracranial hypertension.

Young, obese women with headache, vision changes like blurry vision and diplopia and pulsatile tinnitus.

What are the inciting events of vasovagal syncope in someone less than 60 years old? Older then 60?

Younger than 60 are due to emotional or orthostatic stress such as... 1. venipuncture. 2. prolonged standing. 3. heat exposure. 4. exertion. Older than 60, triggers include... 1. micturition 2. cough. 3. defecation

Dysphagia, halitosis and regurgitation of food particles. Diagnosis, treatment and complication?

Zenker diverticulum. Surgery (no medical therapy available). Aspiration pneumonia.

What HIV medicine causes lactic acidosis?

any nucleoside reverse transcriptase inhibitor.

Describe the walking of a hemiparesis patient?

adducted affected arm and leg extension of affected leg.

What risk factors are associated with anterior spinal cord syndrome?

aortic dissection or aortic surgery.

What is endocardial fibroelastosis? What is the complication of this?

an idiopathic, diffuse fibroelastic thickening of the left ventricular endocardium. It occurs in the first 2 years of life and causes a restrictive cardiomyopathy.

How does diabetic gastroparesis present?

anorexia, nausea, vomiting, early satiety, post prandial fullness and impaired glycemic control.

Medication overdose, tinnitus, tachypnea and fever

aspirin intoxication

What are the first line medications for hypertrophic cardiomyopathy? What antiarrhythmic can be used but isn't first line?

beta blockers and calcium channel blockers (diltiazem or verapamil) Disopyramide, has negative inotropic properties but is a pro-arrhythmiogenic.

Woman from the mediterranean is treated for iron deficiency anemia. A month later there is no change in previous labs. Diagnosis?

beta thalassemia.

Those with hereditary spherocytosis are at an increased risk for what?

bilirubin gallstones and parvovirus B19 infection (aplastic crisis).

How does clopidogrel work?

blocks adenosine diphosphate receptors on the surface of platelets.

Side effect of mycophenolate?

bone marrow suppression.

How are clindamycin and metronidazole similar?

both are active against anaerobic organisms.

Best anxiolytic for general anxiety disorder?

buspirone.

Why is bicarb elevated with patients who vomit or have diarrhea?

contraction alkalosis. Kidneys compensate for hypovolemia and dehydration with aldosterone.

When diagnosing tachycardia-mediated cardiomyopathy. What other disease is on the differential?

coronary artery disease

Patient has congenital adrenal hyperplasia. What medicine is needed?

corticosteroids.

How does anticholinergics work on the bladder? How about neurogenic bladders?

detrusor inactivity. Detrusor-sphincter dyssynergia.

Why is there a prevalence of patients with hyperaldosteronism to develop hypokalemia?

diuretic use in addition to their illness.

Why is primary polydipsia more common in patients with psychiatric conditions?

due to a central defect in thirst regulation as evidence by low serum osmolarity.

How does multiple myeloma present?

elderly man around the age of 65 with normochromic normocytic anemia, fatigue, and bone pain.

other than digoxin's effect on sodium and calcium. What else does it do physiologically? What ADE can it have on people from this mechanism?

enhances vagal tone and slows condition through the AV node. 1. bradycardia in the young 2. ventricular ectopy and tachyarrhythmia in older patients with cardiac disease.

Dusky red, target shaped skin lesions. What is this?

erythema multiforme

How does coccidioidomycosis present?

fever, fatigue weight loss and pleuritic chest pain. Erythema multiforme and erythema nodosum. Arthralgia.

For CSF fluid analysis, what is an elevated opening pressure?

greater than 250mm or 25 cm

What defines fulminant hepatic failure?

hepatic encephalopathy that develops within eight weeks of the onset of acute liver failure.

What is the liver pathology findings of alcoholic hepatitis?

hepatocyte swelling and necrosis, Mallory bodies, and neutrophilic infiltration.

Any time there is a a deep intracranial hemorrhage to structures like the basal ganglia, cerebellum, thalamus or pons, what is the pathophysiology?

hypertensive vasculopathy.

Is diabetes insipidus more commonly seen in infiltrative disorders or pituitary tumors?

infiltrative disorders.

What does cilostazol do? What is it often used for?

inhibits platelet aggregation and causes vasodilation. Chronic claudication (definitely not acute limb ischemia). remember vascular surgery outpatient with this drug?

Why is CT better than abdominal radiographs for diagnosing kidney stones?

it detects radiopaque and radiolucent stones.

Treatment for histoplasmosis is what?

itraconazole.

At what potassium level is insulin with held when managing a patient with DKA?

less than 3.3

What labs would rule in a diagnosis of erogenous insulin use vs insulinoma?

low C-peptide would indicate exogenous insulin use.

How does one diagnose interstitial pulmonary fibrosis and interstitial lung disease due to rheumatoid arthritis?

lung biopsy.

What demographic is associated with ALS?

men ages 40-60.

What is the genetic anomaly in DiGeorge?

microdeletion of 22q11

Anterior mediastinal mass on lateral chest x-ray projection in the setting of systemic muscle weakness.

myasthenia gravis.

What HIV medicine is associated with liver failure?

nevirapine.

Is their pain associated with Mallory-Weiss tears?

no

Does mycoplasma stain well?

no, but you will find PMNs.

Is there an association with nephrotic syndrome and hypoparathyroidism?

no.

Osmolar gap equation? What is a normal osmolar gap.

observed osmolarity minus calculated osmolarity. Normal gap is less than 10.

What amylase levels are associated with perforated esophagus?

over 2,500

What BUN levels are associated with uremic pericarditis?

over 60

Uremic pericarditis is associated with what level of blood urea nitrogen?

over 60.

What are the cutaneous findings associated with granulomatosis with polyangiitis?

painful subcutaneous nodules, palpable purpura and or pyoderma gangrenosum-like lesions.

Epigastric pain and abdominal tenderness

pancreatitis

Prader-willi what is the gene that is lost?

paternal copy of 15q11-q13.

What defines complicated cystitis?

patients with cystitis with risk factors that are concerning for antibiotic resistance such as the following: 1. diabetes. 2. chronic kidney disease. 3. pregnancy. 4. immunocompromised state. 5. urinary tract obstruction. 6. hospital-acquired infection. 7. infection associated with a procedure. 8. indwelling foreign body.

What patients are at increased risk for anaerobic bacterial pneumonia?

patients with neurological disorders (advanced dementia, Parkinson's, and stroke) due to increase risk of aspiration.

Actinomyces treatment?

penicillin

70 year old male admitted to hospital with chest pain. ECG and cardiac enzymes confirm MI and is treated appropriately. Patient complains of leg pain after 4 days. Angiogram shows occluded popliteal artery. Next best step and why?

percutaneous thrombolysis or surgical embolectomy. Patient likely has an emboli from the infarcted heart (stasis and endothelial injury)

What is contraindicated in a patient with dysphagia?

perforation can occur due to endoscopy and NGT placement.

What are the liver pathology findings of primary sclerosing cholangitis?

periductal portal tract fibrosis and segmental stenosis of extrahepatic and intrahepatic bile ducts.

What is dipyridamole and what is it used for?

phosphodiesterase inhibitor which prevents platelet aggregation. Patients already on aspirin who are having another stroke.

Define measurement bias.

poor data collection with inaccurate results.

What is expected to be seen on CSF for a patient with prion disease?

positive 14-3-3 CSF assay.

Patient with renal colic. Urine pH of 4.5 and microhematuria on urinalysis. Stones are analyzed as 100% uric acid. Treatment and why?

potassium citrate because it alkalizes the urine and directly prevents crystallization. hydration. low-purine diet. allopurinol can be added if there are recurrent symptoms despite initial measures.

Risk factor for chorioamnionitis?

prolonged rupture of membranes.

Myasthenia gravis, proximal or distal muscle weakness on onset?

proximal

CT shows wedge-shaped pleural-based opacification in a distribution distal to an pulmonary artery. Diagnosis?

pulmonary embolism

What id D-lactic acidosis?

rare condition in patients with short-bowel syndrome.

Lesion to the left temporal lobe leads to what kind of aphasia?

receptive aphasia.

How do thiazide diuretics precipitate hyperosmolar hyperglycemic state?

reduced intravascular volume, which decreases GFR and activates counter-regulatory hormones.

Why is it 50% of multiple myeloma patients develop some degree of renal insufficiency? List some conditions that are associated with this.

renal insufficiency is likely due to obstruction of the distal and collecting tubules by large laminated casts containing paraproteins (mainly Bence Jones proteins). 1. hypercalcemia. 2. hyperuricemia. 3. amyloid deposition. 4. pyelonephritis. Note: some triggers for renal failure in these patients include infection, dehydration and IV dye.

pH of 7.5, PaCO2 of 30 and HCO3 of 22. What is the acid-base disturbance?

respiratory alkalosis.

What is the best predictor of opioid intoxication?

respiratory rate.

What is the treatment for HCV?

ribavirin, peginterferon and an additional protease inhibitor such as telaprevir or boceprevir. HCV viral load will need monitoring.

Why is a lumbar puncture contraindicated in someone with an epidural abscess?

seeding of bacteria to the CSF.

What are the lab findings that would suggest hypothyroidism-induced hyponatremia? What two other diseases present like this?

serum osmolarity less than 290, urine osmolarity greater than 100, and urine sodium greater than 25.

Presentation for crystal-induced synovitis?

sever pain, redness, swelling and pain with movement at the hands, knee, ankle, elbow and wrist.

What is alprazolam?

short-acting benzodiazepine used in treating panic attacks and anxiety.

How does albumin work in treating bronchospasm?

stimulating beta2-adrenoreceptors.

What neurologic condition is nimodipine good for?

subarachnoid hemorrhage.

Where are Lewy bodies concentrated on pathology?

substantia nigra. locus ceruleus. dorsal raphe. Substantia innominata.

Where is the lesion with a patient who has unilateral, violent arm flinging?

subthalamic nucleus

Bones, stones, abdominal groans and psychic moans

symptomatic hypercalcemia

What is carotid sinus hypersensitivity syndrome?

syncopal episodes triggered by neck movement or pressure on the carotid sinus by tight neck collars.

Symtoms of mumps orchitis?

testicular pain and atrophy as well as low testosterone levels and high FSH and LH levels.

What deficiency causes beriberi? What are the symptoms?

thiamine. Dry beriberi: symmetrical peripheral neuropathy of sensory and motor Wet beriberi: includes dry symptoms and CHF findings

What is metolazone?

thiazide diuretic.

What HEENT exam finding should one look for in someone with suspected of having a seizure?

tongue laceration.

In OBGYN, what is methotrexate used for?

treating ectopic pregnancy

What is esmolol?

ultra-short acting beta blocker used for rapid rate control in atrial flutter of fibrillation.

Patient presents with jaundice. Next best step?

ultrasound.

What are the key findings to hone in on when making an ALS diagnosis?

upper and motor neuron degeneration. Absence of sensory abnormalities and ocular deficits (such as ptosis and diplopia)

75 year old woman with multiple atherosclerotic risk factors has a fall 2 years ago and relies on next of kin for help. Patient has another fall a month ago and has memory impairment, mood swings and urinary incontinence. Diagnosis?

vascular dementia

65 year old woman brought in for cognitive impairment. 3 months ago stopped cooking and cleaning the house. Gait has been unsteady. Two near falls over the last 2 months. History includes hypertension and coronary artery disease. Weakness on left side and prominent pronator drift of left arm. No muscle rigidity. Unsteady in Romberg position. Diagnosis?

vascular dementia..

Treatment choice for aspergillosis?

voriconazole

What is effect modification?

when an external variable positively or negatively impacts the effect of a risk factor on the disease of interest.


Related study sets

Chapter 6: Risk Aversion and Capital Allocation to Risky Assets (Review Questions)

View Set

Rubins - Female Reproductive System

View Set

American Government Chapter 14: Foreign Policy

View Set

Occupational Rehabilitation and Return-to-Work Programming

View Set

The Industrial Revolution (Chapter 9)

View Set